Sei sulla pagina 1di 443

ENDODONTICS

Which of the following situations has the best long term prognosis?
A. Vertical Root Fracture
B. Oblique root fracture extending from middle to apical third
C. Horizontal root fracture in middle third
D. Horizontal root fracture in apical third

Which of the following situations has the best long term prognosis?
A.
B.
C.
D. Horizontal root fracture in apical third

Root
fractu
re

http://www.aae.org/NR/rdonlyres/73E3698B-CABB-4D80-B4C4A9EE07ECDAC9/0/2004TraumaGuidelines.pdf
3-4 radiographs- Straight on PA, Mesial PA, Distal PA, (Occlusal)
Reposition
Splint 3-4 Weeks
Adjust occlusion
F/U at 6weeks, 6 mo, 1year, yearly for 5 years
RCT of coronal segment if necrosis, apical matrix may be needed
SX removal of apical segment if possible or necessary
Andreasen 1967 JOS 4 types of healing/ location of fx did not
determine success, mobility of coronal segment is important
Calcified (callous)
Connective tissue
Bone/connect tissue
Granulation/inflammatory (nonunion)
Barnett & Tronstad et al 1988 Endo Dent Trauma Rigid splinting for 24 months is recommended for horizontal root fractures
Bender JADA 1983 Recommends 3 radiographs with different vertical
angulations to view horizontal fracture
Cvek & Andreasen 2001 Dent Trauma A positive effect of splinting
(various methods) could NOT be demonstrated. In conclusion, the
findings from this study have cast doubt on the efficacy of splinting for
root fracture healing; hard tissue consolidation of a root fracture may
take place more advantageously under functional stress.
Keller 2002 Dent Trauma Clinical and epidemiological study of
traumatic root fractures
34% had no complications
40% developed pulpal necrosis
6% had root resorption
20% developed pulpal canal obliteration
54% no treatment needed

Jacobsen 1975 Long term prognosis of anterior teeth with root


fractures
location of the fracture influenced repair only slightly
longevity (prognosis) of teeth was not shortened even when necrosis
occurs
optimal treatment
reposition
relief of occlusion
Vertical Root Fracture:
Pitts identification requires direct visualization, transillumination, is
an endo-perio lesion. Consider root resection, amputation and
extrusion.
Testori in endodontically treated teeth, occurs most often in
premolars, usually observe narrow periodontal defect.

A positive response in an endodontically treated tooth to the following


endo test would be pathognomonic for a missed canal?
A. Ethyl Chloride
B. EPT
C. Heat Test
D. Percussion

A positive response in an endodontically treated tooth to the following


endo test would be pathognomonic for a missed canal?
A.
B.
C. Heat Test
D.
Endo Vitality testing only assesses the Nerve not the blood supply.
Cold test
a. Peters no damage to tooth from CO2 snow or endo ice
b. Jones & Miller 2004 use large cotton pellet
c. Peterson cold test 90% accurate
d. Trowbridge mode of action hydrodynamics
e. Temperature:
a. Endo Ice -30 degrees centigrade
b. CO2 Snow -78 degrees centigrade
Heat test
A. Schindler used on refractory cases to identify missed canals
or late stage of an irreversible pulpitis.
Vitality testing
Seltzer/Bender no histologic correlation w/diagnostic tests
Friend patient accuracy of etiology only 37%
Reliability
a. Andreasen immature developing teeth, unreliable response to EPT,
use CO2 snow
b. Bhaskar trauma cases, EPT, cold and heat tests unreliable (due
to nerve damage w/out altering blood supply)
c. Fuss unreliable EPT response in presence of large restoration,
endo ice, CO2 and EPT equally reliable in adult teeth.
EPT
a.
Mode of action stimulate A fibers
b.
Technique Wahab slowly increased current is more accurate
c.
Bender test incisal edge in anterior teeth
d.
Jacobson test middle-third of incisors, occlusal-third premolars

Union of two or more teeth via fusion of the cementum only is called
concresence. The most common location for this process is the
mandibular molars.
a. The first statement is TRUE and the second statement is
FALSE.
b. The first statement is FALSE and the second statement is
TRUE
c. Both statements are TRUE
d. Both statements are FALSE

Union of two or more teeth via fusion of the cementum only is called
concresence. The most common location for this process is the
mandibular molars.
a. The first statement is TRUE and the second statement is
FALSE.
b.
c.
d.
Teeth are initially separate and join by excess deposition of cementum
on the roots of closely approximated teeth. Most commonly seen in
Permanent Molars, mainly in the Maxillary Arch. This is different than
fusion: which is a union of 2 tooth germs during development.
Concresence is the union of Cementum ONLY (no enamel or dentin).
Surgical intervention may be necessary in extraction of these teeth.
Source: Fuller, et al. Concise Dental Anatomy and Morphology. 4th
ed.2001. pg196.

After viewing an irregularly shaped moth-eaten radiolucency located in


the middle third of tooth #8, a second X-ray was taken with the tube
head positioned to the distal. The lesion in the second x-ray moved to
the distal.
Which of the following statements would be true of this scenario.
a.
b.
c.
d.
e.

The internal resorption is located on the Facial


The internal resorption is located on the Lingual
The external resorption is located on the Lingual
The external resorption is located on the Facial
A diagnosis of internal or external cannot be made based on a
radiograph

After viewing an irregularly shaped moth-eaten radiolucency located in


the middle third of tooth #8, a second X-ray was taken with the tube
head positioned to the distal. The lesion in the second x-ray moved to
the distal.
Which of the following statements would be true of this scenario.
a.
b.
c. The external resorption is located on the Lingual
d.
e.
Resorption
Asymptomatic
Tooth tests vital
Percussion/Palpation response
Normal
Lesion moves as X-ray tube head
is moved*
Canal can be traced on Xray
Pulpal wall intact
Pink Appearance
Lesion has smooth borders
Lesion has ragged appearance
*- in relation to the canal

Internal
X
X
X

External
X
X
X
X
X
X

X
X
X

Internal resorption:
DX: Results from chronic pulpitis: trauma and infection are important
etiological considerations. Highest incidence: Incisors. Sharp margins.
Symmetrical. Uniform in density. Canal cannot be traced, Balloon
shape stays centered on canals in angled x-rays. Remains vital and
asymptomatic until root is perforated at which time the canal may
become necrotic. May be difficult to detect in post teeth until
Obturation is complete.
TX: Initiate endo ASAP. Resorption process ceases as soon as pulp is
removed. Possible complications: Excessive bleeding, Perforation. Use
CaOH interappointment (Troupe- up to 12 weeks), ultrasonics to clean
and obturate w/ warm gutta percha technique.
External resorption:
DX: More diffuse margins, moth eaten appearance common. Not

symmetrical. Canal can be traced through lesion. Use Buccal object


rule to Dx location. (3 Types)
1-Inflammatory Resorption From pulpal necrosis, trauma, ortho.
2-Replacement Resorption Ankylosis.
3-Extra Canal Invasive Resorption (EICR) Varies: inflammation and/or
replacement.
EICR has a classification system from Heithersay:

EICR Etiology:
Bleaching endodontically treated teeth
Certain systemic diseases
Excessive mechanical or occlusal forces
Idiopathic
Impaction of teeth
Luxation injuries
Periapical inflammation due to a necrotic pulp
Periodontal disease
Radiation therapy
Reimplantation of teeth
Tumors and cysts
Tx for EICR:
Identify & remove cause and predisposing factors
Class 1 Possible RCT and restore defect
Class 2 RCT and restore defect
Class 3 RCT and surgically restore defect
Class 4 Extraction/replace tooth
6 Month Follow up

References:
Gartner, et al. Differential diagnosis of internal and external root
resorption. J Endod (1976)
Andreasen & Andreasen. Essentials of Traumatic Injuries to the
Teeth, pp. 116-9
Heithersay. Invasive Cervical Resorption. Endodontic Topics (2004)
Frank, Torabinejad. Diagnosis and Treatment of Extra Canal
Invasive Resorption. J Endod (1998)
Walker, et al, Color Atlas and Text of Endodontics. pp.201-5
Cohen., Burns. Pathways of the Pulp Eigth Edition. pp. 626-31

In the treatment of avulsion, how should the tooth be placed into the
socket?
A. Forceps
B. Digital pressure
C. Having patient bite down against opposing tooth
D. Having patient bite down on cotton roll/gauze
E. More than one of the above is correct

In the treatment of avulsion, how should the tooth be placed into the
socket?
A.
B. Digital pressure
C.
D.
E.
http://www.aae.org/NR/rdonlyres/73E3698B-CABB-4D80-B4C4A9EE07ECDAC9/0/2004TraumaGuidelines.pdf

Concerning the primary dentition, which procedure is Cvek most


commonly associated with?
a.
b.
c.
d.

A Partial Pulpotomy
A Pulpectomy
Direct Pulp Capping
Indirect Pulp Capping

Concerning the primary dentition, which procedure is Cvek most


commonly associated with?
a. A Partial Pulpotomy
b.
c.
d.
The Cvek pulpotomy (or partial pulpotomy) is when removal of the
coronal pulp tissue is done with a high speed instrument to the level of
a healthy pulp (typically 1-2mm). This procedure is done predictably in
traumatized teeth because we know that that the exposed tissue is
subject to necrotic changes.
Technique: RD, disinfection, 1-2mm deep cavity prepared with high
speed sterile bur (with water spray) <dont use slow-speed or spoon>
If excess hemorrhage is seen remove more pulp until controlled. Rinse
blood with sterile saline or anesthetic. Dry with a sterile cotton pellet.
<Formocresol used to be popular, but now Ferric Sulfate is the material
of choice to address any minimal hemorrhage) Place a thin layer of
hard setting CaOH on pulp. Fill the rest of the chamber prep with Glass
ionomer cement, then restore as you would normally.
CaOH is basic in pH and disinfects dentin and pulp. It serves to reduce
additional pulpal inflammation. The GI cement ensures a bacteria-tight
seal to ensure proper healing.
Reference:
Cohen., Hargreaves. Pathways of the Pulp. 9th Edition. pp.618.

In terms of long-term prognosis, it is better to obturate to


__________________.
a.
b.
c.
d.
e.

0-1 mm from the radiographic apex


Greater than 1 mm from the radiographic apex
Past the radiographic apex
At the radiographic apex
1-3 mm short of the apex

In terms of long-term prognosis, it is better to obturate to


__________________.
a. 0-1 mm short of the radiographic apex
b.
c.
d.
e.

Most studies show that in terms of percentage rates of success,


obturation 0-1 mm short of the radiographic apex is better than
obturation 1-3 mm short of the apex, and both were superior to
obturation beyond the apex. However, some studies have shown no
statistical difference between 0-1 mm short and >1 mm short.
Ref.
1. Naito T. Better success rate for root canal therapy when
treatment includes obturation short of the apex. J Endo, 2005
Apr;31(4):271-4.
2. Schaeffer MA, White RR, Walton RE. Determining the optimal
obturation length: a meta-analysis of literature. J Endod. 2005
Apr;31(4):271-4.

Match the following statements with appropriate technique. Each


choice can only be used three times.
a. Inferior alveolar nerve block
b. Gow-Gates technique
c. Vazirani-Akinosi technique
1.
2.
3.
4.
5.
6.
7.
8.
9.

No bony contact; depth of penetration somewhat arbitrary


Inaccurately referred as mandibular nerve block
True mandibular nerve block
Provide successful anesthesia where a bifid inferior alveolar
nerve and bifid mandibular canals are present.
Targeted area: lateral side of the condylar neck, just below the
insertion of the lateral pterygoid muscle
Indicated for limited mandibular opening
High rate of inadequate anesthesia (15-20%)
Positive aspiration (10-15%, highest of all intraoral injection
techniques)
Provides sensory anesthesia to virtually the entire distribution of
V3: inferior alveolar, lingual, mylohyoid, mental, incisive,
auriculotemporal, buccal nerve

Match the following statements with appropriate technique. Each


choice can only be used three times.
a. Inferior alveolar nerve block
b. Gow-Gates technique
c. Vazirani-Akinosi technique
1.
2.
3.
4.
5.
6.
7.
8.
9.

No bony contact; depth of penetration somewhat arbitrary


Inaccurately referred as mandibular nerve block
True mandibular nerve block
Provide successful anesthesia where a bifid inferior alveolar
nerve and bifid mandibular canals are present.
Targeted area: lateral side of the condylar neck, just below the
insertion of the lateral pterygoid muscle
Indicated for limited mandibular opening
High rate of inadequate anesthesia (15-20%)
Positive aspiration (10-15%, highest of all intraoral injection
techniques)
Provides sensory anesthesia to virtually the entire distribution of
V3: inferior alveolar, lingual, mylohyoid, mental, incisive,
auriculotemporal, buccal nerve

Answer: 1-C, 2-A, 3-B, 4-C, 5-B, 6-C, 7-A, 8-A, 9-B

Inferior alveolar nerve block (IANB) only anesthetizes inferior


alveolar, incisive, mental, and lingual nerves. High failure rate is
deemed to be caused by accessory innervation from cervical nerve
or mylohyoid branch of V3.
Gow-Gates: True mandibular block (see statement #9); needle is
inserted into mucous membrane on the medial of the mandibular
ramus, on a line from the intertragic notch to the corner of the
mouth, just distal to the maxillary second molar in attempt to target
lateral side of the condylar neck
Vazirani-Akinosi: This is also called closed mouth mandibular block.
The targeted area for this technique is soft tissue on the medial
border of the ramus in the region of the inferior alveolar, lingual,
and mylohyoid nerves as they run inferiorly from the foramen ovale
toward the mandibular foramen (the height of injection with the
Vazirani-Akinosi is below that of the GGMNB but above that of the
IANB)
Ref.

1. Stanley F. Malamed, Handbook of Local Anesthesia, 5th ed.


Mosby

What is the treatment protocol for a lateral luxation injury?


a. Reposition, radiograph, stabilize with flexible splint up to 3
weeks, if marginal bone breakdown seen stabilize additional 34weeks
b. Reposition, radiograph, stabilize with flexible splint up to 3 weeks
c. Reposition, radiograph, stabilize with rigid splint up to 3 weeks
d. Reposition, radiograph, stabilize with rigid splint up to 3 weeks, if
marginal bone breakdown
seen stabilize additional 3-4weeks
e. None of the above

What is the treatment protocol for a lateral luxation injury?


a. Reposition, radiograph, stabilize with flexible splint up to 3
weeks, if marginal bone breakdown seen stabilize additional 34weeks
b.
c.
d.
e.
Lateral luxation repositioning is usually a very forceful and, therefore, a
traumatogenic procedure. Prior to this procedure, it is necessary to
anesthetize the area. This type of luxation is characterized by the
forceful displacement of the root tip through the facial alveolar wall,
which complicates the repositioning procedure. In order to dislodge
the root tip from its bony lock, firm digital pressure in an incisal
direction must first be applied immediately over the displaced root,
which can be localized by palpating the corresponding bulge in the
sulcular
fold. Once the tooth is dislodged, it can be maneuvered apically into its
correct position.
If manual repositioning is not possible, a forceps can be applied,
whereby the tooth is first slightly extruded past the bony alveolar lock
and then directed back into its correct position.
Once the tooth is repositioned, the labial and palatal bone plates
should also be compressed, to ensure complete repositioning and to
facilitate periodontal healing. Lacerated gingiva should then be readapted to the neck of the tooth and sutured. The tooth should be
splinted in its normal position. A radiograph is then taken to verify
repositioning and to register the level of the alveolar bone for later
comparison. This is recommended in order to monitor eventual loss of
marginal bone support in the follow-up period.

Ref.
1. Extrusive Luxation and Lateral Luxation F. M. Andreasen
& J. O. Andreasen blackwellpublishing.com

Of the following, which is not a contraindication for internal bleaching?


a.
b.
c.
d.
e.

Severe Dentin loss


Discoloration arising in the pulp chamber
Superficial enamel discolorations
Presence of caries
Defective enamel formation

Of the following, which is not a contraindication for internal bleaching?


a. Discoloration arising in the pulp chamber
b.
c.
d.

Internal bleaching is performed after endodontic therapy is complete.


The indications for this technique are:
Discolorations of pulp chamber origin
Dentin discolorations
Discolorations not amenable to external bleaching.
The technique that should be used in all situations is the walking
bleach technique.
The shade of the teeth is determined and photos made.
The tooth is isolated with a rubber dam and petrolatum placed
on the gingiva to protect it further.
Provisional restoration or old restoration is removed to access the
pulp chamber to a region below the gingival margin.
Glass ionomer, zinc phosphate, polycarboxalate is applied over
the obturating material to prevent leakage
Base needs to be 2mm thick and extend up to or just coronal to
the CEJ
Solvents such as orange oil, chloroform or xylol on a cotton pellet
can remove old sealer on walls
Sodium perborate is mixed with water, saline or anesthetic
solution to the consistency of wet sand and is placed into the
pulp chamber
Excess liquid is removed with cotton pellets
Paste is removed from pulp horn region and undercut areas
Tooth is sealed with a thick mix of IRM
Rubber dam is removed and patient informed that they may not
notice a change for 2 weeks
Reschedule to repeat procedure and evaluation. If a third or
fourth appointment is needed due to a lack in progress, further
treatment may not be beneficial.
Complications seen with internal bleaching include:
Internal resorption

o Chemicals used diffuse into periodontium and cause


necrosis of the cementum and PDL with subsequent root
resorption.
o Extend the bleaching material no further than the cervical
margin as necessary to obtain a satisfactory clinical result
Coronal Fracture
o Increased brittleness of the coronal portion of the tooth is
thought to be due to use of heat to increase bleaching
reaction. This results in desiccation or alteration of the
physiochemical characteristics of the dentin and enamel.
o The above point has not been proved conclusively.
Chemical Burns
o Sodium perborate is safe to use, but if 30% hydrogen
peroxide is used, it will cause burns and sloughing of
tissue. Petrolatum or cocoa butter can be applied to the
gingiva to protect it along with the use of a rubber dam.

Ref.
Rotstein I, Walton RE; Ch. 22 Bleaching Discolored Teeth: Internal and
External; Endodontics: Principles and Practice; Saunders Publishing, St.
Louis, MO, 2008; pp. 398-401.

Which of the following statements is false?


a. Flexible SS files with a non-cutting tip are superior to
conventional SS files, NiTi files, or NiAl files when compared by
cutting efficiency and instrumentation of curved canals.
b. NiTi K-files are extremely flexible and are especially useful for
apical enlargement in severe
apical curves.
c. K-files are manufactured by twisting square or triangular metal
blanks along their long axis, producing partly horizontal cutting
blades.
d. Because of their flexibility, the smaller NiTi files (sizes up to #25)
are especially useful.

Which of the following statements is false?


a.
b.
c. Because of their flexibility, the smaller NiTi files (sizes up to #25)
are especially useful.
d.

Review of the different types of hand instruments available according


to alloy composition and geometric configuration:
K-reamers are more flexible and fracture resistant but offer no
advantages to K-files
Hedstrom files are preferred for maximum cutting efficiency when
enlarging the straight coronal portion of a canal. Safety Hedstrom
files have a non-cutting surface to prevent ledging in curved canals.
Flexible SS files with a non-cutting tip are superior to conventional
SS files, NiTi files, or NiAl files when compared by cutting efficiency
and instrumentation of curved canals.
Half-sized instruments from #12-#37 can be helpful in narrow
sclerotic canals.
ProFile Series 29 provide a constant percentage increase of 29%
between successive instruments, as compared to the variable
percentage increase with ISO instruments.
No file produces an entirely centered enlargement of the curved
canal.
No individual file or filing system is superior and it is up to the
provider to judge each case individually and take advantage of the
system that meets that particular need.
Ref.
Schafer E. Root canal instruments for manual use: a review. Endod
Dent Traumatol
1997;13: 51-64

A horizontal root fracture is sometimes difficult to visualize


radiographically. The recommended series of radiographs for diagnosis
of a horizontal fracture is:
a. One periapical radiograph, perpendicular to the long axis of the
tooth
b. Two periapical radiographs: one perpendicular, one with a mesial
horizontal shift
c. Three periapical radiographs: one perpendicular, one with a
mesial horizontal shift, one with a distal horizontal shift
d. Three periapical radiographs: one perpendicular, one with a
horizontal shift, one with a vertical shift
e. Three periapical radiographs: one perpendicular, two with
positive and negative vertical shift

A horizontal root fracture is sometimes difficult to visualize


radiographically. The recommended series of radiographs for diagnosis
of a horizontal fracture is:
a.
b.
c.
d.
e. Three periapical radiographs: one perpendicular, two with
positive and negative vertical shift

A minimum of 3 PA radiographs should be taken: perpendicular to the


tooth (90) and two with vertical shift (45 and 110) to visualize an
oblique fracture.
The angulation of the cone in radiographic
detection of a horizontal root fracture is critical. In
order to successfully diagnosis a horizontal root
fracture the cone must be between 20 degrees and
+10 degrees. Therefore, if you suspect a horizontal
root fracture, it is a good idea to take multiple
radiographs (see Figure 2).

FIGURE 2: Radiographs taken at various angles to a fracture (top


row) produce images that reveal the fracture to varying degreesor
not at all.
Ref.:
1. Cohen and Burns. Pathways of the Pulp, 7th ed., pp. 567-71.
1. Claudia Hoffman, D.D.S. Alternatives to Management of a

Horizontal Root Fracture (Link to the article:


http://www.endomail.com/articles/ch01alternatives.html)
Recommended guidelines of the American Association of Endodontists
for the treatment of traumatic injuries
Root fracture: Take four radiographs (1-4).
Radiographs: (1) occlusal (2) periapical central angle (3) periapical
mesial eccentric (4) periapical distal eccentric.
Link to the AAE Guidelines and position statements:
http://www.aae.org/dentalpro/EducationalResources/guidelines.htm

Which of the following is not a possible cause of ankylosis?


a.
b.
c.
d.

Damage to more than 20% of the periodontal ligament


Replanted avulsed tooth
Taurodontism
Trauma with severe intrusion of the tooth

Which of the following is not a possible cause of ankylosis?


a.
b.
c. Taurodontism
d.
Ankylosis (replacement resorption) occurs most often when a tooth has
been avulsed (knocked out completely) and replanted; or severely
intruded, in such a way that its periodontal ligament is essentially
destroyed. When this happens, the tooth's cementum layer may fuse
directly to the tooth-supporting alveolar bone.
The initial reaction in response to the traumatic event is inflammation,
resulting with se vere traumatic injury in an area of the root being
devoid of cementum. Cells in the vicinity of the root now compete to
repopulate it and sometimes osseous cells migrate faster than the
periodontal ligament cells; in these situations bone comes into contact
with the root without an intermediate attachment apparatus, resulting
in dentoalveolar ankylosis.
Ref.:
http://www.cda-adc.ca/jcda/vol-71/issue-10/763.html
Pathways of the Pulp; Mosby Publishing, St. Louis, MO, 2006; pp. 630631

EDTA is a chelating agent that decalcifies dentin and removes smear


layer. Before obturation of the canal system EDTA should be used last,
after irrigation with NaOCl is completed.
a.
b.
c.
d.

Both statements are true


Both statements are false
First statement is true and second is false
First statement is false and second is true

EDTA is a chelating agent that decalcifies dentin and removes smear


layer. Before obturation of the canal system EDTA should be used last,
after irrigation with NaOCl is completed.
a.
b.
c. First statement is true and second is false
d.
With its ability to chelate inorganic material, it removes mineral from
the smear layer while NaOCl digests organic material from the smear
layer and kills bacteria and also digests organic debris.
EDTA (ethylenediaminetetraacetic acid) is available in liquid or paste
form and is used in concentrations from 15-17% It is usually combined
with a detergent to decrease the surface tension to increase the
cleaning ability and wetability of the dentin surface. To remove smear
layer the EDTA needs to be left in place for 1-5 minutes and then
rinsed out of the tooth with water or NaOCl.
Nygaard-Ostby in 1957 introduced chelating agents to endodontics for
the treatment of narrow calcified canal systems. It allows for an easier
cutting of the calcified dentin and its removal to aide in treatment.
EDTA should never be used as the final irrigant; while it does dissolve
the smear layer and help cleanse canal walls, it will remain active in
the canal system for 5 days and continue to demineralize tooth
structure if a subsequent sodium hypochlorite rinse is not performed.

Ref.:
1. Walton RE, Torabinejad M. Principles and practice of
endodontics, 3rd Ed. W.B. Saunders Co. 2002. pp. 218-220,
351-360.
2. Johnson WT, Gutmann JL; Ch. 10 Obturation of the Cleaned and
Shaped Root Canal
3. Pathways of the Pulp; Mosby Publishing, St. Louis, MO, 2006; pp.
366-367

All of the following EXCEPT one may be indicative of multiple canals in


a maxillary second premolar. Which is the exception?
a.
b.
c.
d.
e.

Sharp change in canal density - fast break


Root outline unclear
Root has unusual contour
Root differs from expected appearance
Canal is well centered in root

All of the following EXCEPT one may be indicative of multiple canals in


a maxillary second premolar. Which is the exception?
a.
b.
c.
d. Canal is well centered in root
Detecting extra-canals:
Sharp change in canal density fast break
Root outline unclear
Root has unusual contour
Root differs from expected appearance
Canal is not centered in root
Second premolar - Average length - 21.0 mm
- Morphology One canal / one foramen - 48%
Two canals / one foramina - 27%
Two canals / two foramina - 24%
Three canals
- 1%
- Access - ovoid

Example of fast breaking canal.


Ref.: Slowey RR (1974)

Vertucci FJ, Seelig A, Gillis R. Root canal morphology of the human maxillary
second premolar.
Oral Surg 1974;38:456-64.
Maxillary second premolar
In 40% of cases, this tooth, which is similar in length to the first
premolar, has one root with a single canal. Two canals may be found in
about 58% of cases.4 The configuration of the two canals may vary
with two separate canals and two exits, two canals and one common
exit, one canal dividing and having two exits. In one study,5 it was
found that 59% of maxillary second premolars had accessory canals.
As with the first maxillary premolar, the apical third of the root may
curve quite considerably, mainly to the distal, sometimes buccally. The
access cavity is similar to the first premolar.

British Dental Journal 197, 379 - 383 (2004)


www.nature.com/bd/journal/v197/n7/full/4811711a.html

The process of induced root development or apical closure of the root


by hard barrier deposition is known as:
a.
b.
c.
d.

Cvek pulpotomy
Apexification
Apexogenesis
Formocresol pulpectomy

The process of induced root development or apical closure of the root


by hard barrier deposition is known as:
a.
b.
c.
d.

Cvek pulpotomy
Apexification
Apexogenesis
Formocresol pulpectomy

Apexification involves complete pulpal debridement and placement of


calcium hydroxide to encourage the formation of an apical stop. The
canal dressing is changed every few months to freshen its potency.
Once an apical stop is formed, conventional obturation is indicated.
Historically calcium hydroxide has been the material of choice during
treatment.
Apexogenesis
A vital pulp therapy procedure performed to encourage continued
physiological development and formation of the root end.
Apexogenesis is indicated in immature vital teeth. The goal of therapy
is to preserve vital (and healthy) radicular pulp tissue and promote
physiologic closure of the root end. It is contraindicated in teeth with
signs of pulp necrosis (percussion sensitivity, periapical lesions, sinus
tracts, etc.).

Cohn S, Hargreaves, Pathways of the Pulp, Ninth Edition, 2006.


Used in apexification, Mineral trioxide aggregate (MTA) has the ability
to encourage hard tissue deposition similar to Calcium hydroxide
effect. Also both have the same biological and histological properties.
MTA is a cement composed of tricalcium silicate, dicalcium silicate,
tricalcium aluminate, tetracalcium aluminoferrite, calcium sulfate and
bismuth oxide. An independent analysis we requested from a
laboratory reveals that MTA is identical to Portland's cement except for
the addition of bismuthoxide, believed to help modifying its setting
properties.

It is composed of 1. tricalcium silicate, 2. dicalcium silicate, 3.


tricalcium aluminate, 4. tetracalcium aluminoferrite, 5. calcium sulfate
and 6. bismuth oxide. It was developed by the research pioneer and
Endodontist, Dr. Mahmoud Torabinejad at Loma Linda University.
1.Biocompatible with periradicular tissues, 2.non-toxic, 3.nonresorbable and 4.minimal or no leakage around the margins. 5. It is
very alkaline material. 6. As a root-end filling material MTA shows less
leakage than other commonly used root-end filling materials. Which
means bacterial penetration will be less when using MTA.
MTA usage in some clinical cases:
Pulp capping
Internal and external root resorption
Lateral or furcation perforation
Apexification
When the root is not completely formed in case of vital pulp: 1.
Isolate the tooth with a rubber dam 2. Perform a pulpotomy procedure.
3. Place the MTA over the pulp and close the tooth with temporary
cement until the apex is completely formed.
In case of non-vital pulp: 1. Isolate the tooth with a rubber dam 2.
perform root canal treatment. 3. Mix the MTA and plug it down to the
apex of the tooth, creating a 2 mm thickness of plug. 5. Wait for it to
set; then fill in the canal with cement and gutta percha.
References
Rafter M. Apexification: a review. Dental Traumatology. 2005;
21: 1-8.
Walton RE, Torabinejad M. Principles and practice of
endodontics, 3rd Ed. W.B. Saunders Co. 2002. pp. 218-220,
351-360.
Ref: Cohen S. and Hargreaves, K.: Pathways of the Pulp, 9th
edition, 2006
Apexogenesis: p.864-866
Apexification: p.621-622
http://www.endoweb.com/dentist/nah_ber1.htm
Successful apexogenesis is indicated by the continued formation the
apex.

Apexogenesis relies on the apical cell rich zone of the apical papilla.

Figure 1
Apical papilla. (A) An extracted human third molar depicting three
immature roots with two pieces of apical papilla being removed from
their apices (arrow heads) and one piece of apical papilla being peeled
away from the root end but not completely detached (arrow). (B) A
developing root tip with attached apical papilla was cultured in vitro for
3 days before being processed for hematoxylin and eosin (H&E)
staining. Odontoblasts (black arrows), apical cell-rich zone (open
arrowheads), and apical papilla tissue are indicated. (C) Magnified view
of the area indicated by the yellow rectangle.
J Endod. 2008 June; 34(6): 645651.

The causes of endodontic failure may include all but one of the
following:
a.
b.
c.
d.
e.

another tooth
dilaceration
overfill
perforation
periodontal disease

The causes of endodontic failure may include all but one of the
following:
a.
b.
c.
d.
e.

another tooth
dilaceration
overfill
perforation
periodontal disease

The mnemonic POOR PAST can be of use to the dentist in recalling the
list of alternatives in the differential diagnosis of endodontic failures:
P--perforation;
O--obturation;
O--overfill;
R--root canal missed;

P--periodontal disease;
A--another tooth;
S--split;
T--trauma.

With a comprehensive list of diagnoses, recall of the appropriate


questions and tests in examining for each possibility is simplified. The
diagnostician then proceeds to look for a specific symptom or
symptoms that would confirm a particular diagnosis or that would limit
the alternatives to two or three possibilities. Questioning about the
nature and duration of the symptoms is useful in ruling out some
diagnoses. Pulp tests, periodontal probing, radiographs from different
angles, percussion, palpation, and sinus tract exploration each have
special application to one or more of the possibilities on the list. The
wise clinician also evaluates access preparations, occlusion, and those
etiologic factors that could cause pulp or periodontal disease.
Variations in the root canal, requirements of treatment, the nature of
the healing process, expected results, and surgical principles are
considered along with the foregoing in problem solving. After a case
has been thoroughly investigated and if the POOR PAST alternatives
have been ruled out the dentist may then conclude that for some as
yet undetermined reason the lesion exceeds the recuperative powers
of the host and that further consultation or surgical intervention will be
necessary.
Toronto Study established in 1993, to prospectively assess the 4-6 year
outcome of endondontic treatment delivered in the university graduate
clinic environment.
- Healed rate was significantly higher for teeth without apical
periodontitis (92%)
- With apical periodontitis healing was 74%
- Overall, healed rate was 81%

Ref.:
Friedman S, Abitbol S, et al, J Endon 2003; 29: 787-793
M C Crump. Differential diagnosis in endodontic failure, Dental clinics
of North America. 01/11/1979; 23(4):617-35.

The Balanced Force technique is considered by most to be the most


effective method of cutting dentin. Advocated by Roane, includes all of
the following except?
a.
b.
c.
d.
e.

Flex-R files used sequentially


Essentially a step-back preparation technique
Clockwise/counter-clockwise filing motion
Instrumentation to the periodontal ligament
Flaring of canals with Gates-Glidden drills is recommended

The Balanced Force technique is considered by most to be the most


effective method of cutting dentin. Advocated by Roane, includes all of
the following except?
a.
b. Essentially a step-back preparation technique
c.
d.
e.

Balanced force
Balanced force motion is a most effective way to cut dentin.
Instrument is rotated clockwise one quarter turn with gentle inward
pressure to pull file into
canal and engage dentin surface.
Instrument is rotated counter-clockwise (one third to two full turns)
with apical pressure to
shear off dentin.
A passive non-cutting clockwise rotation loads newly cut dentin
into file flutes.
Instrument and loaded debris are removed from canal.
Roane JB, Sabala CL, Duncanson MG. The "balanced force"
concept for instrumentation of curved canals. J Endodon
1985;11:203-11.

Rationale and technique of balanced force technique is presented.


Flex-R (alternate is Flexofile Dentsply Maillefer) files with noncutting
tip are used. Control the restoring forces exerted by the
instrument by modifying the reaming technique so curved canals can
be enlarged without transportation.
Sepic AO,Balanced force concept using Flex-R-Files. After 12 years of
experimentation, Roane and Sabala introduced their Balanced Force
concept of root canal preparation. The concept came to fruition, they
claim, with the introduction of new K-type file design the Flex-R-File
(Rhomboid). Initially, the authors developed a reaming action using
clockwise insertion followed by counterclockwise action.

Essentially the whole preparation is a step-down technique ( not a


step-back preparation) in nature beginning with flaring of the coronal
and mid-thirds of the canal with Gates Glidden drills sizes 1 through 6 (
I would not use a 1 because the y have a tendency to separate). This
essentially increases the radius and decreases the arc or closure of the
canal, thus making it straighter and more accessible to reaming
instruments.
At this point, the balanced force instrumentation begins. It involves
placement, cutting and removal using only rotary motions. Insertion is
done with half-turn clockwise motion with slight apical pressure.
Cutting is accomplished using counterclockwise rotation, again with
apical pressure adjusted to match the files strength, i.e. very light for
fine instruments and heavy for large instruments. Clockwise which
sets the instrument, should never exceed 180 degrees; otherwise the
instrument will start to unwind. Counterclockwise rotation, with apical
pressure, is 120 degrees or greater. This rewinds the instrument and
enlarges the canal to the full instrument diameter, a size that was
established by the counterclockwise twisting during manufacture. In
this way (clockwise insertion and counterclockwise cutting and
removal), the instruments advance toward the apex. Continuing this
technique, the clinician enlarges the canal by advancing up the scale
of larger and larger instruments.
The non-cutting tip and first blades of the Flex-R-Files prevents the
instruments from gouging into the curved walls allowing the file to hug
the inside of the curve and prevent tip transport toward the external
curve. When enlargement has been accomplished, a final clockwise
cleaning rotation is used to load canal debris into flutes and to elevate
that debris away from apical foramen. Irrigation follows
Roane firmly believes in enlarging the apical area to sizes larger than
generally recommended- up to size 80 in a single canal, for example
and size 45 for multiple canal teeth. These sizes are not absolute, of
course, and final sizing depends upon root bulk and/or fragility, or the
extreme curvature of a canal. He also believes in carrying the
preparation through to full length, the radiographic apex of the root.
He purposely violates the apical area and rarely gets flareups. For
examples, the No. 30 instrument is carried to full root length and
larger instruments are stepped back from that -No 40 at 0.5 mm back,
No 45, at 1.0 mm back and back up the canal until the Gates Glidden
preparation is reached. NaOCl irrigation is used.
Ref.
Cohen S. and Burns R. Pathways of the Pulp. 7th edition. 1998.
Mosby. 216-218, 244-248.

ORAL MEDICINE, ORAL


PATHOLOGY
Which of the following is not associated with pernicious anemia?
a.
b.
c.
d.

Vitamin B-12r
Early onset in puberty
Intrinsic factor answer
Neurologic symptoms

Which of the following is not associated with pernicious anemia?


a.
b. Early onset in puberty
c.
d.

Little, JW, Falace, DA, Miller, CS, Rhodus, NL, Dental Management Of
The Medically Compromised Patient. 5th edition. Mosby-Year Book,
Inc., pg 496.
Pernicious anemia is due to a deficiency of intrinsic factor, the substance
secreted by the parietal cells of the stomach that is necessary for the
absorption of vitamin B12, which is needed for the maturation of red
blood cells.
is usually a disease of late adult life. most often occurs in 40-yearold to 70-year-old northern Europeans of fair complexion, with one
notable exception. (early onset in black American women <40 years
old)
Early symptoms include weakness, fatigue, palpitations, syncope,
tingling of the fingers and toes (paresthesias), numbness,
uncoordination, and muscular weakness.
Anemia: A reduction in the oxygen-carrying capacity of the blood and
usually is related to a decrease in the number of circulating red blood
cells or to an abnormality in the hemoglobin contained within the red
blood cells. Anemia is not a disease but rather a symptom complex
that may result from decreased production of red blood cells (from Fe
deficiency, pernicious anemia, folate deficiency), blood loss, or
increased rate of destruction of circulating red blood cells.
Leukopenia: A lack of sufficient leukocytes (white blood cells).
Agranulocytosis: A reduction of or lack of neutrophils is the hallmark
of agranulocytosis. Like many blood dyscrasias the decrease of
granulocytes can manifest as primary (unknown etiology) or secondary
(usually as a reaction to a drug or chemical compound).
Thrombocytopenia: Decrease in the number of circulating platelets.
The primary form (idiopathic purpura) is conjectured to be of
autoimmune etiology with an antiplatelet globulin identified in some
but not all cases. The secondary form is precipitated by numerous
agents among which are ionizing radiation, a wide spectrum of drugs,
congenital disorders, infectious viruses and marrow replacing diseases.
Ref.
Shafer, Hine, Levy. A Textbook of Oral Pathology, Fourth Edition, 1983.

Which of the following statements is correct regarding basal cell


carcinoma (BCC)?
a. BCC is a common, locally destructive, non-metastasizing benign
tumor of skin
b. BCC rarely occurs on the sun-exposed skin
c. The most common site of metastasis of BCC is the lungs.
d. Oral lesions are common findings
e. The skin of the head and neck is the most common site.

Which of the following statements is correct regarding basal cell


carcinoma (BCC)?
a.
b.
c.
d.
e. The skin of the head and neck is the most common site.

Basal cell carcinoma (BCC) is the most common skin cancer in humans.
Basal cell cancer tumors typically appear on sun-exposed skin, are
slow growing, and rarely metastasize. Neglected tumors can lead to
significant local destruction and even disfigurement.
Pathophysiology
Although the exact etiology of basal cell carcinoma is unknown, a wellestablished relationship exists between basal cell carcinoma and the
pilosebaceous unit, as tumors are most often discovered on hairbearing areas. Tumors are currently believed to arise from pluripotent
cells (which have the capacity to form hair), sebaceous glands, and
apocrine glands. Tumors usually arise from the epidermis or the outer
root sheath of a hair follicle.
Frequency
United States
Each year in the United States, 900,000 people are diagnosed with
basal cell carcinoma (550,000 male, 350,000 female). The estimated
lifetime risk of basal cell carcinoma in the white population is 33-39%
for men and 23-28% for women.
Mortality/Morbidity
Although basal cell carcinoma is a malignant neoplasm, it rarely
metastasizes. The incidence of metastatic basal cell carcinoma is
estimated at less than 0.1%. The most common sites of metastasis are
the lymph nodes, the lungs, and the bones.1 Typically, basal cell tumors
enlarge slowly and relentlessly and tend to be locally destructive.
Periorbital tumors can invade the orbit, leading to blindness, if
diagnosis and treatment are delayed. Perineural invasion can occur,
leading to loss of nerve function.
Race

Although basal cell carcinoma is observed in people of all races and


skin types, it is most often found in light-skinned individuals; darkskinned individuals are rarely affected.
Sex
Historically, men are affected twice as often as women. The higher
incidence in men is probably due to increased recreational and
occupational exposure to the sun, although these differences are
becoming less significant with changes in lifestyle.
Age
The likelihood of developing basal cell carcinoma increases with age.
With the exception of basal cell nevus syndrome, basal cell carcinoma
is rarely found in patients younger than 40 years.

Clinical
History
Patients often complain of a slowly enlarging lesion that does not heal
and that bleeds when traumatized. As tumors most commonly occur on
the face, patients often give a history of an acne bump that
occasionally bleeds.

Patients often have a history of chronic sun exposure.


o

Recreational sun exposure (eg, sunbathing, outdoor sports,


fishing, boating)

Occupational sun exposure (eg, farming, construction)

Occasionally, patients have a history of exposure to ionizing


radiation. X-ray therapy for acne was commonly used until 1950.

Occasionally, patients have a history of arsenic intake; arsenic is


found in well water in some parts of the United States.

Physical
Clinical presentation of basal cell carcinoma varies by type.

Nodular basal cell carcinoma


o

Nodular basal cell carcinoma is the most common type of


basal cell carcinoma and usually presents as a round,

pearly, flesh-colored papule with telangiectases. As it


enlarges, it frequently ulcerates centrally, leaving a raised,
pearly border with telangiectases, which aids in making the
diagnosis.
o

Most tumors are observed on the face, although the trunk


and extremities also are affected.

Cystic basal cell carcinoma


o

An uncommon variant of nodular basal cell carcinoma,


cystic basal cell carcinoma is often indistinguishable from
nodular basal cell carcinoma clinically, although it might
have a polypoid appearance.

Typically, a bluish-gray cyst-like lesion is observed. The


cystic center of these tumors is filled with clear mucin that
has a gelatin-like consistency.

Pigmented basal cell carcinoma


o

Pigmented basal cell carcinoma is an uncommon variant of


nodular basal cell carcinoma that has brown-black macules
in some or all areas, often making it difficult to differentiate
from melanoma.

Typically, some areas of these tumors do not retain


pigment; pearly, raised borders with telangiectases that
are typical of a nodular basal cell carcinoma can be
observed. This aids clinically in differentiating this tumor
from a melanoma.

Morpheaform (sclerosing) basal cell carcinoma


o

Morpheaform basal cell carcinoma is an uncommon variant


in which tumor cells induce a proliferation of fibroblasts
within the dermis and an increased collagen deposition
(sclerosis) that clinically resembles a scar. The tumor
appears as a white or yellow, waxy, sclerotic plaque that
rarely ulcerates.

Because the tumor infiltrates in thin strands between


collagen fibers, treatment is difficult, and the clinical
margins are difficult to distinguish. Mohs micrographic
surgery is the treatment of choice for this type of basal cell
carcinoma.

Superficial basal cell carcinoma


o

Superficial basal cell carcinoma clinically appears as an


erythematous, well-circumscribed patch or plaque, often
with a whitish scale. Occasionally, minute eschars may
appear within the patch or plaque.

The tumor appears multicentric, with areas of clinically


normal skin intervening among clinically involved areas.

Causes
The exact cause of basal cell carcinoma is unknown, although
environmental factors that are believed to predispose patients to this
disorder include the following:

Exposure to sunlight, the most frequent association (UVB, 290320 nm, which causes sunburn, is believed to play a greater role
in the development of basal cell carcinoma than UVA.)

Exposure to artificial ultraviolet light (eg, tanning booths,


ultraviolet light therapy)

Ionizing radiation exposure (eg, x-ray therapy for acne)

Arsenic exposure through ingestion (eg, arsenic used as a


medicinal agent, predominantly Fowler's solution of potassium
arsenite that was used to treat many disorders, including asthma
and psoriasis). A contaminated water source has been the most
common source of arsenic ingestion.

Immunosuppression: Immunosuppression has been associated


with a modest increase in the risk of basal cell carcinoma.
Therefore, recipients of organ or stem cell transplants have a
higher lifetime risk of developing basal cell carcinoma.

Xeroderma pigmentosum: This autosomal recessive disease


results in the inability to repair UV-induced DNA damage.
Pigmentary changes are seen early in life followed by the
development of basal cell carcinoma, squamous cell carcinoma,
and malignant melanoma. Other features include corneal
opacities, eventual blindness, and neurological deficits.

Nevoid basal cell carcinoma syndrome (basal cell nevus


syndrome, Gorlin syndrome): This autosomal dominant disorder
results in the early formation of multiple odontogenic

keratocysts, palmoplantar pitting, intracranial calcification, and


rib anomalies. Various tumors such as medulloblastomas,
meningioma, fetal rhabdomyoma, and ameloblastoma also can
occur.

Bazex syndrome: Features include follicular atrophoderma ("ice


pick" marks, especially on dorsal hands), multiple basal cell
carcinomas, and local anhidrosis (decreased or absent sweating).

History of previous nonmelanoma skin cancer (basal cell


carcinoma or squamous cell carcinoma): Persons who have been
diagnosed with one nonmelanoma skin cancer are at increased
risk of developing additional tumors in the future. The risk of
developing new nonmelanoma skin cancers is reported to be
35% at 3 years and 50% at 5 years after an initial skin cancer
diagnosis.

References and Additional Resources:


Sapp JP et al, Contemporary Oral and Maxillofacial Pathology, Mosby
Akinci M, Aslan S, Markoc F, et al. Metastatic basal cell carcinoma. Acta
Chir Belg. Mar-Apr 2008;108(2):269-72. [Medline].
Orengo IF, Salasche SJ, Fewkes J, et al. Correlation of histologic
subtypes of primary basal cell carcinoma and number of Mohs stages
required to achieve a tumor-free plane. J Am Acad
Dermatol. Sep 1997;37(3 Pt 1):395-7. [Medline].
Miller BH, Shavin JS, Cognetta A, et al. Nonsurgical treatment of basal
cell carcinomas with intralesional 5-fluorouracil/epinephrine injectable
gel. J Am Acad Dermatol. Jan 1997;36(1):72-7. [Medline].
Greenway HT, Cornell RC, Tanner DJ, et al. Treatment of basal cell
carcinoma with intralesional interferon. J Am Acad
Dermatol. Sep 1986;15(3):437-43. [Medline].
Von Hoff DD, et al. "Efficacy data of GDC-0449, a systemic Hedgehog
pathway antagonist, in a first-in-human, first-in-class phase I study with
locally advanced, multifocal, or metastatic basal cell carcinoma
patients". AACR Meeting. 2008;Abstract LB-138.
Garber K. Hedgehog drugs begin to show results. J Natl Cancer
Inst. May 21 2008;100(10):692-7. [Medline].
[Best Evidence] Eigentler TK, Kamin A, Weide BM, et al. A phase III,
randomized, open label study to evaluate the safety and efficacy of
imiquimod 5% cream applied thrice weekly for 8 and 12 weeks in the

treatment of low-risk nodular basal cell carcinoma. J Amer Acad


Dermatol. Oct 2007;57(4):616-21. [Medline]. [Full Text].
Glass LF, Fenske NA, Jaroszeski M, et al. Bleomycin-mediated
electrochemotherapy of basal cell carcinoma. J Am Acad
Dermatol. Jan 1996;34(1):82-6. [Medline].

Which of the following statement is incorrect regarding oral


contraceptives?
a When prescribing antibiotics, a dentist should advise the patient
to maintain compliance with oral contraceptives when
concurrently using antibiotics.
b Oral contraceptives can exacerbate patients inflammatory
status, causing erythema and an increase tendency towards
gingival bleeding.
c Measurable changes have been observed in the salivary
components and flow in women
taking contraceptive
medications
d These drugs may increase the incidence of local alveolar osteitis
after extraction of teeth.

Which of the following statement is incorrect regarding oral


contraceptives?
a
b Oral contraceptives can exacerbate patients inflammatory
status, causing erythema and an increase tendency towards
gingival bleeding.
c
d

Oral Manifestations
Oral contraceptives can exacerbate patients inflammatory status,
causing erythema and an increase tendency towards gingival bleeding.
In some instances, oral contraceptives have been reported to induce
gingival enlargement.
All studies recording changes in gingival tissues associated with oral
contraceptives were completed when contraceptive concentrations
were at much higher levels than are available today. A recent clinical
study evaluating the effects of oral contraceptives on gingival
inflammation in young women found these hormonal agents to have no
effect on gingival tissues. From these data, it appears that current
composition of oral contraceptives probably are not as harmful to the
periodontium as were the early formulations. Nonetheless, a
controlled oral hygiene program that includes regular oral
examinations, professional cleanings and plaque control will minimize
the effects of oral contraceptives. These drugs may also increase the
incidence of local alveolar osteitis after extraction of teeth.
Reports have shown significant increased risk for developing
myocardial infarction and strokes in women who concomitantly smoke
and take oral contraceptives. This may be a more important issue
among women older than 30 years.
Saliva
Measurable changes have been observed in the salivary components
and flow in women taking contraceptive medications. These changes
include a decrease in concentrations of protein, sialic acid, hydrogen
ions and total electrolytes. Studies have shown both an increase and
decrease in salivary flow.
Localized osteitis (dry socket)

It has been reported that women taking contraceptives may


experience a higher incidence of localized osteitis following extraction
of teeth. However, no additional preventive procedures are
recommended at the time of extraction and treatment for patients
developing localized osteitis is according to the clinicians dry socket
protocol
Interaction between oral contraceptives and antibiotics
Antibiotic interference with contraceptive medication levels is
controversial. Although results from animal studies support antibiotic
interference with contraceptive levels, studies in humans have
presented conflicting results. For most antibiotics, the mechanism of
interference is at the level of the enterohepatic recirculation of the
contraceptives.
DENTAL MANAGEMENT
A comprehensive medical history and assessment of vital signs,
including blood pressure, are extremely important in this group of
patients. Treatment of gingival inflammation exaggerated by oral
contraceptives should include establishing an oral hygiene program
and eliminating all local predisposing factors. Periodontal surgery may
be indicated if there is inadequate resolution after initial therapy
(scaling, root planning and curettage). Antimicrobial mouthwashes
may be indicated as part of the home care regimen.
A recent report from the ADA council on Scientific Affairs noted that,
considering the possible consequences of an unwanted pregnancy,
when prescribing antibiotics to a patient using oral contraceptives, the
dentist should:

Advise the patient to maintain compliance with oral


contraceptives when concurrently using antibiotics.
Advise the patient of the potential risk for the
antibiotics reduction of the effectiveness of the oral
contraceptive.
Recommend that the patient discuss with her
physician the use of an additional nonhormonal
means of contraception

Although in the literature, oral manifestations have been attributed to


oral contraceptive use, it can be presumed that the same effects could
occur with the use of other contraceptive medications (e.g., implants,
transdermal patches).

Choose the correct statement regarding arthrography of the


temporomandibular joint.
a. A contrast medium is injected into the temporomandibular joint
spaces to outline important hard tissue structures
b. Single-contrast arthrography is more accurate than doublecontrast arthography
c. Double contrast arthrography involves injecting air into the joint
space
d. An advantage of arthrography is the low levels of radiation
required for the procedure

Choose the correct statement regarding arthrography of the


temporomandibular joint.
a.
b.
c. Double contrast arthrography involves injecting air into the joint
space
d.

The technique of TMJ arthrography was introduced in the 1940s


(Nrgaard 1947), but it was not extensively used until the late 1970s
(Westesson et al. 1980, Westesson 1993). Arthrography improved
rapidly after the late 1970s (Wilkes 1978, Farrar & McCarty 1979,
Katzberg et al. 1979). There are two technical methods for
arthrography of TMJ. In single-contrast arthrography, radiopaque
material is injected into either the lower or upper joint space, or into
both compartments (Wilkes 1978a,b, Brand 1990). In double-contrast
arthrography, a small amount of air is injected into the joint space after
the injection of contrast materials (Westesson et al. 1980, Westesson
1982, Ma 1987). A comparative study reported that there was no
statistically significant difference in the diagnostic accuracy between
these two techniques (Westesson & Bronstein 1987).
Several studies have shown that arthrography is an accurate imaging
method for evaluating anterior disc displacement. The accuracy for
diagnosing the position of the disc ranged from 84% to 100%
compared with the corresponding cryosectional morphology and
surgical findings (Westesson & Rohlin 1984b, Westesson et al. 1986,
Tanimoto et al. 1989, Schellhas et al. 1988). Perforation and adhesion
of the disc can also be shown by this technique (Helms et al. 1980,
Westesson et al. 1986, Schellhas et al. 1988, Ryan et al. 1990). These
studies have given important evidence for diagnosis and identification
of TMJ internal derangement (Wilkes 1989). Arthrography is based on
plain film (Helms et al. 1980, Van Sickels et al. 1983) and tomography
(Wilkes 1978a,b, Katzberg et al 1979). A recent study reported that
using the arthrography technique might improve the accuracy of
diagnosing perforations and adhesions of the disc in magnetic
resonance imaging of TMJ (Toyama et al. 2000).
There are some advantages of this technique. Arthrography is a
method that depends upon more technical training and experience in
the observation of images, it also has minimally invasive (Westesson et
al. 1993).

Ref.
http://herkules.oulu.fi/isbn9514266439/html/x126.html
Arthrography
Advantages: visualizes shape and position of soft tissues, may be
able to ascertain the condition of the articular disc, dynamic
movements of the disc and condyle may be visualized using a
fluoroscope, since TMJ is anesthetized and a needle is already in the
joint space- the clinician may easily perform a lavage procedure.
Disadvantages: expensive, invasive, expose the patient to relatively
high levels of radiation, special training is required, ballooning and
displacement of the disc occurs in every arthrogram.
Ref.
Okeson. (2003). Management of temporomandibular disorders and
occlusion, 5th Ed. Mosby, p. 296-8.
TMJ Anatomy

Adapted from: Moore and Agur. (2002) Essential clinical anatomy, 2nd
Ed. Lippencott Williams and Wilkins.

A common feature of the congenital melanocytic nevus is?


a. It loses its pigmentation with time
b. Has a predilection for the dorsal of the hands and feet, the scalp
and face
c. There is 5:1 male predilection
d. Affects the skin of the face eyes, and mucous membrane
innervated by V1, V2
e. The presence of hypertrichosis within the lesion which may
become prominent over time

A common feature of the congenital melanocytic nevus is?


a.
b.
c.
d.
e. The presence of hypertrichosis within the lesion which may
become prominent over time

A common feature of the congenital melanocytic nevus is the presence


of hypertrichosis (excess hair) within the lesion which may become
prominent over time (giant hairy nevus). The large congenital nevus
also sometimes is referred to as bathing trunk nevus or garment
nevus because it gives the appearance of the patient wearing an
article of clothing. 5 to 10 percent may undergo malignant
transformation.
Junctional nevus is that of a sharply demarcated, brown or black
macule, typically less than 6 mm in diameter. The lesion may
proliferate over a period of years to produce a slightly elevated, soft
papule with a relatively smooth surface, compound nevus. As time
passes, the nevus gradually loses its pigmentation, the surface may
become papillomatous, and hairs may be seen growing from the center
of the lesion, intradermal nevus.
The blue nevus is explained by the Tyndall-effect, which relates to the
interaction of light with particles in a colloidal suspension. The
common blue nevus has a predilection for the dorsal of the hands and
feet, the scalp and face. Oral lesions are commonly found on the
palate. 50% of the cellular blue nevus arise on the buttocks region.
Nevus of ota (ocular melanosis) is another type of blue nevus that is an
acquired blue-gray macule. It affects the skin of the face eyes, and
mucous membrane innervated by V1, V2. Its common in Japanese,
but rare in other races. There is 5:1 female predilection and is not to
confused with trauma.
Junctional Acquired Melanocytic Nevi
demarcated black macule
Compound Acquired Melanocytic Nevi
smooth surface, tan

Flat sharply
Slightly elevated,

Intradermal Acquired Melanocytic Nevi


papillary, may have hairs
Congenital Nevus
exhibit hair growth as

Loses pigment,
Bathing trunk, can
well as pigment

Common Blue Nevus


palate and hands

Location more on

Cellular Blue Nevus

Mongolian spot, buttocks,


must rule out trauma

Nevus of Ota

Skin of face, eyes, mucous


membranes, innervated by
V1, V2

Ref.
Neville and Damm Oral and Maxillofacial Pathology, 2nd ed., Copyright
2002 W.B. Saunders
Company ISBN 0-7216-9003-3

Which is the incorrect statement regarding the characteristics of


methamphetamine (MA) use:
a. Accelerated tooth decay in teenagers and young adults that is
not accounted for by other factors.
b. Distinctive pattern of decay on the lingual smooth surfaces of the
teeth and the interproximal surfaces of the posterior teeth.
c. Malnourished appearance (MA acts as an appetite suppressant
and increases activity levels)
d. Poor compliance with and poor response to preventive
treatments, combined with
unreliability in keeping
appointments.
e. Excessive tooth wear because of excessive grinding and
clenching.

Which is the incorrect statement regarding the characteristics of


methamphetamine (MA) use:
a.
b. Distinctive pattern of decay on the lingual smooth surfaces of the
teeth and the interproximal surfaces of the posterior teeth.
c.
d.
e.

Dental professionals should watch for the following


characteristics of MA use:
Accelerated tooth decay in teenagers and young adults that is not
accounted for by other factors
Distinctive pattern of decay on the buccal smooth surfaces of the
teeth and the interproximal surfaces of the anterior teeth
malnourished appearance (MA acts as an appetite suppressant and
increases activity levels)
poor compliance with and poor response to preventive treatments,
combined with unreliability
in keeping appointments
excessive tooth wear because of excessive grinding and clenching.
JCDA www.cda-adc.ca/jcda November 2005, Vol. 71, No. 10

Methamphetamine Use and Oral Health (Meth Mouth)


Dental health care professionals should be aware that
methamphetamine use is on the rise in the U.S. The allure of this drug
is that it is cheap, easy to make and the high lasts much longer than
crack cocaine (12 hours versus one hour for cocaine). As well as being
a potent central nervous system stimulant that can cause permanent
brain damage, methamphetamine use has also been associated with
severe oral health effects. Dental professionals should be aware of
methamphetamines oral health effects and the treatment
considerations for users of this drug.
Street names for Methamphetamine
Meth, Speed, Ice, Chalk, Crank, Fire, Glass, and Crystal

Drug description
Methamphetamine can be smoked, snorted, injected or taken orally.
Typically, it is a bitter tasting powder that readily dissolves in
beverages. Another common form of the drug is a clear, chunky
crystal. This is the form known as ice or crystal meth and it is
smoked in a manner similar to crack cocaine. Methamphetamine can
also be in the form of small, brightly colored tablets. The pills are often
called by their Thai name, yaba.
Mechanism of action
Methamphetamine stimulates release and blocks re-uptake of
neurotransmitters called monoamines (dopamine, norepinephrine and
serotonin) in the brain. Several areas of the brain are affected: the
nucleus accumbens, prefrontal cortex, and striatum.1
Cerebral effects
By altering the levels of neurotransmitters in the brain,
methamphetamine causes feelings of pleasure and euphoria.
Methamphetamine is a neurotoxin and potent stimulant, which can
also cause cerebral edema and hemorrhage, paranoia and
hallucinations. Short-term effects include insomnia, hyperactivity,
decreased appetite, increased respiration and tremors. Long term
effects can include psychological addiction, stroke, violent behavior,
anxiety, confusion, paranoia, auditory hallucination, mood
disturbances, and delusions1. Methamphetamine use can eventually
cause depletion of monoamines in the brain, which can have an effect
on learning. 2,3
Systemic effects
With high doses there may be an increase in both systolic and diastolic
blood pressure due to cardiac stimulation. In addition,
methamphetamine may produce arrythmias. Other systemic effects
include: shortness of breath, hyperthermia, nausea, vomiting and
diarrhea.
Oral effects
The oral effects of methamphetamine use
can be devastating. Reports have described
rampant caries that resembles early
childhood caries and is being referred to as
meth mouth.4,5,6 A distinct and often
severe pattern of decay can often be seen
View Larger
on the buccal smooth surface of the teeth
Photo
and the interproximal surfaces of the
anterior teeth.4

The rampant caries associated with


methamphetamine use is probably caused
by a combination of drug-induced
psychological and physiological changes
resulting in xerostomia (dry mouth),
extended periods of poor oral hygiene,
frequent consumption of high calorie,
carbonated beverages and tooth grinding
and clenching. Some reports have also
speculated that the acidic nature of the
drug is a contributing factor.4,5,6
Methamphetamine user profile
Traditionally, methamphetamine use has been most pronounced
among males between the ages of 19 and 40.7 According to the 2003
National Survey on Drug Use and Health8, 12.3 million Americans age
12 and older had tried methamphetamine at least once in their
lifetimes (5.2 percent of the population), with the majority of past-year
users between 18 and 34 years of age. Research funded by the
National Institute on Drug Abuse (NIDA) found 2.8 percent of young
adults (ages 1826) reported the use of crystal methamphetamine in
the past year during 20012002.9 These users were disproportionately
white and male and live in the West. The study found Native Americans
were 4.2 times more likely than whites to use the drug.
What the dentist should be on the
lookout for:

Unaccounted for and


accelerated decay in teenagers
and young adults.

Distinctive pattern of decay on


the buccal smooth surface of
the teeth and the interproximal
surfaces of the anterior teeth. View Larger Photo

Malnourished appearance in
heavy users, because
methamphetamine acts as an
appetite suppressant."

What dental health-care providers can do when they suspect


methamphetamine use:

Complete a comprehensive oral examination that includes taking


a thorough dental and medical history.

Express concern regarding the dental findings.10

If the patient is receptive to a medical consult, have the phone


number of a local physician, clinic or substance abuse
rehabilitation facility available and be familiar with their protocol,
so that the patient can be told what to expect.10

Use preventive measures such as topical fluorides.

Encourage consumption of water rather than sugar-containing


carbonated beverages.

Be cautious when administering local anesthetics, sedatives or


general anesthesia, nitrous oxide, or prescribing narcotics.

Take opportunities to educate your patients about the risks


associated with methamphetamine or any illicit drug use.11,12

The following instructions to dental professionals who suspect patients


may be using
MA were recently developed by the American Dental Association
Complete a comprehensive oral examination, including a
thorough dental and medical history.
Attempt to educate the patient about the profound negative
effects that MA can have on oral health.
Refer the patient to such resources as physicians or drugcounselling services.
Use preventive measures such as topical fluorides,
remineralization products and chlorhexidine applications.
Encourage the patient to drink water or artificially sweetened
drinks instead of sugar-containing beverages.
Be cautious when administering local anesthetics, sedatives,
general anesthesia or nitrous oxide and when prescribing
narcotics because of potential druginteraction.
Take opportunities to educate patients about the risks associated
with the use of MA and other illicit drugs.

Ref.
Academy of General Dentistry. This is your mouth on meth. Any
Questions? AGD Impact. November 2005.
American Dental Association. Methamphetamine use. August 2005.

http://www.ada.org/prof/resources/topics/methmouth.asp.
Specter, Michael. "Higher Risk." The New Yorker. May 23, 2005.
National Institute on Drug Abuse. National Institutes of Health. NIDA
InfoFacts: Methamphetamine.
http://www.drugabuse.gov/infofacts/methamphetamine.html
Substance Abuse and Mental Health Services Administration. Drug
Abuse Warning Network. Results from the National Survey on Drug Use
and Heath. 2005. September 2006.
http://www.oas.samhsa.gov/NSDUH/2k5NSDUH/2k5Results.htm
Office of National Drug Control Policy. Methamphetamine Facts and
Figures.
http://www.whitehousedrugpolicy.gov/drugfact/methamphetamine/met
hamphetamine_ff.html

Which of the following adverse reaction is not caused by epinephrine?


a.
b.
c.
d.
e.

Tremor
Miosis
Anxiety
Headache
Palpitations

Which of the following adverse reaction is not caused by epinephrine?


a.
b. Miosis
c.
d.
e.

Miosis is a closing of the pupil; epinephrine causes mydriasis, or pupil


dilation.
Overdosage/Toxicology:
Based on lidocaine component: Lidocaine has a narrow therapeutic
index. Severe toxicity may occur at doses slightly above the
therapeutic range, especially in conjunction with other antiarrhythmic
drugs. Symptoms of overdose include sedation, confusion, coma,
seizures, respiratory arrest, and cardiac toxicity (sinus arrest, AV block,
asystole, and hypotension). QRS and QT intervals are usually normal,
although they may be prolonged after massive overdose. Other effects
include dizziness, paresthesia, tremor, ataxia, and GI disturbance.
Treatment is supportive.
Based on epinephrine component: Symptoms of overdose include
hypertension, which may result in subarachnoid hemorrhage and
hemiplegia; arrhythmias; unusually large pupils; pulmonary edema;
renal failure; and metabolic acidosis. There is no specific antidote for
epinephrine intoxication and treatment is primarily supportive.

Adverse Reactions:
Degree of adverse effects in the central nervous system and
cardiovascular system are directly related to the blood levels of
lidocaine. The effects below are more likely to occur after systemic
administration rather than infiltration.
Cardiovascular: Myocardial effects include a decrease in contraction
force as well as a decrease in electrical excitability and myocardial
conduction rate resulting in bradycardia and reduction in cardiac
output.

Central nervous system: High blood levels result in anxiety,


restlessness, disorientation, confusion, dizziness, tremor, and seizure.
This is followed by depression of CNS resulting in somnolence,
unconsciousness and possible respiratory arrest. In some cases,
symptoms of CNS stimulation may be absent and the primary CNS
effects are somnolence and unconsciousness.
Gastrointestinal: Nausea and vomiting may occur
Hypersensitivity reactions: Extremely rare, but may be manifest as
dermatologic reactions and edema at injection site. Asthmatic
syndromes have occurred. Patients may exhibit hypersensitivity to
bisulfites contained in local anesthetic solution to prevent oxidation of
epinephrine. In general, patients reacting to bisulfites have a history of
asthma and their airways are hyper-reactive to asthmatic syndrome.
Psychogenic reactions: It is common to misinterpret psychogenic
responses to local anesthetic injection as an allergic reaction. Intraoral
injections are perceived by many patients as a stressful procedure in
dentistry. Common symptoms to this stress are diaphoresis,
palpitation, hyperventilation, generalized pallor and a fainting feeling.
Drug Interactions:
Lidocaine: Substrate of CYP1A2 (minor), 2A6 (minor), 2B6 (minor),
2C8/9 (minor), 2D6 (major), 3A4 (major); Inhibits CYP1A2 (strong),
2D6 (strong), 3A4 (moderate)
Also see individual agents. Note: Significance of interaction may
depend on route of drug delivery and systemic exposure.
Beta-blockers, nonselective: Combination treatment may increase
blood pressure.
Epinephrine (and other direct alpha-agonists): Pressor response to I.V.
epinephrine, norepinephrine, and phenylephrine may be enhanced in
patients receiving TCAs (Note: Effect is unlikely with epinephrine or
levonordefrin dosages typically administered as infiltration in
combination with local anesthetics)
General Anesthetics: May increase sensitivity of myocardium to
dysrhythmic effects of epinephrine.

Tricyclic Antidepressants: Combination treatment may increase blood


pressure.
Mechanism of Action:
Lidocaine blocks both the initiation and conduction of nerve impulses
via decreased permeability of sodium ions; epinephrine increases the
duration of action of lidocaine by causing vasoconstriction (via alpha
effects) which slows the vascular absorption of lidocaine
Pharmacodynamics/Kinetics:
Onset of action: Peak effect: ~5 minutes
Duration: ~2 hours; dose and anesthetic procedure dependent
Absorption: Topical: Lidocaine: Minimal; Epinephrine: Minimal
See individual agents.
Dosage:
Dosage varies with the anesthetic procedure, degree of anesthesia
needed, vascularity of tissue, duration of anesthesia required, and
physical condition of patient.
Dental anesthesia, infiltration, or conduction block:
Children <10 years: 20-30 mg (1-1.5 mL) of lidocaine hydrochloride as
a 2% solution with epinephrine 1:100,000; maximum: 4-5 mg of
lidocaine hydrochloride/kg of body weight or 100-150 mg as a single
dose
Children >10 years and Adults: Do not exceed 6.6 mg/kg body weight
or 300 mg of lidocaine hydrochloride and 3 mcg (0.003 mg) of
epinephrine/kg of body weight or 0.2 mg epinephrine per dental
appointment. The effective anesthetic dose varies with procedure,
intensity of anesthesia needed, duration of anesthesia required, and
physical condition of the patient. Always use the lowest effective dose
along with careful aspiration.
The following numbers of dental carpules (1.8 mL) provide the
indicated amounts of lidocaine hydrochloride 2% and epinephrine
1:100,000 (see table):
Administration:
# of Cartridges

Lidocaine HCl

Epinephrine

(1.8 mL)

(2%)
(mg)

1:100,000
(mg)

(in a 1.8ml carpule)


1

36

0.018

72

0.036

108

0.054

144

0.072

180

0.090

216

0.108

252

0.126

288

0.144

324

0.162

10

360

0.180

For most routine dental procedures, lidocaine hydrochloride 2% with


epinephrine 1:100,000 is preferred. When a more pronounced
hemostasis is required, a 1:50,000 epinephrine concentration should
be used. The following numbers of dental carpules (1.8 mL) provide the
indicated amounts of lidocaine hydrochloride 2% and epinephrine
1:50,000 (see table):
Administration:
# of Cartridges
(1.8 mL)

Lidocaine HCl
(2%)
(mg)

Epinephrine
1:50,000
(mg)

36

0.036

72

0.072

108

0.108

144

0.144

180

0.180

216

0.216

Dermatologic procedure: Children 5 and Adults: Topical: Place 1


transdermal patch over area requiring analgesia; attach patch to
iontophoretic controller and leave on for 10 minutes. Remove patch
and perform procedure within 10-20 minutes of patch removal. Do not
use another patch for 30 minutes.
Cardiovascular Considerations:
Sympathomimetic or sympathomimetic-containing combination
products may increase blood pressure. These preparations are
relatively contraindicated in patients with significant hypertension,
particularly in poorly controlled hypertension. In young, healthy
patients presenting with new onset blood pressure elevations, it is
important to exclude the recent use of sympathomimetics as a cause
for the blood pressure elevation.
Dental Health: Effects on Dental Treatment:
It is common to misinterpret psychogenic responses to local anesthetic
injection as an allergic reaction. Intraoral injections are perceived by
many patients as a stressful procedure in dentistry. Common
symptoms to this stress are diaphoresis, palpitations, hyperventilation.
Patients may exhibit hypersensitivity to bisulfites contained in local
anesthetic solution to prevent oxidation of epinephrine. In general,
patients reacting to bisulfites have a history of asthma and their
airways are hyper-reactive to asthmatic syndrome.
Degree of adverse effects in the CNS and cardiovascular system is
directly related to the blood levels of bupivacaine. Bradycardia,
hypersensitivity reactions (rare; may be manifest as dermatologic
reactions and edema at injection site), asthmatic syndromes
High blood levels: Anxiety, restlessness, disorientation, confusion,
dizziness, tremors, seizures, CNS depression (resulting in somnolence,
unconsciousness and possible respiratory arrest), nausea, and
vomiting.

Dosage Forms:
Injection, solution, as hydrochloride, with epinephrine 1:50,000
(Xylocaine with Epinephrine): Lidocaine 2% [20 mg/mL] (1.8 mL)
[contains sodium metabisulfite]
Injection, solution, as hydrochloride, with epinephrine 1:100,000:
Lidocaine 1% [10 mg/mL] (20 mL, 30 mL, 50 mL); Lidocaine 2% (20
mL, 30 mL, 50 mL)
Xylocaine with Epinephrine: Lidocaine 1% [10 mg/mL] (10 mL 20 mL,
50 mL); Lidocaine 2% (1.8 mL, 10 mL, 20 mL, 50 mL) [contains sodium
metabisulfite]
Injection, solution, as hydrochloride, with epinephrine 1:200,000:
Lidocaine 0.5% [5 mg/mL] (50 mL)
Xylocaine with Epinephrine: Lidocaine 0.5% [5 mg/mL] (50 mL)
[contains sodium metabisulfite]
Injection, solution, as hydrochloride, with epinephrine 1:200,000
[methylparaben free]: Lidocaine 1% [10 mg/mL] (30 mL); Lidocaine
1.5% (5 mL, 30 mL); Lidocaine 2% (20 mL) [contains sodium
metabisulfite]
Xylocaine MPF with Epinephrine: Lidocaine 1% [10 mg/mL] (5 mL, 10
mL, 30 mL); 1.5% [15 mg/mL] (5 mL, 10 mL, 30 mL); Lidocaine 2% [20
mg/mL] (5 mL, 10 mL, 20 mL) [contains sodium metabisulfite]
Transdermal system (LidoSite): Lidocaine 10%, epinephrine 0.1%
(25s) [contains sodium metabisulfite; for use only with LidoSite
controller]

Ref.
Jastak JT and Yagiela JA, "Vasoconstrictors and Local Anesthesia: A
Review and Rationale for Use,"J Am Dent Assoc, 1983, 107(4):623-30.
MacKenzie TA and Young ER, "Local Anesthetic Update,"Anesth Prog,
1993, 40(2):29-34.
Wynn RL, "Epinephrine Interactions With Beta-Blockers,"Gen Dent,
1994, 42(1):16, 18.

Wynn RL, "Recent Research on Mechanisms of Local Anesthetics,"Gen


Dent, 1995, 43(4):316-8.
Yagiela JA, "Local Anesthetics,"Anesth Prog, 1991, 38(4-5):128-41.

Choose the correct statement regarding patients with New York Heart
Association (NYHA) class II through IV CHF.
a. These patients have no problem sleeping in supine position
everyday.
b. NYHA II have no limitations of physical activity.
c. may not tolerate a supine chair position because of pulmonary
edema and will need a
semisupine or upright chair position.
d. NYHA IV have marked limitations of activity but are comfortable
at rest

Choose the correct statement regarding patients with New York Heart
Association (NYHA) class II through IV CHF.
a.
b.
c. may not tolerate a supine chair position because of pulmonary
edema and will need a
semisupine or upright chair position.
d.

Patients with CHF other than NYHA class I can have blood backing up
into lung when their bodies placed in supine position since their heart
cannot pump blood efficiently.
NYHA I: No limitation of physical activity. No dyspnea, fatigue, or
palpitations with ordinary physical activity.
NYHA II: Slight limitation of physical activity. These patients have
fatigue, palpitations, and dyspnea with ordinary physical activity but
are comfortable at rest.
NYHA III: Marked limitation of activity. Less than ordinary physical
activity results in symptoms, but patients are comfortable at rest.
NYHA IV: Symptoms are present at rest, and any physical exertion
exacerbates symptoms.
Congestive heart failure (CHF) is the inability of the heart to deliver an
adequate supply of oxygenated blood to meet the bodys metabolic
demands. CHF affects between two and three million persons in the
United States, with about 500,000 new cases diagnosed each year. It is
very common in the elderly, representing the most frequent hospital
discharge diagnosis in patients over age 65. CHF can involve failure of
the left and right ventricle. Most of the acquired disorders that lead to
CHF will result in failure of the left ventricle, with right ventricle failure
following. In left-sided heart failure, blood backs up from the heart into
the lungs. The result is dyspnea (shortness of breath on exertion).
orthopnea (shortness of breath when supine), and paroxysmal
nocturnal dyspnea (dyspnea awakening the patient from sleep). In
right-sided heart failure blood backs up from the heart into the veins
and organs supplying blood to the heart. The result is peripheral
edema, pedal edema, pitting edema, swelling and congestion of the
liver, and ascites (fluid buildup in the abdominal cavity).

Ref.
JW Little, et al., Dental Management of the Medically Compromised
Patient, 6th ed. 2002, Mosby.

Which of the following oral squamous cell carcinoma locations is more


prone to early metastasis?
a. Lip vermillion
b. Tongue
c. Oropharynx
d. Gingiva

Which of the following oral squamous cell carcinoma locations is more


prone to early metastasis?
a.
b.
c. Oropharynx
d.
A soft palate lesion may present as a localized tumor, but 80% of
posterior oropharyngeal wall lesions have metasized or extensively
involved surrounding structures by the time of diagnosis.
It has been now established that the path that brings people to oral
cancer contains at least two distinct etiologies; one through tobacco
and alcohol and the other via HPV (Type 16). The anatomical
malignancy sites associated with each etiology appears to also be
different. HPV related oral cancer appear to occur on the tonsillar area,
base of the tongue and the orophayrnx non-HPV tumors tend to
involve the anterior tongue, floor of the mouth, the mucosa that
covers the cheeks and alveolar ridges. HPV positive tumors occur most
frequently in younger individuals than tobacco related malignancies.
The also occur more in white males, and in non-smokers. The HPV
group is the fastest growing segment of the oral cancer population.
Ref.
Neville BW, Damm DD, Allen CM, Bouquot JE, Oral and Maxillofacial
Pathology, Second Edition 2002, page: 356-363.
http://www.oralcancerfoundation.org/hpv/

All of the following lesions are radioopaque EXCEPT:


a.
b.
c.
d.
e.

Amalgam tattoo
Odontoma
Osteomyelitis
Condensing osteitis
Osteopetrosis

All of the following lesions are radioopaque EXCEPT:


a.
b.
c. Osteomyelitis
d.
e.
Amalgam tattoos result from the deposition of metal particles in the
oral tissues. The fragments are densely radioopaque, varying from
several millimeters to pinpoint in size on radiographs.
Odontomas are the most common odontogenic lesion. They are
developmental anomalies containing products of odontogenesis.
Radiographically, odontomas may appear as a collection of toothlike
structures (compound odontoma), or as a calcified mass with the
radiodensity of a tooth structure (complex odontoma).
Osteomyelitis is an inflammatory process in the medullary or cortical
portion of bone. In acute osteomyelitis, radiographs may be
unremarkable or demonstrate an ill-defined radiolucency. In chronic
osteomyelitis, radiographs reveal a patchy, ragged, and ill-defined
radiolucency that often contain central radiopaque sequestra.
Condensing osteitis is inflammation of endodontic origin that presents
as a radioopaque lesion in a periapical location. The lesion does not
exhibit a radiolucent border and classically consists of a localized,
usually uniformed zone of increased radiodensity adjacent to the apex
of a tooth that exhibits a thickened periodontal ligament space or an
apical inflammatory lesion.
Osteopetrosis is a hereditary skeletal disorder characterized by a
marked increase in bone density. Radiographically, there is a
widespread increase in skeletal density with; the roots of the teeth
often are difficult to visualize because of the density of the surrounding
bone.
Ref.
Neville, Damn, Allen, Bouquot. Oral & Maxillofacial Pathology. 2nd
edition.

All of the following are radiographic appearances are suggestive of


malignant lesions EXCEPT:
a. Ill-defined borders
b. Irregular widening and destruction of the lamina dura
c. Sun burst appearance
d. Displaced roots
e. Root resorption

All of the following are radiographic appearances are suggestive of


malignant lesions EXCEPT:
a.
b.
c.
d. Displaced roots
e.
Some of the common radiographic appearances of malignant lesions
are: sunburst patterns (osteosarcoma), lucencies with poorly defined
or moth-eaten borders, root resorption, irregular or asymmetrical
widening of the PDL, and onion skinning (Ewings sarcoma). Benign
lesions generally demonstrate well-defined or corticated borders,
displacement of roots, and expansion of bone without destruction of
the cortical plate.
Ref. Neville, Damn, Allen, Bouquot. Oral & Maxillofacial Pathology. 2nd
edition.

Osteosarcoma in a 41-year-old man. (a) CT scan reveals osteoblastic


changes (arrows) within the right mandibular body. Note the abnormal
soft-tissue ossification (arrowhead). (b) Contrast-enhanced T1weighted MR image demonstrates an ill-defined lesion (arrow) arising
from periosteum. Note the decreased marrow signal intensity
(arrowhead) involving the entire right mandibular body.
Ref.
http://radiographics.rsna.org/content/26/6/1751.full

Benign and Malignant Lesions of the Mandible


Cystic lesions

Periapical (radicular) Cyst


Follicular (dentigerous) cyst
Odontogenic keratocyst (OKC)
Primordial Cyst
Residual cyst
Solitary bone cyst (traumatic, simple, hemorrhagic bone cyst)
Medullary pseidocyst
Aneurysmal bone cyst
Static bone cavity (Stafne cyst {not a true cyst})
Solid lesions
Benign
Odontogenic
Odontoma
Ameloblastoma
Odontogenic myxoma
Calcifying epithelial odontogenic tumor (Pindborg
tumor)
Cementoblastoma
Ameloblastic fibroma
Adenomatoid odontogenic tumor
Nonodontogenic
Osseous Lesions
Ossifying fibroma
Juvenile ossifying fibroma
Periapical cemental dysplasia
Florid cemento-osseous dysplasia
Exostosis (torus mandibularis)
Osteoma
Fibrous dysplasia
Paget disease
Nonosseous lesions
Neurofibroma
Schwannoma
Central giant cell granuloma (giant cell
reparative cyst)
Arteriovenous malformation (AVM)
Central hemangioma
Eppilus fissuratum
Malignant
Odontogenic
Odontogenic carcinoma (ameloblastic carcinoma)
Odontogenic sarcoma
Odontogenic carcinosarcoma

Nonodontogenic
Osteosarcoma
Chondrosarcoma
Metastasis
Fibrosarcoma
Leiomyosarcoma
Lymphoma
Leukemia
Multiple myeloma and plasmacytoma
Squamous cell carcinoma
Metabolic bone lesions
Osteoporosis
Osteomalacia
Renal osteodystrophy
Ostetis fibrosa cystica
Ref.
http://radiographics.rsna.org/content/26/6/1751/T2.expansion.html

With regards to pemphigoid and pemphigus, choose the correct


statement?
a. Pemphigoid alters the cellular connections at the basement
membrane (desmosomes)answer
b. Positive Nikolsky sign is seen only with pemphigus answer
c. Pemphus rarely affects the oral cavity
d. Pemphigus alters the cellular connections above the basement
membrane (hemidesmosomes)answer
e. Phemphigoid rarely affects the oral cavity

With regards to pemphigoid and pemphigus, choose the correct


statement?
a.
b.
c.
d.
e. Phemphigoid rarely affects the oral cavity

Phempighus and phempigoid are both are autoimmune disease.


Pemphigoid alters the cellular connections(hemidesmosomes) above
the basement membrane, while pemphigus alters the cellular
connections(desmosomes) at the basement membrane, causing a burn
like blister lesion. Positive Nikolsky sign can be seen in either disease.
Pemphigoid is more common, less dangerous affecting older people
(over 60), produces blisters that appear mainly on the trunk, arms and
legs, rarely in the mouth. Lesion can
be large (3cm), itchy and painful.
Pemphigus can be familial, its precise cause is unknown, possible
triggers include penicillamine derived drugs, HSV, solar irradiation and
stress. The blisters are fragile and burst easily leaving ulcers that do
not heal without treatment. The lesions are itchy, painful, and burning,
and treatment resembles that of burn victims. Three main types are
vulgaris-usually appears in mouth first, foliaceus-does not occur in
mouth, paraneoplastic- occurs in people with cancer.
Treatment of both involve oral steroids in order to suppress the
immune system.

Ref.
Neville, Damn, Allen, Bouquot. Oral & Maxillofacial Pathology. 2nd
edition.

Which statement is true concerning Necrotizing Sialometaplasia?


a.
b.
c.
d.

Radiation therapy used to treat


Locally destructive inflammatory lesion of salivary glands
75% occur on the midline of the mid-palatal region
Females affected 2X more than males

Which statement is true concerning Necrotizing Sialometaplasia?


a.
b. Locally destructive inflammatory lesion of salivary glands
c.
d.

Necrotizing Sialometaplasia is a locally destructive inflammatory


condition of the salivary glands. It is the result of ischemia of the
salivary tissue that leads to local infarction.
75% occur on the posterior palate. The hard palate affected more
so than the soft palate and more laterally located.
Starts as a non-ulcerated swelling. After 3 weeks, necrotic tissue
sloughs out leaving a crater-like ulceration.
Biopsy indicated to R/O malignant disease. Usually self healing in
5-6 weeks.
Men are 2X more likely to develop than women.

Ref.
Neville,BW, Damm, DD, et al, Oral and Maxillofacial Pathology;
Saunders Printing, Philadelphia, PA; 2002; pp. 405-406.

Which of the following is a patient with uncontrolled hyperthyroidism


not sensitive to?
a.
b.
c.
d.

Epinephrine in retraction cord


Pressor amines in local anesthetic
Vasoconstrictor in local anesthetic
Iodine

Which of the following is a patient with uncontrolled hyperthyroidism


not sensitive to?
a.
b.
c.
d. Iodine
Thyroid hormone functions as a stimulus to metabolism and is critical
to normal function of the cell. In excess, it both overstimulates
metabolism and exacerbates the effect of the sympathetic nervous
system, causing "speeding up" of various body systems and symptoms
resembling an overdose of epinephrine (adrenaline). These include fast
heart beat and symptoms of palpitations, nervous system tremor and
anxiety symptoms, digestive system hypermotility (diarrhea), and
weight loss.
Patients with uncontrolled hyperthyroidism require special dental
management. They are sensitive to epinephrine and pressor amines in
local anesthetics and gingival retraction cords. These agents should
not be administered until hyperthyroidism is controlled.
Patients with undiagnosed or poorly controlled thyrotoxicosis may
develop thyrotoxic crisis, a serious complication with an abrupt onset.
Most patients who develop thyrotoxic crisis have a goiter, wide pulse
pressure, eye signs, and a long history of thyrotoxicosis. Precipitating
factors are infections, trauma, surgical emergencies, and surgery.
Symptoms are extreme restlessness,
nausea, vomiting, and abdominal pain. Coma and severe hypotension
may develop. Immediate treatment consists of anti-hyperthyroid
therapies to include propylthiouracil, potassium iodide, propranolol,
hydrocortisone, and ice packs or cooling blankets; CPR may be needed
until medical help arrives.
Long-term follow-up is necessary for patients with Graves disease and
should include an annual physical examination and measurement of
serum concentrations of thyrotropin and free thyroxine. These should
be maintained in the normal range. After the thyrotoxic patient is well
controlled, the dental treatment plan will be unaffected. If acute
infections develop, the patients physician should be consulted.
The use of local anesthetic agent containing a vasoconstrictor in
patients with high concentrations of T4 and T3 is an area of concern.
Thyroid hormones appear to act synergistically with epinephrine by
increasing tissue sensitivity to catecholamines and possibly upregulating adrenergic receptors. An additional problem associated with
the use of local anesthetic agents containing epinephrine is related to
the treatment of hyperthyroid symptoms with a nonselective _-

adrenergic antagonist. However, these concerns must be balanced


against the value of a vasoconstrictor in inducing profound local
anesthesia, which is essential in reducing the physiologic stress
associated with pain. For the patient with overt evidence of
hyperthyroidism, the use of vasoconstrictors should be avoided. For all
other scenarios, the cautious use of vasoconstrictors, based on the
patients functional capacity, should be considered. For those patients
whose functional capacity is equal to or greater than 4 METs, 4.5 mL of
a local anesthetic agent with epinephrine 1:100,000 (or equivalent)
can be administered safely. Furthermore, combination analgesics
containing acetylsalicylic acid (ASA) are contraindicated in patients
with hyperthyroidism because ASA interferes with the protein binding
of T4 and T3, thereby increasing their free form, and can lead to
thyrotoxicosis.
Table: Dental management of the patient with thyroid dysfunction
Euthyroid patient OR patient with mild to moderate thyroid dysfunction
AND/OR minor clinical predictors (advanced age, atrial fibrillation,
history of stroke) OR intermediate clinical predictors (stable angina
pectoris, previous myocardial infarction [MI], compensated heart failure
renal insufficiency) of cardiovascular risk
Blood pressure less than 180/110 mm Hg; normal pulse pressure, rate
and rhythm; functional capacity greater than 4 METs
Comprehensive dental care
Routine referral for medical management and risk factor modification
Blood pressure less than 180/110 mm Hg; normal pulse pressure, rate
and rhythm; BUT functional capacity less than 4 METs
Appropriate limited dental care*
Routine referral for medical management and risk factor modification
Blood pressure greater than 180/110 mm Hg OR systolic blood
pressure less than 90 mm Hg AND/OR abnormal pulse pressure, rate,
or rhythm
Appropriate emergency dental care**
If patient is symptomatic, immediate referral for medical
management and risk factor modification
If patient is asymptomatic, routine referral for medical management
and risk factor modification
Patient with severe hypothyroidism OR thyrotoxicosis AND/OR major
clinical predictors (unstable coronary syndrome, decompensated heart
failure, severe valvular disease, significant arrhythmias) of
cardiovascular risk
Appropriate emergency dental care**
Immediate referral for medical management and risk factor
modification
* Limited office care may include dental prophylaxis, restorative
procedures, simple periodontal and endodontic procedures, and routine
extractions.

** Emergency office care under local anesthesia without a


vasoconstrictor should be based on firm evidence that the benefits
achieved by therapeutic intervention outweigh the risk of
complications associated with the patient's diabetic or cardiovascular
status and may include activities related to pain relief, the treatment of
infection (including simple incision and drainage), and the induction of
hemostasis (avoid the use of epinephrine to control local bleeding).
PDF Full Text (393KB)
Ref.
Huber, Michaell A.; Terzhalmy, Gza T.. Quintessence International,
2008, Vol. 39 Issue 2, p139-150
Silverman S, Eversole L, Truelove E: Essentials of Oral Medicine. BC
Decker Ed. 2001
Yagiela JA, Adverse Drug Interactions in Dental Practice: Interactions
Associated with Vasoconstrictors, J Am Dent Assoc. Vol. 130, May 1999,
pages 701-9

Which of the following is false?


a. Salivary function decreases with age
b. Multiple medication interactions have potential to cause
xerostomia
c. Sjogrens syndrome is considered an autoimmune disorder
d. Ectodermal Dysplasia causes defects in salivary function

Which of the following is false?


a. Salivary function decreases with age
b.
c.
d.
Salivary function does not decrease with age- typically one of the
following situations leads to Xerostomia.
Xerostomia (acute or chronic) may result from: medication side effect,
mechanical blockage, dehydration, emotional stress, infection of the
salivary glands, surgery, avitiminosis, anemia, CT diseases, and
Sjogrens syndrome, and congenital factors (e.g. ectodermal
dysplasia).
Clinical appearance: dry, red, atrophic tissue/mucosa. Tongue may lose
papilla definition, be burning, be fissured, or inflamed. Higher incidence
of root caries.
Treatment of these patients serves to keep mouth moist, aid in
mastication, prevent caries, buffer acid, protect tissue, prevent candida
overgrowth.
Saliva Substitutes contain: Sodium Carboxymethyl Cellulose
Saliva Stimulants: Pilocarpine
Caries prevention: Fluoride, Amorphous Calcium Phosphate
Ref. Little, et al. Dental Management of the Medically
Compromised Patient. 6th ed. (2002) p 554-555.

Edgar, W., OMullane, D. Saliva and oral health. 1996. British Dental
Journal. p30.

Acetaminophen is which of the following?


a.
b.
c.
d.
e.

Pregnancy
Pregnancy
Pregnancy
Pregnancy
Pregnancy

Category
Category
Category
Category
Category

A
B
C
D
X

Acetaminophen is which of the following?


a.
b. Pregnancy Category B
c.
d.
e.
FDA Category B:
Lidocaine/ Prilocaine
Acetaminophen
Meperidine, Morphine
Oxycocone
NSAIDs : Category B/D Dont use in 3rd Trimester
Short-term use of NSAIDs in late pregnancy is associated with a
significant increase in the risk of premature ductal
closure
Aspirin: Category C/D
Codeine: Category C
Reference: Kore, Gideon., et al. Nonsteroidal Antiinflammatory
Drugs During Third Trimester and the Risk of
Premature Closure of the Ductus Arteriosus: A
Meta-Analysis. The Annals of Pharmacotherapy: Vol.
40, No. 5, pp. 824-829.

Ref.
http://depts.washington.edu/druginfo/Formulary/Pregnancy.pdf

A patient with an Anteriorly Displaced Disc without reduction will


complain of which of the following?
a.
b.
c.
d.

Occasional popping and clicking without pain


Occasional popping and clicking with pain
Occasional opening and the jaw will lock in place
Severe pain during meals

A patient with an Anteriorly Displaced Disc without reduction will


complain of which of the following?
a.
b.
c. Occasional opening and the jaw will lock in place
d.

TMJ sounds are detected in 25-35% of the general population. Not all
joint sounds are progressive. Only joint sounds associated with pain
should be considered for treatment, if the pain is intracapsular in origin
(as opposed to extracapsular muscle pain).
In an anteriorly displaced disc, the condyle articulates on the posterior
border of the disc. This may progress to anterior displacement of the
disc. Disc displacement occurs due to ligament elongation (capsular
and discal ligaments), also the disc can be thinned. These changes
occur in response to trauma: macro (pt usually knows about) or micro
(pt unaware (i.e. bruxism or hypoxia-reperfusion injury). Changes start
on cellular level and work up to clinical changes: (e.g. long term
clenching leads to overuse and hypoxia-reperfusion injury eventually
collagen fibrils fragment and loss of disc stiffness, loss of disc shape
Clinically:
ADD with reduction: relatively normal ROM, but may be limited by pain.
Discal movement palpable upon opening/closing. Deviation common.
ADD without reduction: Pts often report the exact onset. Sudden
changes of mandibular movement. Gradual increase of catching and
clicking is reported. Often no joint sounds present after the disc
dislocation.
Treatment:
ADD with reduction: re-establish normal chondyle-disc relationship.
Methods: Anterior positioning device, occlusal adjustment and/or ortho
(very, very cautious use), Physical Self regulation (down-regulation of
the CNS), Biofeedback for exasperating habits, decreased joint loading
(e.g. soft diet, small bites, slow chewing), NSAIDs, passive stretching.
ADD without reduction: NO anterior positioning appliance (pulling the
disc forward in these pts only exasperates symptoms!) Manual
Manipulation (easier to unlock pts who havent been locked for
long- becomes more difficult the longer the condition persists). 1. The
lateral pterygoid muscle has to be relaxed to reduce these patients. 2.
The disc space must be increased before the reduction can take place.
(tell pt to relax and try not to close). 3. Chondyle must be in the
forward most position.
Stabilization appliances can be fabricated to help prevent these

patients from locking. Pt education is key for these pts: many just try
to open harder and wear out their muscles leading to increased pain
and frustration. Decrease hard biting, no gum chewing or other
exasperating habits. NSAIDs prn. Surgical considerations may be
made: Arthrocentesis, arthroscopy, arthrotomy, disc repair (plication),
discectomy, implants. (all surgeries have large risk including potential
trauma to Facial nerve) Surgery is probably only indicated for <5% of
all TMD pts.

Okeson. (2003). Management of temporomandibular disorders


and occlusion, 5th Ed. Mosby, p. 438-53.

http://www.adameducation.com/aiaonline_student.aspx

Which of the following has NOT been linked to Epstein Barr Virus?
a.
b.
c.
d.
e.

Mononucleosis
Burkitts lymphoma
Oral Hairy leukoplakia
Molluscum Contagiosum
Nasopharyngeal carcinoma

Which of the following has NOT been linked to Epstein Barr Virus?
e.
a.
b.
c. Molluscum Contagiosum
d.
Primary EBV infection: Monoucleosis.
Tumors:
Burkitts lymphoma
Oral Hairy Leukoplakia
Nasopharyngeal carcinoma
Infectious Monoucleosis: EBV is transmitted via intimate contact. Kids:
shared toys/cups with saliva contamination. Adults: shared saliva.
Infection is life long. Kissing Disease. 90% have involvement of
anterior & posterios cervical lymph node chains.
Burkitts lymphoma (BL): High grade B-cell Lymphoma. Commonly seen
in equatorial regions. Endemic BL @7yrs old. Sporadic BL @11yrs old.
Fastest growing human neoplasm. One of most common lymphomas in
AIDS.
Oral Hairy Leukoplakia: EBV induced hyperkeratotic lesion in areas of
trauma (oral cavity) In AIDS pts 80% of the time. Seen in some other
pts with immunocompromise. Coincides with CD4+ depression.
Clinically: Flat, white plaques on lateral or dorsal potion of tongue.
fenceposts along ths lateral border of tongue. Asymptomatic. Benign
tumor. Need to biopsy- tx of immunosuppression typically addresses
lesion.
Nasopharyngeal carcinoma: Diet relation (salt-cured fish), EBV, genetic
component, environment: Asia, North Africa. Male/female 3:1. 40s-50s.
Clinical: Epistaxis, nasal obstruction, lymphadenopathy, ear infections
(otitis media). High incidence of distant metastasis. 2 types:
Keratinizing, and Non-keratinizing- treated with surgery and radiation
(respectively)
Ref:
Neville, Damn, Allen, Bouquot. Oral & Maxillofacial Pathology. 2nd
edition.
Lau HY, Twu NF, Chen PC, Lai CR, Juang CM, Yen MS, Chao KC. The
relationship between human papillomavirus and Epstein-Barr
virus infections in relation to age of patients with cervical
Adenocarcinoma. Taiwan J Obstet Gynecol. 2009 Dec;48(4):370-4.
Results do not support a role for EBV in cervical adenocarcinogenesis
or any relationship between EBV and HPV infection in Adenocarcinoma.

Which of the following characteristics is can NOT be evaluated


clinically in assessment of a melanoma?
a.
b.
c.
d.
e.

Symmetry
Size
Color
Duration
Borders

Which of the following characteristics should be evaluated clinically


in assessment of a melanoma?
a.
b.
c.
d. Duration
e.
Melanoma is a malignant neoplasm usually arising on the skin,
but may occur in the mouth, usually affecting white adults, and is
a slowly expanding pigmented area, usually due to excessive sun
exposure.
Oral melanotic macules occur in both sexes at any age, usually
on the lower lip, and are solitary, small, well-demarcated tan to
dark brown lesions.
Acquired melanocytic nevi (moles) usually develop on the skin in
both men and women, usually before 35 years of age, and are
small, slightly elevated brownish-tan papules.
Seborrheic keratosis is a common lesion of the skin (does not
occur in the mouth), usually in older people, and has a raisin-like
appearance.
Amalgam Tatoo: history of trauma during dental procedure.
Entrapment of amalgam fragments in tissue or mercury-collagen
salt formation.

Clinical presentation for Melanoma:

http://www.cancer.gov/cancertopics/types/melanoma

A patient presents with unilateral swelling of the left pre-auricular


area, exam reveals no odontogenic problems. Slight purulence
noted from Stensons duct upon manipulation. What is your next
step in treatment?
a. Surgical referral for removal of sialolithiasis
b. Systemic antibiotics, milking of the gland instructions, follow
up
c. 5% Citric acid (gustatory) stimulation to promote max salivary
flow and unblocking of parotid duct
d. Referral for MRI to rule out salivary neoplasm
e.

A patient presents with unilateral swelling of the left pre-auricular


area, exam reveals no odontogenic problems. Slight purulence
noted from Stensons duct upon manipulation. What is your next
step in treatment?
a.
b. Systemic antibiotics, milking of the gland instructions, follow
up
c.
d.

The most viral infection of salivary gland is mumps caused by


paramyxovirus (often causes parotiditis). Now rare in kids due to
vaccination. Other viruses such as Coxsackie A, ECHO,
choriomeningitis, parainfluenza, and cytomegalovirus can cause
infection of salivary gland. In case of mumps, the treatment is mainly
supportive.
Bacterial infections typically arise from ductal obstruction (i.e. salivary
gland stones) or poor oral hygiene. Commonly observed in dehydrated
or hospitalized individuals.The most common bacterial cause of
salivary gland infection is Staphylococcus aureus (can be Strep).
Non-infectious causes of salivary gland inflammation include Sjogren
syndrome, sarcoidosis, radiation therapy, and various allergens.
Neville, BW et al., Oral and Maxillofacial Pathology, 2nd ed.
Saunders 2002.
Mumps Treatment: Ice or heat packs application. Tylenol for pain relief.
(Caution Aspirin for children with viral infection: Reye Syndrome).
Symptom relief: plenty of fluids, soft foods, warm salt water rinses.
Salivary Gland infection Treatment:
In some cases, no treatment is necessary.
If there is pus or a fever, or if the infection is known or thought to be
bacterial, antibiotics may be prescribed. Antibiotics are not effective
against viral infections. Home care instructions and follow up are
indicated. Medications and/or additional methods to increase salivary
flow may be indicated (i.e. sugar free gum)
If there is an abscess, surgical drainage or aspiration may be done.
Good oral hygiene, with thorough tooth brushing and flossing at least
twice per day, may aid healing and help prevent an infection from
spreading. If you are a smoker, stop smoking as it helps in recovery.
Warm salt-water rinses (1/2 teaspoon of salt in one cup of water) may
be soothing and keep the mouth moist.

Drink lots of water and use sugar-free lemon drops to increase the flow
of saliva and reduce swelling. Massaging the gland with heat may help.
http://www.nlm.nih.gov/medlineplus/ency/article/001557.htm
http://www.nlm.nih.gov/medlineplus/ency/article/001041.htm

Which of the following lesions would have the highest probability to


be malignant?
a.
b.
c.
d.

Leukoplakia
Linea Alba
Erythroplakia
Mixed Leukoplakia/Erythroplakia

Which of the following lesions would have the highest probability to


be malignant?
a.
b.
c. Erythroplakia
d.

Leukoplakia: whitish plaque that does not rub off. Most leukoplakic
lesions are hyperkeratotic responses, but about 20% have been shown
to exhibit some evidence of dysplasia or carcinoma upon discovery.
Leukoplakia/ Erythroplakia (with some interspersed Erythoplakia):
has an increased malignant transformation rate 24%/
Erythroplakia: (far less common in occurrence) has a very high
probability of showing dysplastic or malignant changes upon discovery
(91%).
Lichen Planus: (some debate over this one): 24% of Lichen Planus
lesions exhibit dysplasic or malignancy.
DeJong WFB, Albrecht M, Banoczy J, van Der Waal I. Epithelial dysplasia
in oral lichen planus. Int J Oral Maxillofac Surg 1984;13:221-5.
Clinical Features of Oral Premalignancy
You should biopsy lesions that persist for 14+ days after any obvious
sources of etiology have been addressed.
Age: Majority of cases are diagnosed between age 50-69.
HIGH Risk for malignant change:
1) erythroplakia (by itself or within a leukoplakia)
2) proliferative verrucous appearance,
3) high-risk site: ventral surface of tongue or floor of mouth,
4) multiple lesions,
5) no smoking history
Anatomic Location:
Leukoplakia discovered at the floor of the mouth or ventral surface of
the tongue the malignant potential goes up to 45%.
Different Studies report differing results.

21.8% of oral epithelial dysplasias occurred on the buccal mucosa,


13.7% on the palate, and 12.3% on the floor of the mouth.
Kaugars GE, Burns JC, Gunsolley JC. Epithelial dysplasia of the oral
cavity and lips. Cancer 1988;62:2166-70.
Diagnosis
Requires Biopsy.
Toludine Blue can be utilized and only has false-negative, false-positive
rates of 10% (reportedly)
Remember Vital Staining is only an adjunct not a replacement. If the
Toludine blue stains heavily on a lesion (binds to DNA and RNA both
increased in dyplasia or malignancy) that site would need to be
followed up with a conventional biopsy.
More in-depth info:
http://www.oralcancerfoundation.org/cdc/cdc_chapter4.htm
A CDC Oral Cancer Background Paper

Which of the following may yield a unilateral widening of the PDL


space along the length of the root on a mandibular molar?
a.
b.
c.
d.

Osteosarcoma
Keratocystic Odontogenic Tumor
Recent restoration high in occlusion
Periapical Granuloma

Which of the following may yield a unilateral widening of the PDL


space along the length of the root on a mandibular molar?
a. Osteosarcoma
b.
c.
d.

Unilateral Widening of PDL membrane is associated with


Osteosarcoma, but is also seen in other malignancies
Widening of PDL space may be seen in the
following situations:
Concussion
dental root fractures
extrusion
inflammatory lesions
luxated tooth
close proximity to the mandibular canal
metastatic tumors
periapical cemental dysplasia
periapical inflammatory lesions
progressive systemic sclerosis (scleroderma)
root dilaceration
sarcomas
vertical osseous defects
Brown tumor (associated with Hyper Parathyroidism)
Periapical lesions; widening concentrated at Apex.
Perio Disease: widening noted at alveolar crest.
Chronic Infections: root resorption and cortical boundaries may occur.
Ref:
White. Pharoah. Oral Radiology: Principles and Interpretation. 5th
ed. 2004 p. 170, 283, 459, 467.

What are the ingredients of Tylenol 3?


a.
b.
c.
d.

Acetaminophen
Acetaminophen
Acetaminophen
Acetaminophen

500mg
300mg
250mg
300mg

/
/
/
/

Codeine
Codeine
Codeine
Codeine

50mg
30mg
50mg
10mg

What are the ingredients of Tylenol 3?


a.
b. Acetaminophen 300mg / Codeine 30mg
c.
d.

Each tablet of Tylenol 3 contains:


Acetaminophen. . . . . . . . . . . . . . . . . . . 300 mg
No. 3 Codeine Phosphate . . . . . . . . . . . 30 mg
(Warning: May be habit forming)
Acetaminophen. . . . . . . . . . . . . . . . . . . 300 mg
No. 4 Codeine Phosphate . . . . . . . . . . . 60 mg
Codeine is a narcotic analgesic and antitussive.
TYLENOL with Codeine No. 3 contains powdered cellulose,
magnesium stearate, sodium metabisulfite, pregelatinized starch
(corn), and modified starch (corn).
Rx: Tylenol 3 x 30 tabs. Sig: Take 1 or 2 tablets every 4 hours as
needed for pain.
Other Meds
Vicodin
Hydrocodone
. . . . . . . . . . . 5 mg
Acetaminophen. . . . . . . . . . . . . . . . . . . 325 mg
Percocet
Oxycodone
. . . . . . . . . . . 5 mg (may come in 2.5 or 7.5)
Acetaminophen. . . . . . . . . . . . . . . . . . . 325 mg
www.rxlist.com

A 38 year old black female presents for routine examination. Her


panoramic exhibits focal radiolucencies around #24,25. Her teeth
all test vital. What is the most likely diagnosis?
a.
b.
c.
d.
e.

Focal Cemento Osseous Dysplasia


Compound Odontoma
Florid Cemento Osseous Dysplasia
Complex Odontoma
Idiopathic Osteosclerosis

A 38 year old black female presents for routine examination. Her


panoramic exhibits focal radiolucencies around #24,25. Her teeth
all test vital. What is the most likely diagnosis?
a. Focal Cemento Osseous Dysplasia
b.
c.
d.
e.

Radiolucency around a non-vital tooth: Granuloma, periapical cyst,


Radiolucency around vital tooth: Lateral Periodontal cyst, CementoOsseous Dysplasia.
Nasopalatine Duct Cyst: Seen in Max midline over roots of #8/9. Spade
or hear shaped. Teeth test vital.
Dentgerous cyst, OKC, Ameloblastoma: all around vital teeth (coronal)
(improbable to test these)
Cemento-Osseous Dysplasia: commonly seen in black females,
associated with Mand anterior. Lesions appear radiolucent early and
shift to a mixed lesion and eventually a radiopaque lesion. These teeth
are vital.
Ref.
Neville, Damn, Allen, Bouquot. Oral & Maxillofacial Pathology. 2nd
edition. p. 27-30, 113, 116, 602, 557.

Treatment of a mucocele, without removal of neighboring minor


salivary glands, may lead to the following?
a.
b.
c.
d.

Development of a mucous extravasation phenomenon.


Localized hematoma
Recurrance of the mucocele
Salivary gland fibrosis

Treatment of a mucocele, without removal of neighboring minor


salivary glands, may lead to the following?
a. Development of a mucocele extravasation cyst.
b.
c.
d.

Mucous Extravasation Cyst or Mucous Retention cyst: a.k.a. Mucocele,


Ranula, or Plunging Ranula.
Not considered a true cyst (no epithelial lining), result from disruption
of a salivary gland duct.
Bluish, dome shaped, well defined swelling. Mucin pools into the
surrounding soft tissues. Etiology is most often trauma. If punctured
will drain, but commonly recurs.
Treatment of mucocele is removal of lesion along with associated minor
salivary gland. Warn patient that risk of surgery is potential trauma to
neighboring minor salivary glands that could give rise to a future
mucocele, unless these are removed.
Common location: Mucocele- lower lip (never max lip)
Ranula- floor of the mouth
A ranula is larger than a mucocele. Origin: sublingual gland or
submandibular duct
A plunging ranula occurs when the lesion causes dissection through
the mylohyoid muscle. Swelling is commonly seen on the neck.
Ref.
Neville and Damm, Oral and Maxillofacial Pathology, 2nd ed., pp. 38992.

Which of the following is most characteristic of malignancy?


a.
b.
c.
d.

Slow growing
Painless swelling
Spiking of the roots
Well defined borders

Which of the following is most characteristic of malignancy?


a.
b.
c. Spiking of the roots
d.

Malignant
poorly defined or moth-eaten
borders
root resorption
irregular or asymmetrical widening
of the PDL
onion skinning (Ewings sarcoma)
Fast growing
Painful (not always)
Metastasis Potential

Benign
well-defined or corticated borders
displacement of roots
expansion of bone without destruction
of the cortical plate
Slow growing
Typically painless
No Metastasis

ABCDE System :
For pigmented lesions: Malignant Melanoma vs Benign Lesion (ie
Nevus)
Malignant Melonma
Asymmetry

Assymetrical

Borders

Irregular

Variegation (color
varies) (different shades
of brown black or tan)
Diameter: moles greater >6mm more indicative
than 6 mm are more
of Melanoma (size of
likely to be melanomas
pencil eraser)
than smaller moles
Enlarging: Enlarging or
Enlarging or evolving
evolving
Color

Benign Lesion
Symmetrical

Symmetrical coloring
<6mm

Unchanging

TMN staging chart


Malignant Tumors will be staged according to the following AJCC
classification:

The TNM system is based on the extent of the tumor (T), the extent of
spread to the lymph nodes (N), and the presence of metastasis (M). A
number is added to each letter to indicate the size or extent of the
tumor and the extent of spread.
Primary Tumor (T)
TX
Primary tumor cannot be evaluated
T0
No evidence of primary tumor
Tis
Carcinoma in situ (early cancer that has not spread to
neighboring tissue)
T1, T2, T3, T4
Size and/or extent of the primary tumor
Regional Lymph Nodes (N)
NX
Regional lymph nodes cannot be evaluated
N0
No regional lymph node involvement (no cancer found in the
lymph nodes)
N1, N2, N3 Involvement of regional lymph nodes (number and/or
extent of spread)
Distant Metastasis (M)
MX Distant metastasis cannot be evaluated
M0 No distant metastasis (cancer has not spread to other parts of
the body)
M1 Distant metastasis (cancer has spread to distant parts of the
body)
For example, breast cancer T3 N2 M0 refers to a large tumor that has
spread outside the breast to nearby lymph nodes, but not to other
parts of the body. Prostate cancer T2 N0 M0 means that the tumor is
located only in the prostate and has not spread to the lymph nodes or
any other part of the body.
For many cancers, TNM combinations correspond to one of five stages.
Criteria for stages differ for different types of cancer. For example,
bladder cancer T3 N0 M0 is stage III; however, colon cancer T3 N0 M0
is stage II.
Stage Definition
Stage 0
Carcinoma in situ (early cancer that is present only in the
layer of cells in which it began).
Stage I, Stage II, and Stage III Higher numbers indicate more
extensive disease: greater tumor size, and/or spread of the cancer to
nearby lymph nodes and/or organs adjacent to the primary tumor.
Stage IV The cancer has spread to another organ.
Tumor Grading:
Tumor grade is a system used to classify cancer cells in terms of how
abnormal they look under a microscope and how quickly the tumor is
likely to grow and spread.

The American Joint Commission on Cancer recommends the following


guidelines for grading tumors (1):
Grade
GX
G1
G2
G3
G4

Grade cannot be assessed (Undetermined grade)


Well-differentiated (Low grade)
Moderately differentiated (Intermediate grade)
Poorly differentiated (High grade)
Undifferentiated (High grade)

Staging more important than grade!


Ref.
Neville and Damm, Oral and Maxillofacial Pathology, 2nd ed.
http://www.cancer.gov/cancertopics/factsheet/Detection/tumor-grade

The following is true regarding non-selective beta-blockers and


epinephrine:
a.
b.
c.
d.
e.

Both stimulate beta1 and beta2 receptors


Both block all beta receptors
When combined, an excessive response to epinephrine occurs
When combined, a diminished response to epinephrine occurs

The following is true regarding non-selective beta-blockers and


epinephrine:
a.
b.
c. When combined, an excessive response to epinephrine occurs
d.

Type of Beta
Receptor

Location

Beta 1

Heart, Kidneys, Eyes

Beta 2

Lungs, Liver, Blood vessels, Uterus, Gastrointestinal


tract, Skeletal muscles

Beta 3

Fat cells

Epinephrine stimulates all beta receptors, while norepinephrine


stimulates only beta 1 and beta 3 receptors. Stimulation of beta
receptors leads to increase in heartbeats and blood pressure, and
constriction of blood vessels.
For different beta receptors, there are different beta blockers. Nonselective beta blockers block beta 1 and beta 2 receptors. Selective
beta blockers, also called as cardioselective beta blockers, block only
beta 1 receptors.
Name

Type of Beta
Blocker

Disease Treated

Alprenolol

Non-selective

Angina pectoris

Carteolol

Non-selective

Glaucoma

Levobunolol Non-selective

Glaucoma

Metipranolo
Non-selective
l

Glaucoma

Nadolol

Non-selective

High blood pressure, Migraine, Chest


pain

Oxprenolol

Non-selective

Angina pectoris, Arrhythmia

Acebutolol

Selective

Hypertension, Arrhythmia

Atenolol

Selective

Cardiovascular diseases

Betaxolol

Selective

Hypertension, Glaucoma

Metoprolol

Selective

Hypertension

The two most dangerous outcomes of the interaction of nonselective


beta-blockers and epinephrine are hypertensive crisis and anaphylaxis
that does not respond to epinephrine. In the first case, an excessive
response to epinephrine occurs, and in the latter case, the epinephrine
response is inhibited.
What causes the hypertensive reaction?
Epinephrine has both alpha-adrenergic effects (producing
vasoconstriction) and beta-adrenergic effects (producing vasodilation).
If a patient on a nonselective beta-blocker receives a systemic dose of
epinephrine, however, the beta-blocker prevents the vasodilation,
leaving unopposed alpha vasoconstriction. The resulting hypertensive
reaction can be large, with systolic pressure well over 200 mm Hg.
Cardioselective beta-blockers (such as acebutolol, atenolol, betaxolol,
bisoprolol, esmolol, and metoprolol) would not be expected to cause
hypertensive reactions following a systemic dose of epinephrine,
because cardioselective beta-blockers have little effect on the betaadrenergic receptors in the arterioles
Is the dose of the epinephrine important?
Yes. The small amounts of epinephrine (combined with local
anesthetics) that may be used in routine dental procedures are unlikely
to be a problem. Nonetheless, some patients undergoing surgery with
lidocaine and epinephrine injections have developed hypertensive
reactions.
What about the anaphylaxis problem?
It is well documented that patients on beta-blockers who develop
anaphylaxis are likely to be resistant to the therapeutic effects of
epinephrine used to treat the anaphylaxis. Although most of the
reports have involved nonselective beta-blockers, it is generally
assumed that all beta-blockers can inhibit the response to epinephrine
in anaphylaxis.
There also is evidence that patients on beta-blockers have an
increased incidence and severity of anaphylaxis, which is another
reason for concern.
www.PharmacyTimes.com
References:
Yagiela JA, Adverse Drug Interactions in Dental Practice: Interactions
Associated with Vasoconstrictors, J Am Dent Assoc. Vol. 130, May 1999

John R. Horn, PharmD, FCCP, and Philip D. Hansten, PharmD The


Dangers of Beta-Blockers and Epinephrine; Published Online: May 15,
2009 Pharmacy Times | 05.09 n 31
Little JW, Falace DA, Miller CS, Rhodus NL. Dental Management of the
Medically Compromised Patient. 7th ed. St Louis: Mosby Elsevier; 2008

The most useful therapeutic agents for systemic and local relief of
severe lichen planus are:
a.
b.
c.
d.
e.
f.

antibiotics
antifungals
analgesics
antipyretics
immunosuppressants

The most useful therapeutic agents for systemic and local relief of
severe lichen planus are:
a.
b.
c.
d.
e. immunosuppressants

Lichen planus is a mucocutaneous autoimmune disorder with a genetic


predisposition that is initiated by a variety of factors, to include
emotional stress, hypersensitivity to drugs, dental products or foods.
No known cure exists. Systemic and local relief with anti-inflammatory
and immunosuppressant agents is indicated to provide oral comfort if
the lesions are symptomatic. Treatment or prevention of a secondary
fungal infection with a systemic antifungal agent also should be
considered.
CORTICOSTEROIDS are the single most useful group of drugs in the
management of patients with lichen planus, due to their ability to
modulate inflammatory and immune response. Topical application and
local injection can be used. In severe cases systemic steroids may be
used for initial management.
Immunosuppressant drugs can be classified according to their specific
molecular mode of action. The four main categories of
immunosuppressant drugs currently used in treating patients with
transplanted organs are the following:
Cyclosporins (Neoral, Sandimmune). These drugs act by inhibiting Tcell activation, thus preventing T-cells from attacking the
transplanted organ.

Azathioprines (Imuran). These drugs disrupt the synthesis of DNA


and RNA as well as the process of cell division.

Monoclonal antibodies, including basiliximab (Simulect), daclizumab


(Zenpax), and muromonab (Orthoclone OKT3). These drugs act by
inhibiting the binding of interleukin-2, which in turn slows down the
production of T-cells in the patient's immune system.

Such corticosteroids as prednisolone (Deltasone, Orasone). These


drugs suppress the inflammation associated with transplant
rejection.

GLUCOCORTICOIDS AND THEIR RELATIVE POTENCY (using 20 mg over a


24h period as the normal secretion rate of cortisol):
Compound
Antiinflammatory
Approximate
Potency
equivalent dose (mg)
Short-acting (<12
hours)
Cortisol
1
20
Cortisone
0.8
25
Intermediate-acting
(12 to 36 hours)
Prednisone
4
5
Prednisolone
4
5
Methylprednisolone
5
4
Triamcinolone
5
4
Long-acting (>36
hours)
Paramethasone
10
2
Betamethasone
25
0.75
Dexamethasone
25
0.75
References:
Little JW, Falace DA, Miller CS, Rhodus NL: Dental Management of the
Medically Compromised Patient. Mosby Ed., 2002.
www.surgery.com/article/immunosuppressant-drugs

Which of the following is not used in the treatment of tuberculosis?


a.
b.
c.
d.
e.
f.

ethambutol
isoniazid
pyrazinamide
streptomycin
vancomycin

Which of the following is not used in the treatment of tuberculosis?


a.
b.
c.
d.
e. vancomycin
Tuberculosis is a chronic bacterial infection of the lungs and other
organs caused by the tubercle bacilli, Mycobacterium tuberculosis, and
Mycobacterium bovis. In the United States disease caused by M. bovis
is rare.
The typical mode of transmission of the bacteria is by way of infected,
airborne droplets of mucus or saliva that have been forcefully expelled
from the lungs, most commonly by a cough but also by sneezing and
talking.
The interval from infection to development of active TB is widely
variable, ranging from a few weeks to decades. Most cases of
tuberculosis are due to reactivation of a tubercle; only about 10% of
cases result from the initial infection. The number of organisms inhaled
and the level of immunocompetency determine largely whether the
disease is contracted.
M. tuberculosis is obligate aerobe, growing best in oxygen
concentrations similar to that found in alveolar air (140 mm Hg). This
may account for the frequent appearance of infection in organs with
high oxygen content (kidneys, bones and the apices of the lungs).
Lesions are resistant to most common antibiotics. A combination of
two antimycobacterial agents is most commonly used: isoniazid and
rifampin.

References:
American Thoracic Society, CDC, and Infectious Diseases
Society of America: Treatment of Tuberculosis.
Recommendations and Reports, June 20, 2003 / 52(RR11);1-77
Michael D. Iseman: Section XXIII Infectious Diseases - 345
TUBERCULOSIS. CECIL TEXT BOOK of MEDICINE. Ed. Elsevier.
Little JW, et al: Dental Management of the Medically Compromised
Patient, 6th ed.
Rose and Kaye: Internal Medicine for Dentistry, 1990, Mosby

Which of the following is not an advantage of MRI when imaging the


TMJ?
a.
b.
c.
d.
e.

Superior hard tissue characterization


Superior contrast resolution
Superior multiplanar capabilities
No exposure to ionizing radiation

Which of the following is not an advantage of MRI when imaging the


TMJ?
a. Superior hard tissue characterization
b.
c.
d.

Medical images produced with magnetic resonance are tomographic


images based on a matrix of numbers using a gray tone display in
which each gray level represents the physical property of the tissue
voxel (volume element). The numerical value assigned to each voxel is
proportional to the intensity of the radiowave signal emanating from
the tissue in which nuclei (usually hydrogen) have been perturbed by a
characteristic radio frequency energy input. The contrast depicted on
MR images is directly related to this radiowave signal intensity radiated
from the tissue. On the image, it has arbitrarily been decided to display
high signal areas as white. Areas of reduced signal are displayed as a
darker gray tone.
Some advantages of magnetic resonance imaging include:
1. Superior contrast resolution
2. Superior tissue characterization
3. Superior multiplanar capabilities
4. No exposure to ionizing radiation
5. Possibly safer intravenous contrast agents
6. Angiography without using contrast.
Disadvantages include:
1. Expense
2. Sensitive to motion artifacts
3. Many "technical" artifacts are possible
4. Some patients cannot be examined
5. It is difficult to monitor patients during the study

6. Prolonged imaging times


Magnetic resonance (MR) can be used to diagnose internal
derangement and other disorders of the TMJ. The patient is scanned in
the sagittal plane using a surface coil and a high resolution technique.
The low intensity cortex of the condyle surrounds the high signal fat in
the marrow. The meniscus is a low intensity structure which is attached
posteriorly by the intermediate intensity bilaminar zone. Normally, the
anterior band lies immediately in front of the condyle. The junction of
the bilaminar zone and the meniscus normally lies at the superior
aspect of the condyle.

normal TMJ MR showing normal meniscus (m) posterior and superior to


condyle (C) -- the articular eminence (E) and auditory canal (AC) are
also shown
In internal derangement, the meniscus is abnormally positioned
anterior to the condyle.

displaced meniscus (arrows, m) anterior to the condyle (C) and


auditory canal (AC) and beneath the articular eminence (E)
Computed Tomography
CT is the procedure of choice for studying the hard tissue components
of the TMJ. Images acquired in one plane of view may be digitally

enhanced and reconstructed by computer to show the condyle and


glenoid fossa from virtually any perspective in both two and three
dimensions.

Magnetic Resonance Imaging


MRI is a noninvasive procedure that does not use X-rays. It is the only
method available that will directly and consistently show soft tissue
components of the TMJ. MRI is useful for locating the position of the
disk and for the assessment of inflammatory conditions of the TMJ
area.

Conventional Film Tomography


Conventional film tomography, like CT, is used for the assessment of
hard tissue components of the TMJ. Individually corrected lateral and
coronal views of the TMJ are recorded on radiographic film. Unlike CT,
these images are not digital and may not be manipulated.

References:
University of Washington, Diagnostic Radiology Anatomy Modules - TMJ
Tutorial: http://uwmsk.org/tmj/mr.html
Texas A&M Health Science Center, Baylor College of Dentistry
University of South Carolina Radiology web site

Which of the following is not a radiographic change noted in


hyperparathyroid disease:
a.
b.
c.
d.
e.

complete or partial loss of lamina dura


alveolar bone demineralization
step-ladder appearance of the bony trabeculae
giant-cell lesions of bone (radiographic cysts)

Which of the following is not a radiographic change noted in


hyperparathyroid disease:
a.
b.
c. step-ladder appearance of the bony trabeculae
d.

Hyperparathyroid disease results in hypercalcemia.


Clinical Features of Hypercalcemia
Site or System Manifestation
Oral
Jaw bone demineralization, loss of
lamina dura, osteitis fibrosa cystica
Gastrointestinal
Anorexia, constipation, nausea, pain
Genitourinary
Polydipsia or polyuria,
calculi,nephroinsufficiency
Neurologic
Fatigue, muscle weakness,
disorientation or stupor, coma or
death
Psychiatric
Apathy, depression, psychotic
behavior
Ocular, kidney, vascular, periarticular Calcifications (no association with
salivary stones)
From the dental point-of-view, the diagnosis of disturbed calcium
metabolism is based upon the abnormal radiographic findings of
altered lamina dura, demineralized bone, or radiographic cysts. When
biopsy specimens from bone show giant cells in the fibrous stroma,
hyperparathyroidism must be ruled out.
Biochemically, the diagnosis is established by parathyroid hormone
levels and, indirectly, by serum levels of high calcium, low phosphate,
and high alkaline phosphatase.
Confusion in determining the differential diagnosis often arises
because of many nonendocrine conditions that may mimic
characteristics found in hypercalcemic states.
The dental findings of hyperparathyroid disease are usually found as
abnormalities on dental radiographs.
The changes include one or more of the following:
(1) complete or partial loss of lamina dura;
(2) alveolar bone demineralization (ground-glass appearance);

(3) fibrous giant-cell bone lesions (brown tumor, osteitis fibrosa


cystica), which must be
confirmed by biopsy.
Bone changes almost always reflect late-stage disease.
AB
C
Figure: Hyperparathyroidism.

A, Demineralized maxillary bone


with ground-glass appearance
and loss of lamina dura.

B, Osteitis fibrosa cystica of skull


(multiple radiolucent giant-cell
lesions).

C, Osteitis fibrosa cystica of


mandibular bone, which can
resemble other types of
nonendocrine bone pathology.

D, Giantcell lesion (brown


tumor) of osteitis fibrosa cystica

Differential Diagnosis: Serum, Radiographic, and Microscopic Findings

Disease or
Condition
Hyperparathyr
oidism
Renal rickets
and
hyperparathyr
oidism
Fibrous
dysplasia
Pagets
disease
Osteomalacia

APL

Loss
of
Lam
ina
Dur
a

Groundglass
Appeara
nce*

Bo
ne
Cy
st

Gia
nt
Cel
ls

Increa
se

Incre
ase

Increase

Increa
se

Incre
ase

Decre
ase

Parathor
mone

Seru
m
Calci
um

Increase

Incre
ase
Incre
ase

Giant-cell
0
0
0
granuloma
Hemorrhagic
0
0
0
cyst
Osteoporosis
0
0
0
*Radiolucency.
Demineralization.
APL = alkaline phosphatase; + = present; 0

= normal or absent.

Hypoparathyroidism is relatively rare and may be attributable to a


genetic disorder (congenital), or induced by parathyroid or thyroid
surgery, or radiation to that area. These events can lead to a low
serum calcium level (hypocalcemia), which can lead to muscular
spasms and tetany. The dental manifestations only occur in the
congenital form, which affects the teeth at the developmental stage,
causing mottling (hypoplastia) and discoloration.
There are no dental findings in acquired hypoparathyroidism.

Congenital hypoparathyroidism showing mottled enamel from


decreased serum calcium during tooth calcification.
Treatment for calcium disorders requires surgery for gland tumors and
medications to adjust calcium and phosphorus levels. Abnormalities in
bone revert to normal after calcium metabolism is stabilized.
Therefore, specific local treatment to the dental structures, other than
routine care, is contraindicated. It must be remembered that similar
radiographic bone alterations may represent several different diseases.
Correlations between clinical, radiographic, and microscopic findings
are critical to arriving at a definitive diagnosis. For example, giant-cell
lesions of bone can be the manifestation of an inflammatory process,
neoplasia, or a parathyroid adenoma.
References:
Silverman S, Eversole L, Truelove E: Essentials of Oral Medicine. BC
Decker Ed., 2001.

Which of the following would not be in your differential diagnosis of a


unilocular radiolucency?
a.
b.
c.
d.
e.
f.

ameloblastoma
aneurysmal bone cyst
calcifying odontogenic cyst
cemento-osseous dysplasia
Stafne bone defect

Which of the following would not be in your differential diagnosis of a


unilocular radiolucency?
a.
b. aneurysmal bone cyst
c.
d.
e.

PERIAPICAL RADIOLUCENCIES
- Granuloma
- Radicular cyst
- Abscess
- Apical scar
- Surgical defect
- Periodontal disease
- Chronic suppurative
osteomyelitis
- Periapical cemental
dysplasia (osteolytic stage)
PERICORONAL RADIOLUCENCIES
Do NOT contain radiopacities:
- Follicular space
- Dentigerous cyst
- Unicystic (mural)
ameloblastoma
- Odontogenic keratocyst
- Ameloblastoma
- Ameloblastic fibroma
MAY contain radiopacities:

Cementoblastoma
(osteolytic stage)
Cementifying and ossifying
fibromas (osteolytic stage)
Odontogenic and
nonodontogenic cysts
Odontogenic and
nonodontogenic benign
tumors
Malignant tumors
Ameloblastic fibro-odontoma
Adenomatoid odontogenic
tumor
Calcifying epithelial
odontogenic tumor
(Pindborg tumor)
Calcifying odontogenic cyst
(Gorlin cyst)

SOLITARY CYSTLIKE RADIOLUCENCIES


Normal anatomy and Variations
- Odontogenic keratocyst
- Tooth follicle (early stage)
Fissural cysts:
- Large marrow space (wide
- Incisive canal (nasopalatine)
inter-trabecular space)
- Globulomaxillary
- Post extraction socket
- Median palatine
- Surgical defect
- Median mandibular
Odontogenic cysts:
False cysts:
- Primordial
- Traumatic bone cyst
- Lateral periodontal
- Stafne bone cavity
- Residual
Odontogenic tumors:

Ameloblastoma
Odontogenic adenomatoid
tumor
Calcified epithelial
odontogenic tumor(Pindborg
tumor)
Ameloblastic fibroma
Cementifying and ossifying
fibromas (osteolytic stage)

Giant cell and


nonodontogenic tumors:
Central giant cell granuloma
Giant cell lesion of
hyperparathyroidism
Non-odontogenic benign
tumors

SOLITARY RADIOLUCENCIES WITH DIFFUSE IRREGULAR


BORDERS
- Chronic alveolar abscess
- Fibrous dysplasia (early
- Chronic osteomyelitis
stage)
- Osteoradionecrosis
- Histiocytosis X
- Squamous cell carcinoma
- Osteosarcoma and
- Metastatic tumors to the
chondrosarcoma
jaws
- Ewing's sarcoma
- Burkitt's lymphoma
MULTILOCULAR RADIOLUCENCIES
Tumors:
Cysts:
- Ameloblastoma
- Odontogenic keratocyst
- Odontogenic myxoma
(basal cell nevus syndrome)
- Central hemangioma of
- Aneurysmal bone cyst
bone
Others:
- Metastatic tumors to the
- Cherubism
jaws
- Central giant cell granuloma
- Giant cell lesion of
hyperparathyroidism
Prevalence of Cystic Mandibular Lesions

*Contains cystic and solid components.

Full Text (PDF)


Hyperplastic dental follicles
Are associated with teeth and appear as pericoronal, unilocular
radiolucencies.
Dentigerous cyst
Results from cystic changes in the enamel organ of a tooth after the
crown has been formed. It is often associated with the impacted teeth.
Primordial cyst
Cystic changes take place in the stellate reticulum of a tooth germ
before mineralization begins. The tooth then does not develop and a
primordial cyst forms instead.
Periapical granuloma (Chronic Apical Periodontitis)
Presents radiographically as a unilocular well circumscribed radiolucent
lesion at the root apex, associated with a non-vital tooth. The lesion
may occasionally be displaced laterally and the tooth may show deep
caries, a large restoration, or evidence of previous endodontic therapy.
Periapical cyst (Radicular Cyst) is a proliferation of odontogenic
epithelium in a pre-existing periapical granuloma. The odontogenic
epithelium originates from the rest of Malassez.
Periapical cemento-osseous dysplasia
Can present as a radiolucenct, radioopaque or mixed lesion and is
commonly found apical to the mandibular anterior teeth. Periapical
cemento-osseous dysplasia is associated with VITAL teeth. Teeth
involved are usually free of caries or restorations. The lesions are
usually asymptomatic and are more common in middle-aged AfricanAmerican females.
Stafne bone defect
Well defined unilocular radiolucent lesion located at the angle of the
mandible, below the inferior alveolar canal.
Traumatic bone cyst
It is not a true cyst, as it lacks an epithelial lining and is often empty. It
displays a cyst-like radiographic appearance. The cause is unknown, but it
is thought to be a result of trauma.
Periapical scars
Not to be confused with residual (periapical) cysts, which are
manifestations of incomplete removal of inflammatory tissue at the time
of extraction, are composed of dense fibrous tissue. Periapical scars most
commonly occur when buccal and lingual cortical plates are lost due to

the inflammatory response.


Nasopalatine duct cyst
Classic location: between and apical to the maxillary central incisors.
Lateral radicular cysts
Not to be confused with lateral periodontal cysts, which are of
developmental origin, are inflammatory in nature and are similar to
periapical cysts with the exception of their location.
Lateral periodontal cyst
Arises from epithelial rests in the periodontium lateral to the root. It is
considered to be the intrabony counterpart of the gingival cyst of the
adult.
Focal Sclerosing Osteomyelitis (Condensing Osteitis)
Presents as a circumscribed radiolucent lesion at the root apex of a tooth
with a deep carious lesion. Chronic Osteomyelitis that develops at the
root apex of a tooth with chronic pulpitis secondary to deep caries.
Mandibular molars and premolars most frequently involved. The pulps are
almost always nonvital.
Calcifying Epithelial Odontogenic Tumor (CEOT) (Pindborg tumor)
Presents as a unilocular or multilocular lesion with focal or diffuse
calcifications producing a mixed radiolucent-radiopaque appearance.
Benign neoplasm that arises from the reduced enamel epithelium that
lines the dental follicle or from remnants of the dental lamina. It is
commonly associated with the crown of an unerupted mandibular
molar. It is more common in adults and cortical expansion may be
present.
Aneurismal bone cyst
Presents as a multilocular radiolucency, especially in younger patients.
Ameloblastoma
It has a multilocular soap bubble appearance. It can appear as a
unilocular radiolucency in the early stages.

Ref.
Neville, Damn, Allen, Bouquot. Oral & Maxillofacial Pathology. 2nd
edition.
Radiologic and Pathologic Characteristics of Benign and Malignant
Lesions of the Mandible. RadioGraphics November 2006 vol. 26 no. 6
1751-1768

The most appropriate biopsy technique for a diffuse intraoral soft


tissue lesion would be:
a. Incisional biopsy at the best site (indicated with toluidine blue
staining)
b. Excisional biopsy
c. Punch biopsy in the center of the lesion
d. Cytologic smear at the edge of the lesion
e. Fine needle aspiration anywhere into the lesion, deep into the
submucosal tissue
f.

The best technique for the biopsy of a diffuse intraoral soft tissue
lesion would be:
a. Incisional biopsy at the best site (indicated with toluidine blue
staining)
b.
c.
d.
e.

An incisional biopsy samples only a particular or representative part of


the lesion. If the lesion is large or has different characteristics at
different locations, more than one area of the lesion may require
sampling. If the area under investigation appears difficult to excise
because of its extensive size (i.e. larger than 1 cm in diameter),
hazardous location, or whenever the clinician suspects malignancy,
incisional biopsy is indicated.

Figure: Use of fluorescence visualization and toluidine blue (TB) to


select the biopsy site.
(a) Clinical picture of a nonhomogeneousleukoplakia under white light.
(b) The same lesion under direct fluorescent visualization. Note that
the extent of the lesion, as seen by loss of green autofluorescence, is
bigger than the clinical lesion seen by the unaided eye
(c) The same lesion stained with TB showing TB positivity with varying
intensity of TB staining. In our experience, the area of the lesion with
strong TB staining usually has worse histologic results than the less
stained or negative area of the lesion.

FULL TEXT
Excisional biopsy is used to remove the entire lesion. This is ideal for
small lesions, where wide enough margins can be taken (about 3x the
size of the lesion).
Any lesion that can be removed completely without mutilating the
patient is best treated by excisional biopsy. Pigmented and small
vascular lesions should also be removed in their entirety.
A punch biopsy is similar in that it takes the entire lesion, but a much
smaller margin.
Fine needle aspiration (FNA) is used for deep, hard to reach structures,
fluid filled, thyroid, or salivary gland tumors and intraosseous lesions
before surgical exploration.
Oral Brush Biopsy is a noninvasive method to evaluate oral mucosal
lesions for cellular atypia.
For excisional and incisional
biopsies, two incisions forming
an ellipse at the surface and
converging to V at the base
provide a good specimen and
leave a wound that is easy to
close.
Incisional biopsies should be
thin and deep, including normal
surface and underlying tissue. If
malignant cells are present at
the base of a lesion, a broad and
shallow incision may not obtain
these diagnostic cells.
Specimens should be
immediately places in 10%
formalin solution (4%
formaldehyde) with a volume at
least 20 times the volume of the
surgical specimen. The tissue
must be totally immersed in the
solution.

Ref.
R. J. Oliver, P. Sloan, M. N. Pemberton: Oral biopsies: methods and
applications. British Dental Journal Volume 196 No. 6 March 27 2004
emedicine.medscape.com/article/1079770-overview
J.P. Handlers: Diagnosis and Management of Oral Soft-tissue Lesions:
The Use of Biopsy, Toluidine Blue Staining, and Brush Biopsy. Copyright
2001 Journal of the California Dental Association
C.F. Poh et Al.: Biopsy and Histopathologic Diagnosis of Oral
Premalignant and Malignant Lesions. JCDA Vol. 74, No. 3 April 2008
Petersons Contemporary Oral and Maxillofacial Surgery, 4th ed.(2003)

Which of the following statements is false about INR:


a.
b.
c.
d.
e.

INR
INR
INR
INR

stands for International Normalized Ratio


is a measure of the PT
is a measure of the intrinsic pathway of coagulation
is increased in vitamin K deficiency

Which of the following statements is false about INR:


a.
b.
c. INR is a measure of the intrinsic pathway of coagulation
d.

INR (International normalized ratio)


The international normalized ratio (INR), prothrombin ratio (PR) and
prothrombin time (PT) are measures of the EXTRINSIC pathway of
coagulation. They are used in the measure of warfarin dosage, liver
damage and vitamin K status. The reference range for prothrombin
time is 7-10 seconds, the range for the INR is 0.8-1.2.
The prothrombin time (PT) is the time it takes plasma to clot after
addition of tissue factor (obtained from animals). This measures the
quality of the extrinsic pathway (as well as the common pathway) of
coagulation.
The speed of the extrinsic pathway is very affected by levels of
coagulation factor VII in the body. Factor VII has a short half-life and its
synthesis requires vitamin K. Deficiencies in vitamin K, which can be
caused by warfarin (Coumadin), liver damage, or an immature liver in
newborns, result in an increased prothrombin time.
Because of differences between different batches and manufacturers of
tissue factor (it is a biologically obtained product) the INR was devised
to standardize the results.
Each manufacturer gives an ISI (International Sensitivity Index) for any
tissue factor they make. This ISI says how their tissue factor compares
to other companies.
The INR is the ratio of a patient prothrombin time to a normal (control)
sample, raised to the power of the ISI.

The response to oral anticoagulants can vary greatly in patients, and


regular monitoring is warranted, with a current evaluation close to the
surgery date (the morning of or the day before).
Interpretation of PT and PTT in Patients with a Bleeding
Disorder

PT
PTT
RESUL RESUL
T
T
Prolong Normal
ed

POSSIBLE CONDITION PRESENT


Liver disease, decreased vitamin K, decreased or
defective factor VII

Normal Prolong Decreased or defective factor VIII, IX, or XI, or lupus


ed
anticoagulant present
Prolong Prolong Decreased or defective factor I, II, V or X, von
ed
ed
Willebrand disease, liver disease, disseminated
intravascular coagulation (DIC)
Normal Normal

Decreased platelet function, thrombocytopenia,


factor XIII deficiency, mild deficiencies in other
factors, mild form of von Willebrands disease

Ref.
www.labtestsonline.org/understanding/analytes/pt/test.html
www.doctorslounge.com/hematology/labs/inr.htm
Dental Management of the Medically Compromised Patient, 6th Edition.

Which of the following is not true about perioperative steroid


supplementation:
a. Recommended for patients who undergo long-term corticosteroid
therapy
b. Recommended for patients who have recently stopped (<1
month) long-term corticosteroid therapy
c. The stress dose should be proportional with the severity of the
surgical stress
d. Should be started one week prior to the surgery
e.

Which of the following is not true about perioperative steroid


supplementation:
a.
b.
c.
d. Should be started one week prior to the surgery

Most experts still recommend perioperative steroid coverage for


patients who are receiving more than 5 mg/day of prednisone or an
equivalent and for patients who have recently stopped long-term
steroid therapy.
The stress dose should be proportionate to the severity of surgical
stress and should be given for no longer than 1 to 3 days
perioperatively.
The physiologic rationale for steroid coverage is that long-term
corticosteroid therapy for chronic autoimmune or inflammatory
diseases (such as rheumatoid arthritis, ulcerative colitis, or asthma)
suppresses the hypothalamic-pituitary-adrenal (HPA) axis. In normal
patients, severe illness, trauma, stress, and surgery are accompanied
by activation of the HPA axis. Patients with HPA axis suppression from
long-term corticosteroid therapy may be unable to produce this
physiologic response to stress.
Patients with a history of steroid use
Some patients previously treated with glucocorticoids should also
receive steroid coverage. The literature suggests that, in some
patients, the HPA axis may not recover for up to a year after
glucocorticoid therapy is stopped, so it would be reasonable to
prescribe supplements for patients who have stopped long-term
glucocorticoid use within the past year. On the other hand, stress
doses are not required for patients who have recently received short
bursts of corticosteroids (therapy lasting 1 week or less), because in
these patients, HPA function recovers within 1 week.
Consensus recommendations on doses
Traditionally, the dosage used for steroid coverage has been 100 mg of
hydrocortisone every 8 hours, sometimes with a prolonged taper. This
dose is far higher than the physiologic cortisol increase, which peaks at
150 mg/day after major surgery and returns quickly to baseline. There
is no evidence to suggest that steroid supplementation needs to be
tapered over a prolonged period. A taper over 1 to 3 days is adequate
in uncomplicated situations, and this helps to minimize any adverse
effects of high-dose steroids.

Recommendation Perioperative
Hydrocortisone Dosage For
Patients On Long-Term Steroid
Therapy Surgery Type
Minor (Out patient)
Moderate (Total joint)

Major (CABG)

Stres
s
Dose

Durati
on

Order

25
mg
5075
mg

1 day

100150
mg

2-3
day

Hydrocortisone
25 mg X 1
Hydrocortisone
25 mg q12h x 1
days
then 25 mg x 1
next day
Hydrocortisone
50 mg IV q8hrs
X 1 day
Hydrocortisone
50 mg IV q12hrs
X 1 day
Hydrocortisone
25 mg IV q12hrs
X 1 day

1-2
days

Ref.
Ali Olyaei PharmD, 2004
Dental Management of the Medically Compromised Patient, 6th Edition.

Which of the following is not a sialogogue?


a. Cevimeline HCl (Evoxac)
b. Pilocarpine HCl (Salagen)
c. Citric acid
d. Oralube
e.

Which of the following is not a sialogogue?


a.
b.
c.
d. Oralube

A sialogogue is a product that stimulates salivary production. We


normally think of this as pharmacological stimulation, but it also
includes mechanical, chemical, and electrical stimulation of saliva.
Oral moisturizers / salivary substitutes are NOT sialogogues, though
they are often used by xerostomic patients if sialogogues have been
unsuccessful is stimulated a sufficient amount of saliva.
Cholinergic agents stimulate acetylcholine receptors of the major
salivary glands. The use of parasympathomimetic drugs such as
Pilocarpine hydrochloride (Salagen) can stimulate salivary gland
secretions and has been shown to be effective for patients with
Sjgrens syndrome and for those who have had irradiation therapy or
bone marrow transplantation. Another cholinergic agent, Cevimeline
hydrochloride (Evoxac), recently was approved for use in patients with
Sjgrens syndrome.
Pilocarpine HCl a prescription cholinergic agonist (e.g. Salagen).
This is the most commonly prescribed drug for treating xerostomia
Optimoist a chemical stimulant that contains citric acid
Cevimeline HCl a prescription cholinergic agonist (e.g. Evoxac)
Oralube a salivary substitute
Biotene oral rinse is a salivary substitute not a sialogogue
Biotene chewing gum is a mechanical stimulant (with xylitol) a
sialogogue
XEROSTOMIA ETIOLOGY, RECOGNITION AND TREATMENT
TEXT (PDF)

FULL

Ref:
Guggenheimer J, Moore P, Xerostomia Etiology, recognition and
treatment; J Am Dent Assoc; 134 (1): 61.
Lexi-Comp Drug Information Handbook for Dentistry 11th Ed.
Treatment of Drug-induced Xerostomia;
http://www.drymouth.info/practitioner/treatment.asp

Johnson JT, Ferretti GA, Nethery WJ, et al. Oral pilocarpine for postirradiation xerostomia in patients with head and neck cancer. N Engl J
Med 1993;329:3905

PEDODONTICS
The procedure known as Cvek pulpotomy is a:
a. Pulp cap
b. Partial pulpotomy
c. Deep pulpotomy
d. Pulpectomy

The procedure known as Cvek pulpotomy is a:


a.
b. Partial pulpotomy
c.
d.

Cvek showed in his research with monkeys that when you remove the
inflamed pulp tissue in young teeth, you encounter healthy pulpal
tissue about 2 mm below this. If hemorrhage is controlled, MTA can be
placed and allowed to harden and then a restoration placed in the
tooth and vitality can be retained and the root apical region can
completely form and if need be, conventional endodontics performed
with a better apical stop.
Cvek technique
- Anesthetize and isolate the tooth with a rubber dam
- Use a diamond bur to access the pulp and amputate the pulp
about 2mm below the exposure
- Make sure all tissue tags above this are removed or hemorrhage
control can be difficult
- Place cotton pellets moistened with saline against the stumps
and held in place with dry cotton pellets
- Once hemorrhage is controlled, MTA is placed against the pulp
stumps
- A thin layer of flowable composite is placed over this and light
cured
- The tooth is then sealed and restored with an acid etch bonded
composite restoration
- A Cvek pulpotomy is a partial pulpotomy. Its indications are the
same as for pulp capping. The idea behind the partial pulpotomy
is to allow some healthy pulp tissue to remain so as to allow the
root to fully develop, and allow the apex to constrict on its own.
It makes NSRCT easier since there will be a constriction present,
and bridging procedure will not need to be attempted. It is
performed under RDI with a sterile diamond at high speed. Care
is taken to avoid blood clot formation. A thin layer of CaOH is
placed over the pulp, and a restorative material is placed over
that to achieve a seal. Formocreosol is no longer the preferred
agent for controlling the heme. Ferric sulfate has been found to
have equal success when used to control heme during
pulpotomy procedures, and it is non-carcinogenic.

Ref.:
Camp JH, Fuks AB; Ch. 22 Pediatric Endodontics: Endodontic Treatment
for the Primary and Young Permanent Dentition; Pathways of the Pulp;
Mosby Publishing, St. Louis, MO, 2006; pp. 838,859-864.
Pathways of the pulp, 9th ed, pg 618, pgs 862-864, and Capt
Alexanders Pedo lectures.
Serial extraction is best to use during the early mixed dentition in
children with no skeletal problem and severe crowding. The minimum
space discrepancy recommended when using serial extraction to treat
severe crowding is:
a.
b.
c.
d.

5 mm per arch
10 mm per arch
15 mm per arch
15 mm for the two arches combined

Serial extraction is best to use during the early mixed dentition in


children with no skeletal problem and severe crowding. The minimum
space discrepancy recommended when using serial extraction to treat
severe crowding is:
a.
b.
c.
d.

5 mm per arch
10 mm per arch
15 mm per arch
15 mm for the two arches combined

Serial extraction is a procedure that should take place in children with


SEVERE crowding, during the early mixed dentition. By having a
planned sequence of tooth removal, crowding and irregularities in the
transition from primary to permanent dentition can be reduced.
Typically, baby teeth are removed with the expectation that certain
permanent teeth will erupt early; and their removal will allow most - if
not all space - to close naturally. This can significantly shorten the
duration of treatment, as well as its complexity, and greatly reduce its
cost.
Originally proposed by Kjellegren in 1948.
Classically it involves:
1) Extraction of primary canines as the permanent lateral incisors
are erupting, to allow incisors to align
2) Extraction of primary first molars to accelerate eruption of the
permanent first premolars (approximately 2 years after the
extraction of the primary canines)
3) Extraction of the permanent first premolars (re-evaluate the
crowding, then extract)
It is best to use this method when no skeletal problem exists and the
space discrepancy is large greater than 10mm per arch. The goal is
to influence certain permanent teeth to erupt without becoming
impacted, so foresight is necessary.
Ref.
B. Kjellgren. 1948 Acta Odontol Scand; 8(17).
Proffit WR. Contemporary Orthodontics 4th ed. Mosby Publishing
2007:490-92

The primate space:


a.
b.
c.
d.

In the mandibular arch is located distal of the primary canine


In the maxillary arch is located distal of the primary canine
When present in the primary dentition is an indication of overjet
Has first been observed in chimpanzees (therefore the name)

The primate space:


a. In the mandibular arch is located distal of the primary canine
b.
c.
d.

Premolars are smaller than the primary teeth they replace. This extra
space is called Leeway space. This space is used up in the Late Mesial
Shift.
Leeway space in each quadrant:
Maxilla - 1.5 mm
Mandible - 2.5 mm
Leeway space is the difference between the sum of the M-D width of C,
D, E and permanent teeth 3, 4 and 5. The mandibular primary second
molar is on average 2mm larger than the second premolar, while in the
maxillary arch, the primary second molar is 1.5mm larger. The primary
first molar is only slightly larger than the first premolar, but does
contribute an extra 0.5mm in the mandible. The result is that each
side in the mandibular arch contains about 2.5mm of Leeway space,
while the maxillary arch has about 1.5mm on each side. These
numbers do show some variability in different studies. When the
second primary molars are lost, the first permanent molars move
forward relatively rapidly into this Leeway space. This space can be
useful to relieve crowding if orthodontic treatment is needed.
Spacing is normal throughout the anterior part of the primary dentition
but is most noticeable in two locations called the primate spaces. Most
primates have this space throughout life (hence the name).
In the maxillary arch, the primate space is located between the lateral
incisors and canines.
In the mandibular arch, the space is located between the canines and
first molars.
The primate spaces are normally present from the time the teeth
erupt. Developmental spaces between the incisors are often present
from the beginning, but become somewhat larger as the child grows
and the alveolar processes expand. Generalized spacing of the
primary teeth is a requirement for proper alignment of the permanent
incisors.

primate space
Ref.
Contemporary Orthodontics, Fourth Edition. Proffit. 2007.
Contemporary Orthodontics, 3rd ed.,. Proffit. 2007.

Which of the following statements is false:


a. Tooth movement is associated with an inflammatory reaction of
the supporting bone and PDL
b. Tension of the PDL fibers results in bone apposition
c. Orthodontic intrusion requires more force than molar uprighting
d. Light and continuous force produces the most effective tooth
movement

Which of the following statements is false:


a.
b.
c. Orthodontic intrusion requires more force than molar uprighting
d.

Areas of compression on the periodontal ligament result in the


resorption of bone, while areas of tension on the periodontal ligament
result in the deposition of bone.
From lowest to highest, the force required to move teeth is:
Intrusion
Tipping/rotation/extrusion
Uprighting
Bodily movement (translation)
Light and continuous force, 24 hours per day, produces the most
efficient tooth movement, but successful tooth movement can be
produce by shorter durations, with a minimum of 6 hours per day.
Animal model s have shown that only after approximately 4 hours,
chemical changes in the PDL occur that stimulate cell differentiation,
leading to resorption/apposition of bone.
If a relatively constant force is applied on a tooth, the resulting
movement can be differentiated into three phases. The initial phase is
characterized by a period of rapid tooth movement that usually lasts a
few days. The onset occurs immediately after application of a force
and seems to represent displacement of the tooth in the periodontal
space. Next is a lag phase with little or no movement, followed by a
post lag phase during which the movement increases again. Various
theories have been developed to explain the lag phase, with the two
most common being that the lag is caused by nonvitalization of the
PDL in areas of maximal stress that halts movement until cellular
process revitalize the areas and that the lag period represents the
interval required for absorption of compact bone of the lamina dura.
The lag phase typically lasts during the second week and is followed by
an increase in movement that lasts for about an additional 2 weeks
Ref.
Proffit, Contemporary Orthodontics, 3rd ed., pp. 298-306.

Graber, Vanarsdall, Vig Orthodontics: Current Principles and


Techniques Elsevier Mosby 2005, p. 907.
Graber et al, Orthodontics Current Principles and techniques, 4th ed., p.
293-301 Copyright 2005 by Elsevier Inc., ISBN -13: 978-0-323-02621-5

Fig.In 1930, Oppenheim republished his earlier research on tooth


movement (which had been illustrated by drawings and criticized as
such) together with the original photomicrographs. This is Figure 1
from the paper showing (A) the original drawing and (B) the
photomicrograph of the bone changes following labial movement of a
lower incisor for 40 days; tooth sectioned vertically. The text states
It hardly leaves any doubt that while osteoclasts ( ok ) are absorbing
bone on the inner wall, simultaneous formation of new bone is under
way on the opposite side of the process. At ob , near the crest of the
alveolus, compact bone has disappeared and has been replaced by
cancellous bone; the trabeculae of the latter have been arranged
perpendicular to the long axis of the tooth; k 1 , new bone trabeculae
lined
with osteoblasts ( ob ). (From Oppenheim, 1930, International Journal
of Orthodontia, Oral Surgery and Radiography.)

European Journal of Orthodontics 28 (2006) 221240


doi:10.1093/ejo/cjl001
The Author 2006. Published by Oxford University Press on behalf of
the European Orthodontics Society.
The tissue, cellular, and molecular regulation of orthodontic tooth
movement: 100 years after Carl Sandstedt
Murray C. Meikle
Department of Oral Sciences, Faculty of Dentistry, University of Otago,
Dunedin, New Zealand
[Begin manual download] Downloading the PDF version of: Eur
J Orthod Meikle 28 (3): 221. (2302K)

Fig. Schematic drawing of a tooth, the periodontal ligament with cells


and alveolar bone. (a) An external force is applied (arrow). (b) At the
apposition side, fi bres are stretched. Compression of fi bres takes
place at the resorption side. (c) After prolonged force application, bone
formation by osteoblasts can be found at the apposition side and
osteoclasts resorb the bone at the resorption side.
The introduced theoretical model (Figure 1) is a schematic overview of
the processes taking place after the application of an orthodontic force.
The strain in the matrix of the PDL and the alveolar bone immediately
after force application, resulting in a fluid flow in both tissues is
represented by (a). As a result of matrix strain and fluid flow, cells are
deformed (b). In response to the deformation, fibroblasts and
osteoblasts in the PDL as well as osteocytes in the bone are activated
(c).
European Journal of Orthodontics 30 (2008) 299306 The Author
2008. Published by Oxford University Press on behalf of the European
Orthodontic Society.
doi:10.1093/ejo/cjn020
Mechanobiology of tooth movement
S. Henneman , J. W. Von den Hoff and J. C. Maltha
Department of Orthodontics and Oral Biology, Radboud University
Nijmegen Medical Centre, The Netherlands

[Begin manual download] Downloading the PDF version of: Eur


J Orthod Henneman et al. 30 (3): 299. (277K)

Which statement is true about an Angle class I malocclusion?


a. It is characterized by a negative overjet
b. In the class I division 2 the central incisors are retroclined
c. The mesiobuccal of the upper first molar rests in the
mesiobuccal groove of the mandibular first molar
d. There is no malpositioning of the teeth: Angle class I is not a
malocclusion class.

Which statement is true about an Angle class I malocclusion?


a.
b.
c. The mesiobuccal of the upper first molar rests in the mesiobuccal
groove of the mandibular first molar
d.

Edward Angle, who is


considered the father of modern
orthodontics, was the first to
classify malocclusion. He based
his classifications on the relative
position of the maxillary first
molar. According to Angle, the
mesiobuccal cusp of the upper
first molar should rest on the
mesiobuccal groove of the
mandibular first molar. The
teeth should all fit on a line of
occlusion which is a smooth
curve through the central fossas
and cingulums of the upper
canines, and through the buccal
cusp and incisal edges of the
mandible. Any variations from
this resulted in malocclusion

types. It is also possible to have


different classes of maloclusion
on left and right sides.

Class I: the molar relationship of the occlusion is normal or as described


for the maxillary first molar, but the other teeth have problems like
spacing, crowding, over or under eruption, etc.

Class I with severe crowding and labially


erupted canines
Class II: ("overbite") the upper molars are placed not in the mesiobuccal
groove but anteriorly to it. Usually the mesiobuccal cusp rests in between
the first mandibular molars and second premolars. There are two
subtypes:
Class II Division 1: the anterior teeth are protruded.
Class II Division 2: the central incisors are retroclined.

class II molar relationship


Class III: (prognathism, "underbite" or "negative overjet") the lower front
teeth are more prominent than the upper front teeth.

Class III Malocclusion

the edge of the lower incisor is anterior to the


upper incisor

Limitation: The classification assumes that the teeth are correctly


positioned in the dental arches.

References:
Jacobson A. Chapter 1: The significance of radiographic cephalometry. pages
1-16 (pages 2-4). IN: Jacobson A (editor). Radiographic Cephalometry.
From Basics to Videoimaging. Quintessence Publishing Co. Inc.1995.
Mitchell DA, Mitchell A. Oxford Handbook of Clinical Dentistry, Second
Edition. Oxford University Press. 1995.
Yamashita DDR, Urata MM. Chapter 8: Maxillofacial trauma. pages 94-113
(pages 99-100). IN: Demetriades D, Asensio JA. Trauma Management.
Landes Bioscience. 2000.

Other malocclusion classification systems:


Katz: Class I exact fit between mid cusp of most anterior upper premolar and
embrasure created by distal contact of most anterior lower premolar. Use
+/- to signify Class II,III.
Ref.
Angle EH (1899). Classification of malocclusion. Dental Cosmos, 4: 248-264
Proffit et al, Contemporary Orthodontics, 4th ed.,p. 4-5 Copyright 2007 by
Mosby, Inc., ISBN -10: 0_323-04046-2

Which of the following responses is not seen in pulps that are treated with
Formocresol Pulpotomies:
A. Coagulation necrosis
B. Fixation
C. Vital tissue
D. Inflammatory response

Which of the following responses is not seen in pulps that are treated with
Formocresol Pulpotomies:
A. Coagulation necrosis
B. Fixation
C. Vital tissue
D. Inflammatory response
Formocresol use causes three zones in radicular pulp: 1) fixation, 2)
coagulation necrosis, 3) vital tissue
Pulpotomy: coronal extirpation of vital pulp tissue
Formocreosol pulopotomy: in use since 1930s. Long successful track record.
Was (is?) considered the gold standard for pulpectomies.
What is formocresol?
- Formaldehyde, cresol, glycerin, and water
-Buckleys Formula: 1:5 dilution produces equiv results as full strength.
How is it used?
How: After hemorrage control is obtained (assuming appropriate
amount of affected pulp was removed) a Cotton pellet that was soaked in
Formocresol then significantly wrung out, is placed in direct contact with the
remaining pulp for 5mins. Next restore with ZOE in access and SS crown.
Is it still considered safe?
Pashley study- (1980)- Dog study- formocresol detected systemically.
Conclusion: systemic spread is possible.
Ribeiro 2004 JOE Formocresol, paramonochlorophenol and calcium
hydroxide do not promote DNA damage in mammalian cells.
What should I use?
Many authors have argued this point, hence research for other suitable
materials: ferric sulfate & MTA
Fuks (1997) Ped Dent. -Study compared ferric sulfate (Viscostat) vs.
formocresol for use in pulpotomy in primary teeth. Success: 92% ferric
sulfate vs. 84% formocresol. (Not statistically different)

The ideal radicular pulp dressing material should be


1) bactericidal
2) harmless to the pulp and surrounding structures
3) promote healing of the radicular pulp
4) not interfere with the physiologic process of root resorption.

Reference:
Pediatric Dentistry: Infancy through Adolescence (2005), Pinkham, 4th Ed, p.
384-7.

Following premature loss of the Maxillary 2nd primary molar, which of the
following treatment options would you most strongly recommend to the
patient and parent to address an erupting 1st permanent molar?
A. W-holding arch
B. Band and Loop
C. No treatment needed-as there is no loss of space anticipated
D. Distal Shoe

Following premature loss of the Maxillary 2nd primary molar, which of the
following treatment options would you most strongly recommend to the
patient and parent to address an erupting 1st permanent molar?
A.
B.
C.
D. Distal Shoe
Distal Shoe: used when there is premature loss of primary 2nd molar, prior to
eruption of the permanent 1st molar. Care must be used not to overextend
the shoe over the occlusal table of the 1st molar- may prevent eruption. The
distal shoe is typically placed immediately after extraction of the primary 2 nd
molar to prevent mesial movement of the permanent 1st molar.
Band and Loop: used on either the primary 1st molar extending up to the
canine, or band the 1st perm molar and extend the loop up to the 1st primary
molar. This appliance serves eruption of the permanent 1st molar. Care must
be used not to overextend the shoe over the occlusal table of the 1st molarmay prevent eruption. The distal shoe is typically placed immediately after
extraction of the primary 2nd molar to prevent mesial movement of the
permanent 1st molar.
Band and Loop: This appliance serves to maintain the space in situations
where a 1st or 2nd primary molar is lost prematurely. Can be used on either
the primary 1st molar extending up to the canine, or band the 1st perm molar
and extend the loop up to the 1st primary molar.
W-holding arch, Lingual arch, or Nance Holding Arch : Used when bilateral
premature tooth loss occurs.

Lingual Arch. Another variant: W holding arch

Nance Holding Arch

Band and Loop

Distal Shoe
http://www.excelorthodontics.com/fixed.html

Reference:
Textbook: McDonald, Avery and Dean. Dentistry for the Child and
Adolescent 8th ed 2004: Pg 636.

Which of the following ages would be the most ideal to implement nonnutritive sucking cessation treatment options?
A. 1 year old
B. 3 years old
C. 5 years old
D. 7 years old

Which of the following ages would be the most ideal to implement nonnutritive sucking cessation treatment options?
A.
B. 3 years old
C.
D.
Sucking fingers may be a normal part of a childs development. Most children
quit between age 2-4. Minimal effect is reported on the dentition if children
quit before age 3. Position of fingers in the mouth along with intensity,
duration, and frequency of the habit all play a direct role on the effect on the
developing dentition.
AAPD Policy statement on Oral Habits:
Treatment modalities to control habits may include patient/parent
counseling, behavior modification techniques, myofunctional therapy, and
appliance therapy.
Policy Statement
1. The American Academy of Pediatric Dentistry supports the individualized
approach for each child in evaluating oral habits.
2. Where appropriate, the American Academy of Pediatric Dentistry
encourages treatment of oral habits to prevent or intercept possible
malocclusion or skeletal dysplasia from occurring.
http://www.aapd.org/pdf/habits.pdf - xml=http://prdtsearch001.americaneagle.com/service/search.asp?
cmd=pdfhits&DocId=839&Index=F%3a%5cdtSearch
%5caapd.org&HitCount=6&hits=2f+31+33+42+43+61+&hc=193&req=nut
ritive+sucking
Talk with the child. Discuss the problems caused by the habit. Sometimes
this alone is enough to make the child stop sucking.
Use reminder therapy. This approach is appropriate for children who want
to stop sucking but need some help. An adhesive bandage secured with
waterproof tape on the finger or thumb can remind the child not to suck. A
mitten or sock placed on the hand at night can also be effective. Stress to
the child that this is a reminder, not a punishment.
Use a reward system. Under this system, the child, a parent, and the
health professional agree that the child will discontinue the habit within a
specified time period and will then receive a reward. The reward must be
motivating to the child.
Physically interrupt the habit. If none of the preceding methods are
successful, and the child truly wants to stop the habit, two other methods
can be tried: (1) The childs arm can be loosely wrapped in an elastic
bandage during the night to prevent flexing the arm and inserting the thumb
or fingers into the mouth. Stress to the parent that the bandage should not

be wrapped tightly. (2) A dentist can place an intraoral appliance in the


mouth that interferes with sucking.
Reference:
Pinkham JR, Casamassimo PS, Fields HW, McTigue DJ, Nowak AJ, eds. 1999.
Pediatric Dentistry: Infancy Through Adolescence (3rd ed.).
Philadelphia, PA: W. B. Saunders Company.
http://www.excelorthodontics.com/fixed.html

Thumb sucking appliance


a.k.a Crib

Tongue thrusting appliance

You extract nonrestorable tooth #T on a 4 year-old patient. Is space


maintenance indicated and if so, what appliance would you use?
a. Yes, space maintenance is indicated with a lingual arch appliance.
b.
c.
d.
e.

Yes, space maintenance is indicated with a band and loop appliance.


Yes, space maintenance is indicated with a distal shoe appliance.
Yes, space maintenance is indicated with a Porter appliance.
No treatment because unerupted permanent 1st molars do not drift
mesially,
therefore no space will be lost.

You extract nonrestorable tooth #T on a 4 year-old patient. Is space


maintenance indicated and if so, what appliance would you use?
a.
b.
c. Yes, space maintenance is indicated with a distal shoe appliance.
d.
e.
Mesial movement and migration of the first permanent molar will often occur
before eruption in instances of premature loss of the second primary molar.
This is one of the most difficult problems of the developing dentition to
confront the dentist who treats children. A space maintainer that will guide
the first permanent molar into its position is indicated. This is the distal shoe
appliance. A description of this appliance is in the reference text.
A lingual arch is the space maintainer of choice after the multiple loss of
primary teeth in the maxillary or mandibular arch.
A band and loop may be used if the 2nd primary molar is lost after eruption of
the 1st permanent molar.
A Porter appliance, or soldered W lingual arch is an efficient appliance for
the correction of posterior cross-bites.
Ref.
Taken from: McDonald RE, Avery DR. Dentistry for the Child and Adolescent,
6th ed. CV Mosby, St. Louis, MO. (1994): Chapters 26,27.

Which of the following statements is false concerning non-nutritive sucking


habits?
a. Girls are more likely than boys to continue sucking habits after
beginning school
b. As long as the habit stops before the eruption of the permanent incisor,
most of the changes
resolve spontaneously.
c. The effects on the soft and hard tissues associated with a non-nutritive
sucking habit depend
on the frequency and the duration of the
habit.
d. Pacifiers designed to produce a more physiologic sucking pattern have
been shown to be more beneficial when compared to standard
pacifiers or finger sucking.

Which of the following statements is false concerning non-nutritive sucking


habits?
a.
b.
c.
d. Pacifiers designed to produce a more physiologic sucking pattern have
been shown to be more beneficial when compared to standard
pacifiers or finger sucking.

During the primary dentition and early mixed dentition years, many children
engage in digit and pacifier sucking. It is possible to deform the alveolus and
dentition during the primary dentition years with a prolonged, intense habit,
but most of the effect occurs on the eruption of the permanent anterior
teeth. Girls are more likely than boys to continue sucking habits after school
begins. The effect of these habits on the hard and soft tissues depends on
the duration and frequency of the habit. With frequent and prolonged
sucking, maxillary incisors are tipped facially, mandibular incisors are tipped
lingually and eruption of some incisors is impeded. Overjet increases and
overbite decreases. Sometimes posterior crossbites result. There is some
evidence that use of pacifiers increases the incidence of posterior crossbites
more often than digit sucking. Pacifier shapes that are designed to produce
a more physiologic sucking pattern have not been proven to be beneficial
when compared with other pacifiers or to finger sucking. Most children
discontinue pacifier use by age 4 or 5, but digit sucking may continue. The
social pressures of school are usually a strong deterrent. As long as the habit
stops before the eruption of the permanent incisor, most of the changes
resolve spontaneously.
Ref.
Textbook: Contemporary Orthodontics, Fourth Edition. Proffit. 2007. Pgs.
443-5.

In selecting a case for molar uprighting, caution must be exercised when:


a.
b.
c.
d.
e.

The patient presents with a steep mandibular plane angle


The molar is tipped more than 30 degrees
The patient is over 40 years old
The patient has an acceptable occlusal plane
There are spaces between the maxillary anterior teeth

In selecting a case for molar uprighting, caution must be exercised when:


a. The patient presents with a steep mandibular plane angle
b.
c.
d.
e.

A steep mandibular plane angle has a skeletal pattern that will make the
patient naturally want to open. There may already be a minimal anterior
guidance. If you upright the molar you can open the bite even more. The
degree of the tip is not a big concern. The more tipped the, the more of a
challenge it will be to upright the molar, but it can be done. Age is not a
factor. Spacing between maxillary anterior teeth will not affect the
uprighting of the molar. An acceptable occlusion is an indication for molar
uprighting.
Relative contraindications for molar uprighting include:
Severe lingual inclination in addition to mesial tipping
Severe skeletal discrepancies
High mandibular plane angle, antegonial notching
Short or blunted roots
Extruded lower molars
Presence of root resorption
Significant centric relation to centric occlusion discrepancy
Forward bodily movement of molar(s) to close space
The indications are:
Mesially tipped second (and/or third) molar
An acceptable occlusion
Average mandibular plane angle
Skeletal harmony in all three planes
A fully cooperative patient with good oral hygiene
Buccal or lingual rotations can be corrected by bonding a button to the
mesial or lingual cusp of the molar and applying force with a power chain. A
mandibular plane angle of 27o is within the normal range of 22o to 28o.
Intrusion of lower molars is extremely difficult to accomplish and require
gentle force over a prolonged period of time. An extruded mandibular tooth
should be referred to the orthodontist or oral surgeon. In the buccolingual
dimension, the molar that has severe lingual or buccal axial inclination
should be avoided because of the amount of torque that would be necessary

to properly reposition the tooth for restorative dentistry. Mild lingual or


buccal axial inclinations can be managed.
Ref.
Taken from Vanarsdall RL, Swartz ML, Molar Uprighting; Ormco corp., and
Graber TM, Vanarsdall RL (2000), Orthodontics: current principles and
techniques Mosby

Which of the following statements is true concerning Moyer's arch length


analysis?
a. Radiographs of the mandibular teeth only are required answer
b. It predicts space for both the mandible and maxilla answer
c. It uses the sizes of the maxillary incisors to predict maxillary arch
length answer
d. It can be used for a 4 year old patient
e. It predicts the amount of space available for the permanent centrals
and premolars

Which of the following statements is true concerning Moyer's arch length


analysis?
a.
b. It predicts space for both the mandible and maxilla
c.
d.
e.

The analysis advocated by Moyers has a number of advantages. It can be


completed in the mouth as well as on casts and it may be used for both
arches. The analysis is based on a correlation of tooth size; one may
measure a tooth or a group of teeth and predict accurately the size of the
other teeth in the same mouth. The mandibular incisors, since they erupt
early in the mixed dentition and may be measured accurately, have been
chosen for measuring to predict the size of upper as well as the lower
posterior teeth.
The following procedure has been suggested by Moyers to determine the
space available for teeth in the mandibular arch:
1. Measure the greatest mesiodistal width of each of the four
mandibular incisors with the aid of a Boley gauge and record the
value.
2. Determine the amount of space needed for the alignment of the
incisors. This may be accomplished as follows: Set the Boley gauge
to a value equal to the sum of the widths of the right central incisor
and the right lateral incisor. Place one point of the gauge at the
midline between the central incisors and let the other point lie along
the line of the dental arch. Mark on the tooth or the cast the precise
point where the distal tip of the Boley gauge has touched. This
represents the point where the distal surface of the lateral incisor
will be when it has aligned properly. Repeat the procedure for the
opposite arch.
3. Determine the amount of space available for the permanent canines
and premolars incisor alignment. This may be accomplished by
measurement of the distance from the point mark in the line of the
arch to the mesial surface of the first permanent molar. This
distance is the space available for the permanent canine and
premolars as well as for first permanent molar adjustment.
4. Predict the combined width of the mandibular canines and
premolars with the aid of the probability chart. Locate at the top of
the mandibular chart the value over a column of figures that most
nearly corresponds to the sum of the widths the four mandibular
incisors.

5. Compute the amount of space remaining in the arch for first


permanent molar adjustment. The estimated canine and premolar
size value is subtracted for the measured space. It must be
assumed that the first permanent molar will move mesially on each
side unless restrained with a holding appliance.
After all values have been recorded, a complete assessment of the space
situation in both arches is possible.
Ref.
McDonald and Avery, Dentistry for the Child and Adolescent, 1988, Mosby)

A space maintainer that will guide the first permanent molar into its normal
position is a:
a. removable bilateral space maintainer
b. lingual holding arch
c. crown and loop
d. distal shoe
e. Nance appliance

A space maintainer that will guide the first permanent molar into its normal
position is a:
a.
b.
c.
d. distal shoe
e.

Types of Space Maintainers


There are two main types of space maintainers: removable and fixed.
1. Removable removable space maintainers are similar to orthodontic
appliances and are usually made of acrylic. In some cases, an artificial
tooth may be used to fill a space that must remain open for the
unerupted tooth.
2. Fixed there are four different kinds of fixed space maintainers:
unilateral, crown and loop, distal shoe and lingual.
The distal shoe space maintainer is usually used for an unerupted first
permanent molar.
Another space maintainer classification criteria: unilateral and bilateral.
Unilateral Space Maintainer
Crown and Loop
Band and Loop

Mesial movement and migration of the first permanent molar often occurs
before eruption in instances of premature loss of the second primary molar.
This is one of the most difficult problems of the developing dentition to
confront the pediatric dentist. Use of a space maintainer that will guide the

first permanent molar into its normal position is indicated.

Distal shoe

Bilateral Space Maintainer: usually used with more than one tooth missing.
Lingual Holding Arch/LHA
Nance Appliance/TPA

Removable Bilateral Space Maintainer

Treatment considerations
Tooth lost
Time elapsed since tooth loss

Pre-existing occlusion
Favorable space analysis

Cooperation
Oral Habits
Permanent tooth presence
and root development
Space Maintenance Disadvantages
Dislodged, broken, lost
appliance
Plaque
Caries
Interference w/successor
eruption

Amount of alveolar bone


covering successor
Patients health status
Oral hygiene
Over-retain w/damage to
adjacent permanent tooth
Undesirable tooth movement
Inhibition of alveolar growth
Soft tissue impingement

Ref.
Textbook: McDonald, Avery and Dean. Dentistry for the Child and
Adolescent 8th ed 2004: Pg.636.

a.
b.
c.
d.

Which of the following drugs is a potential asthma trigger:


Aspirin
Epinephrine
Prednisone
Tylenol


Which of the following drugs is a potential asthma trigger:
a. Aspirin
b.
c.
d.

Asthma triggers:
Allergens
Upper respiratory tract infection
Exercise
Cold air
Certain medications
Chemicals, smoke

- Stress
Asthma trigger medications include beta blockers, aspirin and other
nonsteroidal anti-inflammatory drugs.
Asthma is a chronic inflammatory respiratory condition characterized
by hyperresponsiveness of the trachea-bronchial tree that results in
narrowing or constriction of the air passages, accompanied by
wheezing, coughing, short breath and reduced air flow in the lungs.
Several categories of asthma have been identified:
- Extrinsic (allergic or atopic)
- Intrinsic (nonallergenic, idiosynchratic)
- Drug induced
- Exercise induced
- Infectious (bacterial, viral, fungal)
While the short-term allergic asthma treatment can be the same as in
general, with bronchodilators and corticosteroids, the allergy induced
asthma should also be treated with appropriate medication such as
antihistaminics and anti-inflammatory drugs.
Aspirin and other non-steroidal anti-inflammatory drugs have been
shown to precipitate or worsen the symptoms of asthma via excessive
leukotriene production and consequently, bronchial constriction. Beta2 receptors on the lung mediate bronchodilation. Antagonizing this
effect worsens bronchoconstriction suffered by asthmatic patients.
Acetaminophen (Tylenol) is safe to use in asthmatic patients.
Severe and unstable asthma require medical consultation; no routine
dental treatment.

Ref.

a.
b.
c.
d.
e.

Effect of atenolol on ventilatory and cardiac function in asthma. J S


Vilsvik and J Schaanning Br Med J. 1976 August 21; 2(6033): 453455.
http://www.asthma.partners.org/newfiles/AspirinSensitivity.html

A fiberotomy is indicated:
At the completion of all orthodontic treatment
At the beginning of orthodontic forced extrusion
As an adjunctive treatment for tooth intrusion
As an adjunctive treatment for molar uprighting
As an adjunctive treatment for correction of tooth rotation


A fiberotomy is indicated:
a.
b.
c.
d.
e. As an adjunctive treatment for correction of tooth rotation

A fiberotomy is a procedure used to sever the periodontal ligament


attachment to the tooth following orthodontic movement. As a tooth
moves through bone, the periodontal ligament fibers are stretched and
can generate a recoiling force if not severed or allowed to remodel in
the new position.

transseptal fiberotomy/supra crestal fiberotomy:


The supraosseous connective tissue attachment is surgically severed
around the involved teeth. Where there are adjacent teeth, the
transseptal fiberotomy of a single tooth will involve a minimum of three
teeth. Since the incisions are within the gingival sulcus and tissue and
the root surface is not instrumented, this procedure heals by the
reunion of connective tissue with the root surface on which viable
periodontal tissue is present (reattachment).

frenulectomy (frenectomy or frenotomy):


The frenum may be excised when the tongue has limited mobility
(ankyloglossia), for large diastemas between teeth, or when the
frenum interferes with a prosthetic appliance; or when it is the etiology
of periodontal tissue disease.

Ref.

Graber, Vanarsdall, Vig Orthodontics: Current Principles and


Techniques Elsevier Mosby 2005, p. 919.
http://www.austinperiodontal.com/procedures-fiberotomy.asp
American Dental Association Current Dental Terminology D7000-D7999
Oral and Maxillofacial Surgery Section 9 / Part 29, April 10, 2006

In order to carry out orthodontic treatment, a provider should use?


a. Continuous NiTi archwire during the whole orthodontic treatment
answer
b. A continuous steel archwire during the whole orthodontic treatment
answer
c. A sequence of archwires from NiTi to TMA to steel answer
d. A sequence of archwire from NiTI to steel to TMA answer


In order to carry out orthodontic treatment, a provider should use?
a.
b.
c. A sequence of archwire from NiTi to TMA to steel TMA
d.

Austenite martensite (TMA)


The exceptional springiness of a NiTi wire makes it a particularly
attractive alternative to steel wires in the initial phases of treatment
when the teeth are severely malaligned. A continuous NiTi archwire of
either type will have better properties than multistrand steel wires and
properties similar to a steel archwire with loops. TMA, as an
intermediate between NiTi and steel, is less useful than either in the
first stage of full-appliance treatment. Its excellent overall properties,
however, make it quite useful in the later stages of treatment. It is
possible, and frequently desirable, to carry out Ortho tx with a series of
wires of approximately the same size, using a sequence form NiTi to
TMA to steel.
The best definition of the optimum force for Orthodontic purposes is
the lightest force that produces a maximum of near-maximum
response (i.e., that brings pressure in the PDL to the edge of the
nearly-constant portion of the response curve). The magnitude of this
optimum force will vary depending on the way it is distributed in the
PDL (i.e., is different for different types of tooth movement, like tipping,
bodily movement, intrusion, etc).

Ref.
Proffit, William et al. Contemporary Orthodontics. 2nd. Ed, 1986. Mosby.
Pp 301-307.

When examining a 10 year old child, which of the following teeth would
you expect to have root formation completed?
a. Maxillary centrals, mandibular centrals and laterals
b. Maxillary centrals and laterals, mandibular centrals and laterals
c. Mandibular centrals and laterals
d. Maxillary centrals, mandibular centrals and laterals, all first molars


When examining a 10 year old child, which of the following teeth would
you expect to have root formation completed?
a.
b.
c.
d. Maxillary centrals, mandibular centrals and laterals, all first molars

Chronology of the human dentition

Amount of enamel

at birth

Decidu
ous
dentiti
on

Enamel

Completed
C
en
tr
al
Ma
in
xill
ci
ary
so
r

Ma
ndi
bul
ar

Ma

La
te
ral
in
ci
so
r

C
us
pi
d

Hardtissue
Root
Tooth
Formation
Formed
Eruption
Completed
4
Five
1
7
1
m
sixths

years
o
mo
m
in
nth
o
u
s
nt
t
hs
e
r
o
4
Two
2
9
2

thirds

m years
mo
o
m
nth
nt
o
s
hs
in
u
t
e
r
o
5
One
9
1
3
m
third
mo
8 years
o
nth
m
in
s
o
u
nt
t
hs
e
r
o

Perman
ent
dentiti
on

xill
ary

Ma
ndi
bul
ar

Fir
st
m
ol
ar

Se
co
nd
m
ol
ar

C
en
tr
al
in
ci
so
r

La
te
ral
in
ci
so
r

C
us
pi
d

5
m
o
in
u
t
e
r
o
6
m
o
in
u
t
e
r
o

m
o
in
u
t
e
r
o
4

m
o
in
u
t
e
r
o
5
in
u
t

Cusps
unite
d

6
mo
nth
s

1
2
4 years
m
o
nt
hs

Cusp
tips
still
isolat
ed

11
mo
nth
s

2
3
4 years
m
o
nt
hs

Three
fifths

mo
nth
s

6
1
m years
o
nt
hs

Three
fifths

3
mo
nth
s

7
1
m years
o
nt
hs

One
third

9
mo
nth
s

1
3
6 years
m
o

Fir
st
m
ol
ar
Se
co
nd
m
ol
ar

C
en
tr
al
in
ci
so
r
La
te
ral
in
ci
so
r

C
us
pi
d

e
r
o
5
in
u
t
e
r
o
6
in
u
t
e
r
o

4
m
o
n
t
h
s
1
0

1
2
m
o
n
t
h
s
4

5
m
o
n

nt
hs
Cusps
unite
d

mo
nth
s

1
2
2 years
m
o
nt
hs

Cusp
tips
still
isolat
ed

10
mo
nth
s

2
3
0 years
m
o
nt
hs

4
5
ye
ars

7
1
years
8
ye
ar
s

4
5
ye
ars

8
1
years
9
ye
ar
s

6
7
ye
ars

1
1
1 -15 yea

1
2
ye

Fir
st
bi
cu
sp
id

Se
co
nd
bi
cu
sp
id

t
h
s
1

3
/
3
/
4
y
r
2

ar
s

5
6
ye
ars

1
1
0 13 yea

1
1
ye
ar
s

6
-7
ye
ars

1
0

1
2
ye
ar
s
6

7
ye
ar
s
1
2
1
3
ye
ar
s
1
7

2
1
ye
ar
s

y
r

Fir
st
m
ol
ar

A
t
bi
rt
h

Some
times
a
trace

2
3
ye
ars

Se
co
nd
m
ol
ar

3
y
r

7
8
ye
ars

Th
ird
m
ol
ar

7
9
y
r

12

16
ye
ars

C
en

4
5

1
-14 yea

9
10 year

1
16 yea

1
25 yea

9
years

tr
al
in
ci
so
r
La
te
ral
in
ci
so
r
C
us
pi
d

Fir
st
bi
cu
sp
id
Se
co
nd
bi
cu
sp
id
Fir
st
m
ol

4
m
o
n
t
h
s
3

4
m
o
n
t
h
s
4

5
m
o
n
t
h
s
1

y
r
2

y
r
A
t
bi
rt

ye
ars

7
ye
ar
s

4
5
ye
ars

7
1
years
8
ye
ar
s

6
-7
ye
ars

9
1
14 yea
1
0
ye
ar
s

5
-6
ye
ars

6
7
ye
ars

Some
times
a
trace

2
3
ye

1
0

1
2
ye
ar
s
1
1

1
2
ye
ar
s
6

7
ye

1
13 yea

1
14 yea

9
10 year

ar

ars

ar
s
Se
2

7
1
co

8
1
nd
ye

m
3
ars
1
ol
y
3
ar
r
ye
ar
s
Th
8

12
1
ird

7
m
1
16

ol
0
ye
2
ar
y
ars
1
r
ye
ar
s
From Logan, W.H.G., and Kronfeld, R.: J.A.D.A. 20:379, 1933; modified
by McCall and Schour.

Ref.
Cassamassimo et al: Pediatric Dentistry Infancy through adolescence.
2nd ed. Pinkham.

1
15 yea

1
25 yea


Early child caries (ECC) has a multifactorial etiology but follows a
definite pattern. Which of the following statements is true concerning
ECC?
a. There is early carious involvement of the mandibular first primary
molars (first)answer
b. The mandibular canines and incisors are usually unaffected answer

c. The maxillary incisors are characteristically affected early answer


d. The lesion starts as a yellow band on the smooth surface of the tooth
followed by a brown discoloration


Early child caries (ECC) has a multifactorial etiology but follows a
definite pattern. Which of the following statements is true concerning
ECC?
a.
b.
c. The maxillary incisors are characteristically affected early answer
d.

The clinical appearance of the teeth is typical and follows a definite


pattern. This specific pattern of tooth decay was called Baby Bottle
Tooth Decay (BBTD). BBTD was believed to be
primarily associated with the use of a sleep-time bottle that contains a
liquid with natural or added sugars such as formula, juice or Kool-Aid. It
generally occurs between the ages of 12-18 months. In recent years,
similar cases of early and severe tooth decay have been found in
children who do not fit the classic BBTD pattern of bottle use. In light of
this, the term Early Childhood Caries (ECC) is now being used to reflect
a broader concept of the problem of tooth decay in infants and young
children. ECC includes decay associated with any causative
factors, including continuous use of a sippy-cup, at-will breast-feeding
throughout the night, use of a sweetened pacifier or where chronic
illness requires regular use of a sugar-based oral medication.

ECC is an infectious disease and is defined as the presence of 1 or


more decayed (cavitated lesions0, missing (due to caries), or filled
tooth surfaces in the primary tooth in a child 71 months of or younger.
In children younger than 3 years of age, any sign of smooth-surface
caries is indicative of severe early childhood caries (S-ECC). From ages
3 through 5, 1 or more cavitated, missing (due to caries), or flled
smooth surfaces in primary maxillary anterior teeth or decayed,
missing, or filled score of > 4 (age 3), > 5 (age4), or >6 (age 5)
surfaces constitute S-ECC.

The AAPD recognizes caries as a common, complex, chronic disease


resulting from an imbalance of multiple risk factors and protective
factors over time. The following are characteristics of ECC, formerly
known as Baby Bottle Tooth Decay:

The onset of ECC can be 12 to 18 months or earlier!


It develops rapidly. Progression from the enamel (the thin, hard,

little

and

outer layer of the tooth) into the dentin (the softer, inner layer of
the tooth) occurs in six months or less. Once the decay is in the
dentin, because it is so soft, the tooth can be destroyed in as

as six weeks!
The upper front teeth are affected first. These teeth usually erupt

at around eight months of age.


Primary molars, which begin to erupt at about 12 months of age,

are the next teeth to be affected.


Finally, the lower front teeth are affected when the disease

becomes very severe.

Currently, ECC is recognized as an infectious disease. Its extent

influences.

decay

high as

severity vary with cultural, genetic and socio-economic


According to the American Association of Pediatric Dentists, the
level in three to five year old U.S. Head Start children may be as
90 percent in some populations.

Ref.
American Academy of Pediatric Dentistry Reference manual, Vol 30/ No
7 08/09
And
health.nv.gov/PDFs/OH/ECClongversiontext.pdf.


Which of the following ages would be the most ideal to implement nonnutritive sucking cessation treatment options?

A. 1 year old
B. 3 years old
C. 5 years old
D. 7 years old

Which of the following treatment modalities would not be employed in


the management of a 5 year old who exhibits a non-nutritive sucking
habit.

A. Discuss the habit and cessation with the child


B. Fabrication of an appliance
C. Physically disrupt the habit
D. Use negative feedback reminder therapy


Which of the following ages would be the most ideal to implement nonnutritive sucking cessation treatment options?

A.
B. 3 years old
C.
D.

Which of the following treatment modalities would NOT be employed in


the management of a 5 year old who exhibits a non-nutritive sucking
habit.

A.
B.
C.
D. Use negative feedback reminder therapy

Sucking fingers may be a normal part of a childs development. Most


children quit between age 2-4. Minimal effect on the dentition if
children quit before age 3. Position of fingers in the mouth along with
intensity, duration, and frequency of the habit all play a direct role on
the effect on the developing dentition.

AAPD Policy statement on Oral Habits:


Treatment modalities to control habits may include patient/parent
counseling, behavior modification techniques, myofunctional therapy,
and appliance therapy.
Policy Statement
1. The American Academy of Pediatric Dentistry supports the
individualized approach for each child in evaluating oral habits.
2. Where appropriate, the American Academy of Pediatric Dentistry
encourages treatment of oral habits to prevent or intercept possible
malocclusion or skeletal dysplasia from occurring.
http://www.aapd.org/pdf/habits.pdf - xml=http://prdtsearch001.americaneagle.com/service/search.asp?
cmd=pdfhits&DocId=839&Index=F%3a%5cdtSearch
%5caapd.org&HitCount=6&hits=2f+31+33+42+43+61+&hc=193&re

q=nutritive+sucking
Talk with the child. Discuss the problems caused by the habit.
Sometimes this alone is enough to make the child stop sucking.
Use reminder therapy. This approach is appropriate for children who
want to stop sucking but need some help. An adhesive bandage
secured with waterproof tape on the finger or thumb can remind the
child not to suck. A mitten or sock placed on the hand at night can also
be effective. Stress to the child that this is a reminder, not a
punishment.
Use a reward system. Under this system, the child, a parent, and the
health professional agree that the child will discontinue the habit within
a specified time period and will then receive a reward. The reward
must be motivating to the child.
Physically interrupt the habit. If none of the preceding methods are
successful, and the child truly wants to stop the habit, two other
methods can be tried: (1) The childs arm can be loosely wrapped in an
elastic bandage during the night to prevent flexing the arm and
inserting the thumb or fingers into the mouth. Stress to the parent that
the bandage should not be wrapped tightly. (2) A dentist can place an
intraoral appliance in the mouth that interferes with sucking.

Ref.:
Pinkham JR, Casamassimo PS, Fields HW, McTigue DJ, Nowak AJ, eds.
1999. Pediatric Dentistry: Infancy Through Adolescence (3rd
ed.). Philadelphia, PA: W. B. Saunders Company.


Upon placement of orthodontic separators, when do you anticipate
being able to seat bands on the teeth.

A. 4-5 days
B. 7-10 days
C. 2 weeks
D. any time after 2 weeks


Upon placement of orthodontic separators, when do you anticipate
being able to seat bands on the teeth.

A. 4-5 days
B. 7-10 days
C. 2 weeks
D. Any time after 2 weeks

Elastic separators act to slowly separate the teeth apart over a period
of several days, as the elastic contracts towards its original shape.
Elastics wear out with time and should not be left in place longer than
2 weeks.
Brightly colored spacers make them easily visible in case of
displacement interproximally.
Other separators such as brass wires and separating springs, require
about a week prior to seating a band.

Proffit, Contemporary Orthodontics, 3rd ed., p. 392.


Which of the following Angle Classifications might be expected to result
from a Mesial Step?

A. Class 1 or End-to-end
B. Class 1 or Class 3
C. Class 1 or Class 2
D. Class 1 only
E. Class 3 only


Which of the following Angle Classifications might be expected to result
from a Mesial Step?

A.
B. Class 1 or Class 3
C.
D.
E.

In consideration of the primary dentition. Assesment of the terminal


plane of primary mandibular to the maxillary second molar.

3 relationships: mesial step, distal step, and flush terminal plane.


(These were described by Baume (1950) and may be considered as Baumes
Classification)

Mesial step = mandibular molar is mesial to the maxillary molar

Distal step = mandibular molar is distal to the maxillary molar

Flush terminal plane is the normal relationship.

The developmental paths are as follows:

Distal step
- Class II tendency
Flush terminal plane - Class I tendency
Mesial step
- Class I or Class III

Proffit, Contemporary Orthodontics, 3rd ed., p. 90, 217.


http://jdr.sagepub.com/cgi/reprint/29/3/331 (Baumes 1950 article with diagrams!)


By what age should growth modification appliance therapy be
completed (e.g. Frankel Appliance)?

A. 7-8 years old


B. 11-12 years old
C. 12-14 years old
D. 16-18 years old


By what age should growth modification appliance therapy be
completed (e.g. Frankel Appliance)?

A.
B. 11-12 years old
C.
D.

The Frankel appliance, by Dr. Rolf Frankel of Germany


It holds away the tissue of the lips and chin allowing the skeletal
system to develop in its most natural way.
Frankel appliance used to:

-promote change in sagittal, transverse, & vertical jaw relationships

-remove growth restrictive muscle forces in labial and buccal areas

-provide environment to maximize skeletal growth.


It works within the vestibule of the mouth.
It holds away the tissue of the lips and chin allowing the skeletal
system to develop in its most natural way.

Growth modification appliances should not be used after age 11 as


there is insufficient growth occurring to modify. (Use before age 11)

Before
After

Four types of appliances by Frankel: FR I, II, III and IV.


Frankel I - for Class I and II cases with minimal crowding
(Positions Mandible more forward. Muscle pull on the Maxilla restricts
growth)
Frankel II - for Class II division 1 and 1 cases
(Postures Mandible forward and creates tension in joint space- leads to
mandibular growth, also has screw for further advancement)
Frankel III - for skeletal Class III cases
Frankel IV - Uses muscles and headgear to move the mandible forward,
retard maxilla growth, and flatten open bite tendencies

Exacting fabrication technique!

Proffit, Contemporary Orthodontics, 3rd ed.,

PERIODONTICS

Success of a free gingival graft depends on survival of the connective tissue.


Revascularization of the graft starts by the second or third day, when capillaries
from the _______ proliferate into the graft to form a network of new capillaries and
anastomoses with preexisting vessels.

a. PDL
b. recipient bed
c. epithelial downgrowth
d. graft
e. subperiosteal plexus

Success of a free gingival graft depends on survival of the connective tissue.


Revascularization of the graft starts by the second or third day, when capillaries
from the _______ proliferate into the graft to form a network of new capillaries and
anastomoses with preexisting vessels.

a.
b. recipient bed
c.
d.
e.

Success of the graft depends on survival of the connective tissue. Epithelial


sloughing occurs in most cases, but the fate of the graft depends on the
connective tissue that withstands the transfer. Fibrous organization of the
interface between the graft and the recipient bed occurs within 2 to several days.

The graft is initially maintained by diffusion of fluid from the host bed,
adjacent gingiva, and alveolar mucosa. During the first day the CT becomes
edematous and disorganized and undergoes degeneration and lysis of some of its
elements. The degenerated connective tissue is replaced by new granulation
tissue as healing progresses.

Revascularization of the graft starts by the second or third day. Capillaries


from the recipient bed proliferate into the graft to form a network of new capillaries
and anastomoses with preexisting vessels. Epithelium undergoes degeneration
and sloughing with complete necrosis in some areas, and is replaced by new
epithelium from the borders of the recipient site. Rete Pegs develop by the
seventh day.

Healing of a graft is complete by 10.5 weeks; thicker grafts (1.75mm) require


16 weeks.

Beagle studies of healing (GTR):

At 7 days, a clot was present at the demarcation zones and it was more
organized at 14 days.

At 28 days, the junctional epithelium was formed and the demarcation zones
could not be delineated.

At 60 days, the oral epithelium had regained its normal appearance.

The attachment of the graft to the root surface was mediated by a


combination of epithelial downgrowth and connective tissue attachment.

Minimal new bone and cementum formation was observed.

Healing Rates:
- Curettage: Epithelium 3-7 days / CT - 7 to 15 days.
- Gingivectomy: Epithelium 7-14 days / CT 14 to 35 days.
- Gingival flaps: Epithelium 7 days / CT 14 days.
- Pedicle flap: Epithelium 10-14 days / CT 21 days.
- FGG: Epithelium 8-14 days / CT 17 - 30 days.

Four stages of wound healing: inflammation, epithelialization, fibroplasia, and


remodeling.
- Inflammation - week 1: WBCs remove bacteria and break down debris
-

Fibroplasia occurs along with Epithelialization week 1 to 3:


Fibroplasia consists of the ingrowth of fibroblasts and capillaries. Osteoclasts
accumulate along the lamina dura. Granulation tissue develops and an osteoid
material is laid down starting at the socket wall.
Epithelialization consists of the migration of gingival/mucosa from the
boarders of the wound into the socket and over the granulation tissue. The
duration is dependent on how large the secondary healing site is, but usually
complete be week 3 or 4.
- Remodeling starts from week 5 to 6 months. Osteoclasts resorb the cortical
bone lining the socket, and together with osteoblasts reorganize the haphazard
pattern laid down earlier into regular bone with the reestablishment Haversian
canal systems. As bone fills the socket, the epithelial covering is raised to a
level that roughly equates to the surrounding tissues.

Ref.:

Newman, Takei, Klokkevold, Carranza. Carranzas Clinical Periodontology.


th
10 Edition. Suanders, 2006.

Beagle studies. J Periodontol 2001;72:470-478

Myron Nevins, James Mellonig, Periodontal Therapy: Clinical Approaches and


Evidence of Success. Quintessence Books. Page 242

Which of the following is an advantage of systemic antibiotic use in


controlling periodontal disease?

a. Improved patient compliance with therapy.

b. Lower cost of treatment.

c. Serves as an adjunct for treating aggressive and refractory forms of


periodontal

disease.

d. A single antibiotic properly prescribed can inhibit the growth of all


microorganisms

that cause periodontal disease.

Which of the following is an advantage of systemic antibiotic use in


controlling periodontal disease?

a.

b.

c. Serves as an adjunct for treating aggressive and refractory forms of


periodontal

disease.

d.

Antibiotic therapy is beneficial in patients who do not respond to mechanical


therapy alone.

This treatment is an adjunct, not a sole means of treatment.

SYSTEMIC ANTIBIOTICS

The main reason for using systemic antibiotics for treating periodontal
disease is to control a periodontal infection that is failing to respond to mechanical
methods of treatment and local delivery of chemotherapeutics. Systemic
antibiotics serve as an adjunct to treatment and should not be used alone for
treatment. An ideal antibiotic for use in the treatment of periodontal disease
should be specific for periodontal pathogens, be non-toxic, be substantive, not be
used for treatment of other diseases and be inexpensive. This ideal antibiotic does
not currently exist. No single antibiotic at concentrations achieved in body fluids
inhibits all known periodontal pathogens. Combination therapy with more than
one antibiotic may come close to inhibiting all known pathogens in certain perio
pockets. Clinical diagnosis and situation determine the need for antibiotic therapy
as an adjunct in controlling active periodontal disease. Unresolving disease
activity such as, continuing attachment loss, purulent exudate and continuing
periodontal pockets of 5mm or greater and bleed upon probing are indications for
periodontal intervention and possible antimicrobial therapy. Also, cases of
aggressive and refractory periodontitis may indicate the use of antimicrobial
therapy. Antibiotics have been shown to reduce the need for periodontal surgery
in patients with chronic periodontitis.

Some of the disadvantages of using systemic antibiotics are:


- Allergic/anaphylactic reactions
- Super-infections of opportunistic bacteria
- Development of drug resistant bacteria
- Interactions with other medications
- Upset stomach, nausea and vomiting

Examples: Doxycycline

The subantimicrobial dose doxycycline (SDD) regimen is 20mg bid for 6-9
months. The brand name for this is Periostat. This decreases the activity of

microbial matrix metalloproteinases that are active in periodontal tissue


destruction. As a comparison, the average daily antimicrobial dose of doxycyline is
200mg. An advantage of this regimen is that this does not seem to contribute
toward antimicrobial resistant organisms.

LOCAL ANTIBIOTICS

Local administration of anti-infective agents, generally directly in the pocket


has the potential to provide greater concentrations directly to the infected area
and reduce possible systemic side effects. Localized concentration at effective
levels for sufficient time may be possible.

Local Antibiotics:
- Tetracycline Containing Fibers
- Subgingibal Doxycycline
- Subgingival Minocycline (ARESTIN 1mg minocycline microspheres)
- Subgingival Metronidazole
- PerioChip: chlorhexidine gluconate

Ref.:

Journal of Clinical Pharmacy and Therapeutics volume 17 issue s1,


19may2008 Antibiotics in Periodontal Therapy:Advantages and Disadvantages

Newman, Takei, Klokkevold, Carranza. Carranzas Clinical Periodontology.


th
10 Edition. Suanders, 2006.

Systemic antibiotics in Periodontics, AAP position paper. J Perio November


2004; 75.

Carranza's Clinical Periodontology, 10th Ed, 2006.

Which periodontal fiber(s) are most resistant to periodontitis?


a.
b.
c.
d.
e.
f.

Alveolar crest group


Horizontal fiber group
Oblique fiber group
Transseptal fiber group
Apical group
Gingival fibers

Which periodontal fiber(s) are most resistant to periodontitis?


a.
b.
c.
d. Transseptal fiber group
e.
f.

Transseptal Fiber Group: These organized gingival fiber groups enable the
gingival to form a rigid cuff around the tooth that can add stability, especially when
a significant portion of the PDL and alveolar support is lost.

Ref.

Rose L, Mealey B, Genco R, Cohen D. Periodontics Medicine, Surgery and


Implants. Mosby 2004.

Which of the following is statement is correct regarding CO2 Lasers


a. The CO2 lasers wavelength is unique because it is absorbed readily by tissue
pigments.
b. The CO2 laser is the only FDA approved laser for hard-tissue uses.
c. FDA approved wavelength for CO2 lasers is 1064 nm.
d. CO2 laser is unique among soft tissue lasers as its wavelength is absorbed by
water.

Which of the following is statement is correct regarding CO2 Lasers


a.
b.
c.
d. CO2 laser is unique among soft tissue lasers as its wavelength is absorbed by
water.

The carbon dioxide (CO2) laser is unique among soft tissue lasers as its
wavelength is absorbed

by water rather than tissue pigments. Because mucosa is 7590% water, it


absorbs the CO2 wavelength easily. A recent position paper from the American
Academy of Periodontology

describes the advantages of using the CO2 laser for periodontic procedures;
these advantages include hemostasis and a relatively dry surgical field. The CO2
laser also may be utilized to reduce periodontal probing depths by using an
epithelial exclusion technique. Peer-reviewed literature details how the CO2 lasers
energy can enhance guided tissue regeneration techniques and enhance reduction
in pocket depths; these studies provide histological proof of enhanced osseous
regeneration in monkeys, beagles, and humans. Though its wavelength (10,600
nm) can be absorbed by hydroxyapetite, the CO2 primarily is a soft-tissue laser.
CO2 is not FDA-approved for hard-tissue uses at this time.

Current laser manufacturers and their FDA-approved wavelengths

Manufacturer
Wavelength in
Absorption

nanometers (nm)

American Dental Technologies


Nd:YAG 1064
Pigment

American Dental Technologies


Diode 810830
Pigment

Biolitec
Diode 980
Pigment

Biolase
Er,Cr:YSGG 2790
Water

Biolase
Diode 810830
Pigment

Hoya Conbio
Er: YAG 2940
Water

Hoya Conbio
Diode 810830
Pigment

Lares Research
Nd:YAG 1064
Pigment

Millenium Dental Technologies


Nd:YAG 1064
Pigment

OpusDent
CO2 10,600
Water


OpusDent
Diode 810830
Pigment

OpusDent
Er:YAG 810830
Water

Zap Lasers
Diode 810830
Pigment

FDA-approved soft-tissue procedures.*

Abscess incision and drainage

Aphthous ulcer treatment

Biopsies (incisional/excisional)

Crown lengthening (soft tissue only)

Hemostatic assistance

Fibroma removal

Frenectomy

Frenotomy

Gingival excision/incision

Gingivectomy/gingivoplasty

Operculectomy

Oral papillectomy

Tissue retraction for impression

Vestibuloplasty

Exposure of nonerupted or partially erupted teeth

Implant recovery

Lesion (tumor) removal

Leukoplakia

Pulpotomy

Pulpotomy as an adjunct to root canal therapy

Removal of filling material, such as gutta-percha or resin, as an adjunct


treatment during root canal retreatment

Sulcular debridement (removal of diseased or inflamed soft tissue in the


periodontal pocket) to improve clinical indices, including gingival index, gingival
bleeding index, probe depth, attachment level, and tooth mobility

*Not all lasers have FDA approval for every procedure listed above. Please
check with each manufacturer for a complete current listing.

FDA-approved hard-tissue procedures.*

Removal of caries

Cavity preparation

Etching of enamel

Enameloplasty, excavation of pits and fissures for placement of sealants

Cutting, shaving, contouring, and resection of oral osseous tissue

Apicoectomy


Endodontics

*Not all lasers have FDA approval for every procedure listed above. For a
complete current listing, please check

Ref.

An overview of lasers in dentistry Robert A. Convissar, DDS, FAGD | Ellen E.


Goldstein, DDS

http://www.dyadent.com.ec/downloads/biolasedownloads/Lasers%20en
%20Odontologia.pdf

Effects of smoking on the response to periodontal therapy include all of the


following except?
a. Decrease reduction in pocket depth answer
b. Decrease in clinical attachment level answer
c. Increase in gingival inflammation and bleeding on probing answer
d. Increase in post-surgical healing time answer
e. Increase pocket depth during maintenance answer

Effects of smoking on the response to periodontal therapy include all of the


following except?
a.
b.
c. Increase in gingival inflammation and bleeding on probing answer
d.
e.

In the case of gingivitis, gingival inflammation is decreased and bleeding on


probing is decreased in the smoker.

Nicotine and its major metabolite, cotinine, are deposited on root surfaces
resulting in discoloration. Diffuse grayish discoloration and leukoplakia of the
gingiva may occur.

Smokers palate (nicotine stomatitis), characterized by prominent mucous


glands with inflammation of the orifices and a diffuse erythema or by a wrinkled,
cobblestone surface, may occur. Post-surgical healing is delayed. There is a
correlation between smoking and ANUG, but a direct cause-and-effect has not been
shown. Smoking induces an immediate transient, but marked increase in gingival
fluid flow, probably as a result of blood flow changes induced by nicotine. More
severe gingivitis and periodontitis have been reported in smokers, probably due to
increased plaque accumulation.

The following laboratory findings may point to changes responsible for


increased periodontal problems in smokers: Keratinized cells in the gingiva are
increased in smokers, but no changes other than altered oxygen consumption can
be detected in the buccal mucosa. Nicotine metabolites have been found in saliva
and crevicular fluid. Oral polymorphonuclear leukocytes from smokers show a
reduced ability to phagocytize particles. Vascular reaction associated with plaqueinduced gingivitis is suppressed in smokers.

Ref.

Carranza, FA Jr, Newman MG (1996). Clinical Periodontology 8th ed. WB


Saunders Co. Philadelphia, PA, p.169.

Restorative margin placed no greater than _____ into the gingival sulcus
allows for the maintenance of the biologic width.
a. 0.2 mm
b. 0.3 mm
c. 0.4 mm
d. 0.5 mm

Restorative margin placed no greater than _____ into the gingival sulcus
allows for the maintenance of the biologic width.
a.
b.
c.
d. 0.5 mm

The biologic width is defined as the physiologic dimension of the junctional


epithelium and connective tissue attachment. This measurement has been found
to be relatively constant at approximately 2 mm (+ 30%). The healthy gingival
sulcus has shown an average depth of 0.69 mm. It has been theorized that
infringement on the biologic width by the placement of a restoration within its zone
may result in gingival inflammation, pocket formation, and alveolar bone loss.
Consequently, it is recommended that there be at least 3.0 mm between the
gingival margin and bone crest. This allows for adequate biologic width when the
restoration is placed 0.5mm within the gingival sulcus.

When the restoration margin is placed 2 mm or less away from the alveolar
bone and the gingival tissues are inflamed with no other etiologic factors evident,
it is called biologic width violation.

Ref.

Carranzas Clinical Periodontology, Tenth Edition, 2006, pages: 1044-5.

Gargiulo AW: Dimensions and relations of the dentogingival junction in


humans, J Periodontol 32:261, 1961

Newman et al, Carranzas Clinical Periodontology, 10th ed., Saunders, 2006

Which is not a goal of scaling and root planing:


a. Reducing vertical bone defects
b. Eliminating inflammation
c. Reducing probing depths
d. Improving clinical attachment

Which is not a goal of scaling and root planing:


a. Reducing vertical bone defects
b.
c.
d.

Scaling and root planing together with plaque control constitute the major
means by which periodontal disease progression is prevented.

Subgingival scaling and root planing are measures which can be effective in:
Eliminating inflammation
Reducing probing depths
Improving clinical attachment

The objective of Scaling is to remove plaque and calculus and the associated
pathogenic bacteria from the tooth surfaces supragingivally and subgingivally.

The objective of root planing is to remove residual embedded calculus and


portions of cementum and a smooth and hard surface is left. When this is
accomplished, there is a shift from a high number of gram negative anaerobes to
one dominated by gram positive facultative bacteria that are compatible with
health.

Supragingival

The goal is to obtain a tooth surface that does not encourage the
accumulation of deposits and can be maintained by the patient. Scaling and
polishing are procedures indicated to achieve a clean, smooth supragingival tooth
surface.

Subgingival

The objective of root preparation is to clean and detoxify the root surface to
Minimize the toxic root contribution as an ongoing insult to the adjacent
periodontal tissues
Obtain a biologically acceptable root surface for tissue adaptation and
potential new attachment

Rationale for scaling and root planing:


Removal of calculus and "infected" root structure
Achievement of a smooth root surface which is less prone to plaque
accumulation

Garret in 1977 set forth the rationale for root planing as:
Root Smoothness

Removal of Diseased Cementum


Preparation for New Attachment

Objectives of root planing:


Securing biologically acceptable root surfaces
Resolving inflammation
Decreasing pocket depth
Facilitating oral hygiene procedures
Improving or maintaining attachment level
Preparing the tissues for surgical procedures

Ref.:

JADA, 2003, Vol 134, No 2, 259: Treating Periodontal Disease: Scaling and
Root Planing.

Newman, MG, Takei, HH, Carranza, FA; Clinical Periodontology: Ch. 47


Scaling and Root Planing; WB Saunders Pub.; Philadelphia, PA; 2002.

Fedi P. Jr., Vernino A.: The Periodontic Syllabus, 3rd Ed., Williams and Wilkins
Pub.,1995.

Which of the following is no longer considered a periodontal disease class in


the new American Academy of Periodontology adopted system for the classification
of periodontal disease?
a. Chronic periodontitis
b. Aggressive periodontitis
c. Necrotizing ulcerative periodontitis
d. Refractory periodontitis

Which of the following is no longer considered a periodontal disease class in


the new American Academy of Periodontology adopted system for the classification
of periodontal disease?
a.
b.
c.
d. Refractory periodontitis

The diagnosis of periodontal disease is a process that involves observation


and measurement of numerous factors and then combining one's findings into a
summary statement of the patient's oral disease status.

The previous (1989) classification forms of periodontal disease as listed in


Carranza's Clinical Periodontology are:

1. ADULT

2. EARLY-ONSET
Prepubertal (localized and generalized)
Juvenile (localized and generalized)
Rapidly Progressive

3. PERIODONTITIS ASSOCIATED WITH SYSTEMIC DISEASE

4. NECROTIZING ULCERATIVE

5. REFRACTORY

The American Academy of Periodontology adopted a new system for


the classification of periodontal disease; it attempts to overcome some
of the shortcomings of the old classification and establish a more
clinically relevant and specific classification.

The new system contains a section devoted to GINGIVAL disease.


Gingival diseases are divided into those lesions that are either plaqueinduced or those that are not primarily associated with dental plaque.


The gingival lesion classifications are applied to a periodontium with no
attachment loss or on a periodontium with attachment loss that is not progressing.

Characteristics common to all gingival diseases include reversibility of the


disease by removing the etiology and precursor to attachment loss around teeth.

The description of the PERIODONTAL diseases has been similarly revised. A


major change in the new system eliminates the age-related component for
diagnosis, since it has been shown that forms of periodontitis most commonly
found in adults can also be found in adolescents.


includes a possible problems such as tooth anatomy, lack of
keratinized tissue, and occlusal trauma

The category for Refractory periodontitis was eliminated. However it was


recognized that a small percentage of all types of cases do not respond to
treatment. Therefore, rather than have "refractory" as a separate entity, the term
is now applied in conjunction with the new system. For example, refractory chronic
periodontitis, refractory aggressive periodontitis, etc.

Ref.:

Armitage, G.: Development of a Classification System for Periodontal


diseases and Conditions. Ann Periodontal 4: 1-6, 1999

Carranzas Clinical Periodontology, 9th ed.

Pathogenesis of Inflammatory Periodontal Disease, Page Roy C, et al.:


Laboratory Investigation, vol.34 no.3, 1976

Which of the followings is non-absorbable suture?


a.
b.
c.
d.

ePTFE
Plain gut
Monocryl
Vicryl

Which of the followings is non-absorbable suture?


a. ePTFE
b.
c.
d.

Sutures for Periodontal Flaps


Nonabsorbable (Nonresorbable)
Silk: braided

Nylon: monofilament (Ethilon)

ePTFE: monofilament (Gore-Tex)

Polyester: braided (Ethibond)

Absorbable (Resorbable)
Surgical: gut

Plain gut: monofilament (30 days)

Chromic gut: monofilament (45-60 days)

Synthetic
Polyglycolic: braided (16-20 days)

Vicryl (Ethicon)

Dexon (Davis & Geck)

Polyglecaprone: monofilament (90-120 days)


o

Monocryl (Ethicon)

Polyglyconate: monofilament (Maxon)


ABSORBABLE SUTURES are made of materials which are broken down in
tissue after a given period of time, which depending on the material can be from
ten days to eight weeks.

Absorbable sutures were originally made of the intestines of sheep, the so


called catgut. The manufacturing process was similar to that of natural musical
strings for violins and guitar, and also of natural strings for tennis racquets.

Today, gut sutures are made of specially prepared beef and sheep intestine,
and may be
-

untreated (plain gut),

tanned with chromium salts to increase their persistence in the body


(chromic gut)

heat-treated to give more rapid absorption (fast gut).

However, the majority of absorbable sutures are now made of synthetic


polymer fibers, which may be braided or monofilament;

advantages of synthetic absirbable sutures over gut sutures:


-

ease of handling

low cost

low tissue reaction

consistent performance

guaranteed non-toxicity

NONABSORBABLE SUTURES are made of materials which are not


metabolized by the body, and have to be subsequently removed.

There are several materials used for non-absorbable sutures. The most
common is a natural fiber, silk, which undergoes a special manufacturing process
to make it adequate for its use in surgery. Other non-absorbable sutures are made
of artificial fibers, like polypropylene, polyester or nylon; these may or may not
have coatings to enhance their performance characteristics.

Ref.:
Carranzas Clinical Periodontology, 10th ed (2006).

Which of the following is an indication for ostectomy?


a.
b.
c.
d.
e.

Improvement of alveolar contours for flap adaptation


Multiple mandibular tori in an RPD patient
Excess tuberosity in a complete denture patient
Horizontal alveolar bone loss with irregular marginal bone height
Exostosis

Which of the following is an indication for ostectomy?


a.
b.
c.
d. Horizontal alveolar bone loss with irregular marginal bone height
e.

Osseous Surgery Indications:

recontour bone that forms part of the outer wall of the pocket

to prevent recurrence of the pocket

to reshape the alveolar crest

Osteoplasty Indications:

buccal or lingual bony ledges, tori, etc

intrabony defects associated with tilted molars

shallow buccal or lingual intrabony defects

flat interproximal areas

elimination of deep interproximal defects

incipient furcation involvements

for improvement of alveolar contours for flap adaptation

Ostectomy Indications:

elimination of interdental craters

intrabony pockets not amenable to reattachment procedures

horizontal alveolar bone loss with irregular marginal bone height

moderate and advanced furcation involvement

Friedman defined osteoplasty as a plastic procedure in which the


periodontal pocket is eliminated and the bone reshaped to achieve physiologic
contour of the bone and the gingiva overlying it. In this operation, the bone that is

reshaped is not part of the attachment apparatus, thus no bony support of the
tooth or teeth is lost.

Friedman defines osteoectomy (also referred to as ostectomy), as an


operative procedure in which bone that is part of the attachment apparatus, is
removed to eliminate a periodontal pocket and establish gingival contours that will
be maintained. Ostectomy requires the loss of some bony support of the tooth or
teeth, and the amount involved will be an important criterion for its use.

The most common indication for osseous resective surgery is to treat

the shallow two-wall osseous crater. Osseous craters are concavities in the
crest of the

interdental bone that are confined within the facial and lingual walls of the
alveolus.

Practically all craters have a slope from buccal and lingual walls to the base.
These

slopes represent more than two-thirds of the crater.

Contraindications for osseous resective surgery include deep osseous


craters, threewall

osseous defects, moderate to deep circumferential defects, and bony defects

situated on the buccal aspect of terminal mandibular molars associated with


the

external oblique ridge. It is important to remember that osseous surgery is


best-suited

to treating early and moderate periodontal defects. Advanced periodontal


lesions or

isolated deep craters may require some bone contouring, but not for the
express

purpose of eliminating the defect. Selective extraction, grafting procedures,


and/or

root amputations are often necessary to manage such areas. Three-wall bony
defects

should be managed by regenerative techniques since removal of supporting


bone

would often jeopardize the future of the affected tooth, as well as the
adjacent teeth.

Other precautions must also be taken into consideration before one decides
to use

osseous surgery. If too much supporting bone must be sacrificed on sound


teeth to


retain a neighboring affected tooth, it may be better to sacrifice the involved
tooth or

leave a residual bony defect. If a furcation will be exposed because of an


extensive

sacrifice of bone, it may be better to accept a deep gingival crevice. It should


also be

noted that osseous surgery should not be done in areas that have pocket
depths of less

than 5 mm. Shallow pockets treated with osseous surgery result in a net loss
of

attachment. Other contraindications for surgery are inadequate plaque


control by the

patient, noncompliance with supportive periodontal therapy, and certain


medical and

anatomic conditions.

Ref.:

Levine D, Filippelli G. A Review of Osseous Resective Surgery. Feb 1999


Journal of the

California Dental Association.

http://www.cda.org/library/cda_member/pubs/journal/jour299/osseous.html

Ideally, how is the presence of new attachment verified following


regenerative surgical techniques?

a. Absence of bleeding on probing


b. Histologic specimen
c. Reduction of the probing depth
d. Gingival margin at more coronal aspect (for tx of recession)
e. Radiographic evidence of new bone

Ideally, how is the presence of new attachment verified following


regenerative surgical techniques?

a.
b. Histologic specimen
c.
d.
e.

Attachment- collagen fibers connecting the root of the tooth to the alveolar
bone.

New Attachment- the ideal outcome of therapy that results in obliteration


of the pocket and reconstruction of the marginal periodontium.

Regeneration- the natural renewal of a structure, produced by growth and


differentiation of new cells and intercellular substances to form new tissues or
parts.

Reattachment is to attach again, the reunion of connective tissue with a


root surface on which viable periodontal tissue is present; the area of reattachment
is not affected by bacterial contamination (Mellonig 1992; AAP 2001a)

Techniques are available to try to provoke new attachment and are not
predictable. New attachment is the ideal outcome of therapy because it results in
obliteration of the pocket and reconstruction of the marginal periodontium and
entire periodontal attachment apparatus.

Histology is the only way we can truly verify the presence of new attachment
(utilizing reference notches on the tooth to substantiate new attachment)

Regenerative surgical techniques to develop new attachment include:

Non-graft associated new attachment

Removal of junctional and pocket epithelium

Guided tissue regeneration

Autogenous bone grafts

Regenerative flap procedures

Ref.:

Newman, MG, Takei, HH, Carranza, FA; Clinical Periodontology: Ch 63


Regenerative Osseous surgery. (2002) pp. 804-19.

In guided tissue regeneration, which of the following describes the purpose


of the membrane?

a. To contain the graft


b. To support the epithelium
c. To direct epithelium growth
d. To promote osteoconduction
e. None of the above

In guided tissue regeneration, which of the following describes the purpose


of the membrane?

a.
b.
c. To direct epithelium growth
d.
e.

Guided tissue regeneration (GTR):

Membrane- is applied over the osseous surgical wound site that serves to
prevent epithelial / connective tissue ingrowth (prior to bone fill).
Effectively: guiding or directing epithelium growth.

Animal experiments: use Millipore filters and Teflon membranes resulted in


regeneration of cementum and alveolar bone and a functional periodontal
ligament.

Clinical case reports: showed GTR results: gain in attachment level (not
necessarily associated with a buildup of alveolar bone)

Human histologic studies: evidence of periodontal reconstruction in most


cases, even with horizontal bone loss.

Ref.:

Carranzas Clinical Periodontology, 10th ed. (2006), pages: 972-3.

Rose, and Mealey, Periodontics, Medicine, Surgery and Implants, 2004,


pages: 573.

Laurell, L., Gottow, J. Guided tissue Regeneration update. Int Dent J.


(1998) Aug; 48(4):386-98.
ABSTRACT: The ultimate goal of periodontal therapy is to restore periodontal
tissues lost through disease or trauma. The most predictable way to accomplish
this goal is by guided tissue regeneration (GTR). The principle of GTR is to give
preference to certain cells to repopulate the wound area to form a new attachment
apparatus. Clinically this is accomplished by placing a barrier over the defect
thereby excluding gingival tissues from the wound during early healing. The first
generation of GTR barriers were non-resorbable which implies that they have to be
removed in a second surgical procedure. Resorbable barriers have recently been
introduced, changing GTR into a single-step procedure. Periodontal defects that will
predictably benefit from GTR therapy are intrabony, furcation class II and gingival
recession defects. This paper reviews the scientific evidence of what can be
achieved by GTR procedures for various periodontal defects as well as factors of
importance to increase the predictability of a successful treatment outcome.

Which of the following materials would be inappropriate for use as source of


epithelium in a root coverage surgery?

a. Free connective tissue autograft


b. Alloderm
c. Pedicle autograft
d. All of the above
e. None- all are acceptable sources of epithelium

Which of the following materials would be inappropriate for use as source of


epithelium in a root coverage surgery?

a.
b.
c.
d.
e. None- all are acceptable sources of epithelium

Miller Classification of Recession (1985)

Description

Anticipat
ed coverage

Recession that does

Complete

not
root coverage is
Class
I extend to the
mucogingival junction achievable

100%

Recession that

Complete
extends to or beyond the

mucovingival
junction,
root
with
coverage
is
Class II
no periodontal attachment
achievable
loss (ie bone, soft tissue)

100%

Recession that
Only partial
extends to or beyond the
root coverage
mucogingival junction, possible
with
to the
attachmentheight of the
Classperiodontal
III
loss in the interdental area
contour of
or malpositioning of theinterproximal
teeth
tissue

50-70%

Recession that

Root
extends to or beyond the
coverage is
mucogingival junction, with
unpredictable and

<10%
bone or soft-tissue
Classsevere
IV
requires adjunctive
loss in the interdental area
treatment (ie
and/or severe
orthodontics)
malpositioning of the teeth


List of techniques used for gingival augmentation coronal to the recession
(root coverage):
1. Free gingival autograft
2. Free connective tissue autograft
3. Pedicle Autograft
a. Laterally (horizontally) positioned
b. Coronally positioned
c. Semilunar pedicle (Tarnow)
4. Subepithelial connective tissue graft (Langer)
5. Guided tissue regeneration
6. Pouch and tunnel technique (Rathke Pouch)
7. Alloderm (Acellular dermal matrix allograft) (*best for muliple adjacent teeth
with recession)

Technique explanation in detail for each procedure in Carranza text.

Ref.:

Newman. Takei. Carranza. Carranzas Clinical Periodontology. 9th ed.


(2002) pp860-70

Novaes AB Jr, de Barros RR. Acellular dermal matrix allograft. The


results of controlled randomized clinical studies. J Int Acad Periodontol. 2008
Oct;10(4):123-9.

Modaressi M, Wang HL. Tunneling procedure for root coverage using


acellular dermal matrix: a case series. Int J Periodontics Restorative Dent.
2009 Aug;29(4):395-403.


RADIOLOGY, INFECTION
CONTROL

a.
b.
c.
d.
e.

Phenolics have been shown to be all of the following except?


Sporicidal
Tubercolocidal
Fungicidal
Virucidal
Bactericidal

Phenolics have been shown to be all of the following except?

a. Sporicidal
b.
c.
d.
e.

Phenol has occupied a prominent place in the field of hospital


disinfection since its initial use as a germicide by Lister in his
pioneering work on antiseptic surgery. In the past 30 years, however,
work has concentrated on the numerous phenol derivatives or
phenolics and their antimicrobial properties. Phenol derivatives
originate when a functional group (e.g., alkyl, phenyl, benzyl, halogen)
replaces one of the hydrogen atoms on the aromatic ring. Two phenol
derivatives commonly found as constituents of hospital disinfectants
are ortho-phenylphenol and ortho-benzyl-para-chlorophenol. The
antimicrobial properties of these compounds and many other phenol
derivatives are much improved over those of the parent chemical.
Phenolics are absorbed by porous materials, and the residual
disinfectant can irritate tissue. In 1970, depigmentation of the skin was
reported to be caused by phenolic germicidal detergents containing
para-tertiary butylphenol and para-tertiary amylphenol Manufacturers
data using the standardized AOAC methods demonstrate that
commercial phenolics are not sporicidal but are tuberculocidal,
fungicidal, virucidal, and bactericidal at their recommended usedilution.

Instruments
CLASS I CRITICAL All
invasive
instruments (e.g.,
surgical
instruments, IV
catheters,
implanted

Level of
Disinfec
tion
Sterility
required

Procedure
Moist heat, dry heat, or
ethylene oxide.

devices, etc.)
CLASS II SEMI-CRITICAL. All
instruments which
contact mucous
membranes (e.g.,
endoscopes,
airways,
anesthesia masks,
etc).

CLASS III - NONCRITICAL

High
level
disinfecti
on
required
between
patients.

Moist heat, 100 C for 30 min.;


aqueous 2% glutaraldehyde for
30-40 mins.; 1:10 dilution of
bleach for 20 mins. A sterile
water rinse after disinfection is
required to prevent
contamination from tap water.

Low
level
required

Chemical disinfectants include


ethyl or isopropyl alcohol (7090%), phenolic disinfectant
solutions, iodophors and
quartenernary ammonium
compounds. An exposure
(contact) time of at least 10
mins. is required.

NAME
Alcohol
s

Quater
nary
Ammon
ium
Chlorin
e

Aldehy
des

EXAMPLES
*
70% ethyl
alcohol
85%
isopropyl
alcohol
Roccal,
Cetylcide

COMMENTS

Sodium
hypochlorite
(Clorox
10%
solution)Chlo
rine dioxide
(Clidox,
Alcide)
Glutaraldehy
de
(Cidex,
Cide

Corrosive. Presence of organic matter


reduces activity. Chlorine dioxide
must be fresh ( <14 Days old ); kills
vegetative organisms within 3
minutes of contact.

Contact time required is 15 minutes.


Contaminated surfaces take longer to
disinfect. Remove gross
contamination before using.
Inexpensive.
Rapidly inactivated by organic
matter. Compounds may support
growth of gram negative bacteria.

Rapidly disinfects surfaces. Toxic.


Exposure limits have been set by
OSHA.

Phenoli
cs
Chlorhe
xidine

Wipes)
Lysol,
TBQ
Nolvasan ,
Hibiclens

Material
Stone Casts
Fixed
(Metal/Porcel
ain)
Removable
Dentures

Less affected by organic material


than other disinfectants.
Presence of blood does not interfere
with activity. Rapidly bactericidal and
persistent. Effective against many
viruses.

Method
Spray or immerse in
hypochlorite or iodophor
Immerse in glutaraldehyde

Immerse in iodophors or chlorine


compounds (Acrylic/Porcelain)
Immerse in iodophors or
Removable
chlorine compounds
Partials
(Metal/Acrylic)
Wax
Spray, wipe, spray with
Rims/Bites
iodophors
Indications for Sterilization or Disinfection of Dental Instruments

As with other medical and surgical instruments, dental instruments are


classified into three categories -- critical, semicritical, or noncritical -depending on their risk of transmitting infection and the need to
sterilize them between uses. Each dental practice should classify all
instruments as follows:

Critical. Surgical and other instruments used to penetrate soft tissue


or bone are classified as critical and should be sterilized after each use.
These devices include forceps, scalpels, bone chisels, scalers, and
burs.

Semicritical. Instruments such as mirrors and amalgam condensers


that do not penetrate soft tissues or bone but contact oral tissues are
classified as semicritical. These devices should be sterilized after each
use. If, however, sterilization is not feasible because the instrument will
be damaged by heat, the instrument should receive, at a minimum,
high-level disinfection.

Noncritical. Instruments or medical devices such as external


components of x-ray heads that come into contact only with intact skin
are classified as noncritical. Because these noncritical surfaces have a

relatively low risk of transmitting infection, they may be reprocessed


between patients with intermediate-level or low-level disinfection or
detergent and water washing, depending on the nature of the surface
and the degree and nature of the contamination.
After treatment of each patient and at the completion of daily work
activities, countertops and dental unit surfaces that may have become
contaminated with patient material should be cleaned with disposable
toweling, using an appropriate cleaning agent and water as necessary.
Surfaces then should be disinfected with a suitable chemical
germicide.
A chemical germicide registered with the EPA as a "hospital
disinfectant" and labeled for "tuberculocidal" (i.e., mycobactericidal)
activity is recommended for disinfecting surfaces that have been soiled
with patient material. These intermediate-level disinfectants include
phenolics, iodophors, and chlorine-containing compounds. Because
mycobacteria are among the most resistant groups of microorganisms,
germicides effective against mycobacteria should be effective against
many other bacterial and viral pathogens. A fresh solution of sodium
hypochlorite (household bleach) prepared daily is an inexpensive and
effective intermediate-level germicide. Concentrations ranging from
500 to 800 ppm of chlorine (a 1:100 dilution of bleach and tap water or
1/4 cup of bleach to 1 gallon of water) are effective on environmental
surfaces that have been cleaned of visible contamination. Caution
should be exercised, since chlorine solutions are corrosive to metals,
especially aluminum.
Low-level disinfectants -- EPA-registered "hospital disinfectants" that
are not labeled for "tuberculocidal" activity (e.g., quaternary
ammonium compounds) -- are appropriate for general housekeeping
purposes such as cleaning floors, walls, and other housekeeping
surfaces. Intermediate- and low-level disinfectants are not
recommended for reprocessing critical or semicritical dental
instruments.

Ref.
CDC Guideline for Disinfection and Sterilization in Healthcare Facilities,
2008
William A. Rutala, Ph.D., M.P.H., David J. Weber, M.D., M.P.H., and the
Healthcare Infection Control Practices Advisory Committee (HICPAC)

a.
b.
c.
d.
e.

What is the most common site in the mandible for a fracture?


Angle
Symphisis
Body
Ramusanswer
Sub-Condylar


What is the most common site in the mandible for a fracture?

a.
b.
c.
d.
e. Sub-Condylar

Condylar fractures comprise 29.1% of mandibular fractures. They are


followed by:

Angle-24.5%

Symphisis-22%

Body-16%

Alveolus-3.1%

Ramus-1.7%

Coronoid-1.3%

Non-specified-2.2%

Ref.
(Taken from Peterson, L. J., Contemporary Oral and Maxillofacial
Surgery, 3rd ed., 1998, Mosby, p. 595.)


A full mouth series of radiographs clearly shows carious lesions. You
are more interested in observing bony trabeculations. To increase
density and decrease contrast, you would?

a. Increase MA
b. Increase KVP
c. Increase MA, decrease KVP
d. Decrease KVP, increase time of exposure
e. Increase both KVP and MA

A full mouth series of radiographs clearly shows carious lesions. You


are more interested in observing bony trabeculations. To increase
density and decrease contrast, you would?

a.
b. Increase KVP
c.
d.
e.

Radiographic density: the degree of blackness or overall degree of


darkening of the exposed film. When a film is exposed to an x-ray
beam and is subsequently processed, the silver halide crystals that
have been exposed to x-ray photons are converted to grains of metallic
silver, which give the film its black appearance. The density or
blackness of the radiograph varies directly and proportionately as the
milliamperage and exposure time. As kilovoltage becomes larger, a
greater amount of x-radiation will strike the film emulsion, and result in
higher radiographic density if all other factors remain constant.
Radiographic contrast is defined as the difference in densities existing
between various regions on the film. A film that shows very light areas
and very dark areas is said to have high contrast, whereas a film that
shows areas of relatively light grey zones and dark grey zones is said
to be of low contrast. The higher the KVp the lower will be the
contrast, but the higher KVP will cause increased radiographic density.
When the kilovoltage is increased, the MA setting must be decreased
to maintain the same radiographic density. Contrast cant be varied by
a change in MA unless a variation in voltage is made to compensate for
the milliamperage variation. The higher the mA, and the lower the
voltage, the greater the contrast. Density can be altered without
changing the contrast. MA is the prime factor in controlling
radiographic density but is not a controlling factor in contrast. A
change in mA will produce a change in radiographic density but not a
noticeable change in contrast.
It is recommended that kilovoltages of 65-70 kVp be used for the bitewing films in the detection of caries. The kilovoltages in this range
produce a high contrast film (short scale contrast) which is good for
detection of caries. It is recommended that a high kVp (75-100)
technique be used for the periapical films, and a low kVp (65-70)
technique be used for the bite-wing film. The high kVp technique
would give a long-scale contrast (large number of small density
differences) to detect small details in the bone (ie. Trabecular patterns)
and a short scale contrast film to detect caries in the teeth.


Ref.
Langland, F., Sippy, F., and Langlais, R. Textbook of Dental Radiology.
2nd edition. Charles Thomas Pub. 1984. Pgs. 133,134,138, 451-452.


Which of the following does not result in the increase of a films
density?

a. Increasing the milliamperage (mA)


b. Increasing the kilovoltage (kVp)
c. Increasing the processing temperature
d. Increasing the exposure time


Which of the following does not result in the increase of a films
density?

a.
b.
c. Increasing the processing temperature
d.

Film density represents the degree of darkening of an exposed x-ray


film. White areas (e.g., metallic restorations) have no density and black
areas (air spaces) have maximum density. The areas in between these
two extremes (tooth structure, bone) are represented by various
shades of gray.
RADIOLUCENT: refers to high film density, which appears in a range
from dark gray to black. Soft tissue, air spaces, and pulp tissue, all of
which have low object density, appear as radiolucent areas on a film.
RADIOPAQUE: refers to area with low film density, which appear in a
range from light gray to white on the film. (The white areas of the
film are actually clear, but appear white when the light from a viewbox
passes through the film). Structures with high object density, such as
enamel, bone and metallic restorations will appear radiopaque.
The overall density of the film affects the diagnostic value of the film.
The center film below has the proper density. The one on the left is too
light (low density) and the one on the right is too dark (high density);
both of these films are of non-diagnostic quality.

Factors that influence film density:


Patient size: the larger the patients head, the more x-rays that are
needed to produce an ideal film density

Exposure factors (mA, kVp, exposure time): Some patients require a


change in exposure factors (increase for large adult, decrease for child)
to maintain proper film density. Increasing the kVp, mA and exposure
time all increase the density.
Object density: determined by type of material (metal, tooth
structure, composite, etc.) and by amount of material. Metallic
restorations have higher object density than tooth structure. Film
density decreases (film gets lighter) when object density increases,
assuming no changes are made in the exposure factors.
Film fog: This is an increased film density resulting from causes other
than exposure to the primary x-ray beam. This includes scatter
radiation, improper safelighting, improper film storage, and using
expired film. All of these things will cause extra silver halide crystals on
the film to be converted to black metallic silver, resulting in an overall
increase in the film density and making the film less diagnostic

fog

Ref.
Langland OE, Langlais RP, Preece J. Principles of dental imaging. 2nd
Ed. Lippincott Williams and Wilkins. 2002.
White SC, Pharoah MJ. Oral radiology: principles and interpretation.
Mosby. 2000.
Ohio State University College of Dentistry, Robert M. Jaynes, DDS, MS:
http://dent.osu.edu/radiology/resources.htm


Which of the following is not a type of type of sensor used in digital
radiography?

a. Charge-coupled device (CCD) detector


b. Complementary-Metal-Oxide-Semiconductor (CMOS) detector
c. Photo cellular detector (PCD)
d. Photostimulable phosphor plate (PSP)


Which of the following is not a type of type of sensor used in digital
radiography?

a.
b.
c. Photo cellular detector (PCD)
d.

Digital sensor types:


charge-coupled device (CCD) detector
Approximately the size of
dental film, the CCD sensor has
a slightly smaller sensitive area;
a thicker, rigid case and an
electrical lead that attaches to
the computer unit.

A radiation-sensitive circuit
inside of the case determines
the amount of voltage received
from the X-ray beam.
The specified amount is then
converted into a numerical
value, which is assigned a gray
level displayed on the computer
monitor.

Complementary-Metal-Oxyde-Semiconductor (CMOS) detector

CMOS detectors have the same characteristics as CCD sensors


except they use active pixel technology (patented by Schick

Technologies, Inc. for use in dental and medical radiology) and are less
expensive to manufacture. The sensors are visually indistinguishable.

photostimulable phosphor (PSP)

Used most often in medical radiology. The sensors are manufactured in


a variety of sizes similar to dental film (sizes 1, 2, 3, 4, and extraoral).
Unlike the CCD/CMOS, this detector does not require an electrical lead
and has properties similar to intensifying screen phosphors. As the
phosphor layer of the detector is irradiated, the electrons become
trapped in the phosphor. Thus, the plates hold the latent image until it
is processed. During processing with a laser, the electrons are
released and emit a blue light proportional to the intensity of the Xrays attenuated in the phosphor layer. The light is then converted to a
digital form, and the data can be displayed and seen on a computer
monitor.

AirTechniques Phosphor image processing unit 3


Advantages of Sensors (CCD and CMOS)
Instant image display
Consistent quality
Sensor remains in place for retakes
Allows intelligent repositioning of sensor or tube head

Disadvantages of Sensors (CCD and CMOS)


Cost
Unknown life-expectancy of sensor
Size of receptor/wires
Infection control
Requires computer in each operatory (unless using wireless system)

Advantages of Phosphor Plates


Size and thickness analogous to film
Wireless
Inexpensive
Computers not required in every operatory

Disadvantages of Phosphor Plates


Longer time for image acquisition (image must pass through digital
processor)
Quality and integrity of phosphor plate deteriorates over time
Latent images can be left from erasure process
Some require dark lighting conditions for processing and erasing

Ref.
Principles of Dental Imaging, 2nd Ed. (2002). Langland
Dunn SM, Kantor ML. Digital radiology: facts and fictions. JADA 1993;
124(12):39-47
Digital X-Ray Dental Forum 2006; A reference guide for evaluating and
purchasing digital X-ray
Presented by: Dr. Michelle Robinson


After the proper administration of the hepatitis B vaccine series, an
individual develops:

a. Anti-HBsAg
b. IgM anti-HBc
c. HBsAg
d. HBc


After the proper administration of the hepatitis B vaccine series, an
individual develops:

a. Anti-HBsAg (antibody to the hepatitis B surface antigen)


b.
c.
d.

Hepatitis B vaccine is a vaccine developed for the prevention of


hepatitis B virus infection. The vaccine contains one of the viral
envelope proteins, hepatitis B surface antigen (HBsAg). A course of
three (3) vaccine injections are given with the second injection at least
one month after the first dose and the third injection given six months
after the first dose. Afterward an immune system antibody to HBsAg is
established in the bloodstream. The antibody is known as anti-HBsAg.
This antibody and immune system memory then provide immunity to
hepatitis B infection. The first vaccine became available in 1981.
CLINICAL COURSE
The time course for acute HBV infection is depicted in the Figure
below. Several antibodies develop during infection reflecting the
immune systems response to different parts of the virus. The viral
antigens and antibodies that are detected during acute infection are
used to make the diagnosis of acute HBV and to determine whether the
infection has resolved or not.
Following acute infection, hepatitis B surface antigen (HBsAg) is
detected in the blood at a median 30-60 days after exposure to the
virus. The diagnosis of acute HBV infection is based on detection of
HBsAg and IgM antibody to hepatitis B core antigen (IgM anti-HBc) at
the time of symptoms. The IgM anti-HBc level declines within six
months and another form of antibody to hepatitis B core antigen called
IgG anti-HBc develops slightly later and persists indefinitely as a
marker of past infection. In persons whose immune systems eliminate
HBV, an antibody to HBsAg (called anti-HBs) becomes detectable
during convalescence and coincides with the disappearance of HBsAg.
The presence of anti-HBs indicates recovery from acute infection and
immunity from re-infection. Anti-HBs is also the antibody that develops
when vaccination is given. If the immune system is unable to eliminate

HBV, the HBsAg persists in the blood and anti-HBs is not detectable.
These persons have persistent virus and develop chronic hepatitis.

The primary determinant of whether a person with acute infection will


recover from the infection or go on to develop chronic hepatitis is age
at the time of infection. Newborns exposed to HBV develop chronic
infection in over 95% of cases. In contrast, 30% of children between
the ages of one and five years develop chronic infection after exposed
to HBV and only 1-5% of healthy adults develop chronic infection after
exposure.

Serologic Tests (measurement of HBV antigens and antibodies)

HBsAg: a marker present in persons who are currently infected with


HBV (i.e. persons with both recent infection and chronic infection).

Indicates presence of hepatitis B (in both acute and chronic


infection)

HBeAg: indicates phase of infection associated with high level of


hepatitis B in the blood

Anti-HBs (or HBsAb: antibody against HBsAg) - the protective


antibody that develops following recovery from HBV infection and after
vaccination.

When present this indicates resolved infection.


Anti-HBe (or HBeAb): antibody against HBeAg. When present, this is
associated with a lower level of hepatitis B virus in the blood.
Anti-HBc: antibody to hepatitis B core antigen (anti-HBc); produced in
all HBV infections (indicating infection at sometime in the past).

Indicates prior exposure to hepatitis B (present in acute and


chronic HBV and persists for life). There is little role of this test in the
follow-up of patients with chronic hepatitis B.

Ref:
Centers for Disease Control, USA (December 8, 2006). "Hepatitis B
Vaccine: Fact Sheet
http://www.hepb.org/hepb/vaccine_information.htm
http://knol.google.com/k/norah-terrault/hepatitis-b/1w026jckgcwg2/3#


When viewing a panoramic radiograph, the mandibular incisors are
noticeably wider than the actual teeth. What malpositioning error could
you attribute this to?

a. Patient positioned too far forward


b. Patient positioned too far backward
c. Patient not fully biting down on bite tab
d. Patient did not have tongue on roof of mouth during x-ray


When viewing a panoramic radiograph, the mandibular incisors are
noticeably wider than the actual teeth. What malpositioning error could
you attribute this to?

a.
b. Patient positioned too far backward
c.
d.

-Pt positioned too far forward: Teeth appear narrow


-Pt positioned too far backwards: Teeth appear wide
-Pt head rotated to the left: (right side now closer to receptor) Right
side narrow teeth, left wide
(should be easily identifiable from skeletal asymmetry)
-Pt chin too far down: excessive curving of occlusal plane
-Pt chin too far up: flat appearing occlusal plane
-Pt slumped over: ghost image of cervical spine may obscure midline

Ref:
White. Pharoah. Oral Radiology: Principles and Interpretation. (5th
ed) 2004. p. 192-209.
Also link to lecture with photos:
http://www.mmcpub.com/pdf/1999jph/199902jph_pdf/99jphv8n2p15.pd
f


If a HBsAg serology test comes back positive, which of the following is
true of the patient?

a. The patient had a recent immunization


b. The patient has an active infection
c. The patient is immune
d. The patient has been exposed to the antigen, but is not a carrier


If a HBsAg serology comes back positive, which of the following is true
of the patient?

a.
b. The patient has an active infection
c.
d.

Ref: http://www.cdc.gov/hepatitis/index.htm Click link to Medical


Professional and HepB Serology


Which of the following skull radiographs offers the best view of the
zygomatic arches?

a. A-P (Townes) view


b. Caldwells view
c. Modified Townes view
d. Occipito-mental (Waters) view
e. Submental vertex (jug handle) view


Which of the following skull radiographs offers the best view of the
zygomatic arches?

a.
b.
c.
d.
e. Submental vertex (jug handle) view

In an operational environment, the comprehensive dentist should be


familiar with the four basic skull views: Waters, Posterior-Anterior,
Lateral, and the Submentovertex for evaluating facial trauma.
Submental vertex (Jug handle) offers the best view of the zygomatic
arches.
The A-P view of the skull is the best for frontal sinus and orbital ridges.
The Modified Townes gives the best view of the condylar neck.
The Waters view is the best for the maxillary sinuses.
CEPHALOMETRIC RADIOGRAPHY - head measurement x-ray; taken
using a cephalostat [head-holding device] in a precise manner so it can
be repeated at a later date for comparison.
The first word of the description of the x-ray is where the tube is; the
second word is where the film is, i.e. a PA film has the tube at the back
of the head [posterior] and the film is placed by the face [anterior].

Waters: (Occipitomental projection, semiaxial projection)


-variation of the posterior anterior projection.
-Best view for evaluating facial fractures.
-Zygomatic arches, orbital rims and floors, nasal spine and septum,
coronoid processes.

-Can also be used for evaluating the frontal, maxillary, and sphenoid
sinuses.

Semi-axial projection (Water's view). Both maxillary antra are well


depicted and the orbital floor with the infraorbital canal (black arrow)
is separated from the orbital rim (open arrow). The sphenoid
compartments can be seen through the open mouth (white arrow).

Posterior-anterior:
-can be used to evaluate orbital rim, frontal and ethmoid sinuses, nasal
septum, nasal fossa and orbits; invaluable in detecting a fracturedislocation
Caldwells Projection: PA view w/ 15 tilt of the tube caudally

- advantage over straight PA - view of orbits unobstructed by petrous


ridges

Frontal projection (Caldwell view). Frontal sinuses and the ethmoid


aircells are visualized while the petrous bones hide the maxillary antra.

Submentovertex: (axial projection)


-view used to evaluate fractures and displacement of a fractured
zygomatic arch
NOTE: when wish to view the zygomatic arch, the exposure time should
be reduced to 1/3 that is used to view the skull
-contraindicated with patients who have a suspected spinal injury

Axial projection. The posteriolateral walls of the maxillary antraform a smooth s-shaped curve (black
arrow) while the lateral walls of the orbits form a straight line (open
arrow). The depth and posterior wall of frontal sinuses can also be
visualized (asterisk).

Lateral Skull (Cephalometric):

-Used to survey the skull and facial bones for evidence of disease,
trauma, developmental abnormalities, in ortho. Used for assessing
head growth.

Lateral projection. A well developed sphenoid sinus can be seen


extending back under the pituitary fossa. The posterior walls of the
frontal and the maxillary sinuses can also be outlined (arrows).
Panorex: Best single view short of a CT for viewing the mandible.
-View of choice for viewing condyles.

Lateral Oblique: Excellent for viewing the mandibular body and


ramus

Towne's: (anterior-posterior projection) AP view w/ 30 tilt of the tube


caudally
-view can be used to observe condyles, necks, rami and mandibular
symphysis

Reverse Towne's: (Modified Townes) Posterior-Anterior view, mouth


open
-View can be used to observe fractures involving the condylar neck,
and also when displacement of the condyle is suspected

Ref.
Dolan, Jacoby and Smoker; Radiology of Facial Injury, Field and Wood,
Inc., 1988

White and Pharoah, Oral Radiology, principles and interpretation, 5th


ed., 2004 Mosby, ISBN-10: 0-323-02001-1.
Oral and Maxillofacial Surgery, 4th Edition, 2003.


On a radiograph, the buccal root of a maxillary first premolar would
appear distal to the lingual root if:

a. The vertical angle of the cone is increased.


b. The vertical angle of the cone is decreased.
c. The x-ray head is angled from a mesial position relative to the
premolar.
d. The x-ray head is angled from a distal position relative to the premolar.


On a radiograph, the buccal root of a maxillary first premolar would
appear distal to the lingual root if:

a.
b.
c. The x-ray head is angled from a mesial position relative to the
premolar.
d.

The SLOB rule is one of the most widely used radiographic concept in
endodontics. On periapical radiographs, roots are often superimposed
upon one another and require separation for proper identification.
The SLOB rule is an acronym for Same Lingual Opposite Buccal.
The premise is that one radiograph is taken straight on at a 90 degree
angle to the tooth and a second radiograph is taken with the tubehead
shifted either mesially or distally.
The object imaged will move in the same direction as the tubehead is
moved if it is located on the lingual (Same Lingual).
The object being imaged will move opposite the tubehead movement if
it is located on the buccal (Opposite Buccal).
An example of this would be a palatal root, which is on the lingual side
of a maxillary molar, will move mesially on the image if the tubehead
moves mesially (Same Lingual).

Figure 4a.
Postoperative
radiograph of the
mandibular first
molar.

Figure 4b. The


mesio-distal view
of the same tooth
demonstrates that
in addition to the 2
canals of the distal
root, the mesial
canals have
independent apical
foramina that
communicate at
the level of the
middle one third.

Figure 7c. Case


2: radiographic
examination of
the obturation
shows
superimposition
of the root canal
fillings.

Figure 7d. Case


2: mesial-todistal angulation
produces
separation of the
canals. The
mesiolingual and
the distolingual
root-filled canals
are shifted
mesially, and the
mesiobuccal and
the distobuccal

root-filled canals
are shifted
distally on the
radiograph. Note
the mesial
anastomosis in
the middle third
as a result of
proper cleaning,
shaping, and
canal obturation.

Ref.
Alborz S., Imaging in Endodontics, From the Winter 2008 AADMRT
Newsletter.
Richards, A. G., The Buccal Object Rule, Dental Radiography and
Photography, Vol. 53 Number 3, 1980, pages 37-57
White, S.C. and Pharoah, M. J., Oral Radiology, Principles and
Interpretation, Fourth Edition, 2000.


Which of the following is considered a low-level disinfectant?

a. Quarternary ammonium
b. Iodophors
c. Chlorine compounds
d. Glutaraldehyde
e. Hydrogen peroxide


Which of the following is considered a low-level disinfectant?

a. Quarternary ammonium
b.
c.
d.
e.

High level disinfectants:


Glutaraldehydes, hydrogen peroxide, hydrogen peroxide with peracetic
acid
Intermediate level disinfectants:
Iodophors, chlorine compounds, quarternary ammonium compounds
with iodophors or phenolics or chlorines
Low level disinfectants:
Quarternary ammonium compounds alone are considered

Alcohol based compounds may evaporate more quickly, and hinder the
removal of the bioburden. A spray-wipe-spray technique should be
employed for best results. Between patients disposable barriers can
be used to save time.

This article is available in PDF Format: Hard-surface


disinfectants

What products are what?


Glutaraldehyde
Banicide (Pascal)
Banicide Plus (Pascal)

Cidex Plus 28-day Solution Johnson & Johnson)

Metricide 28 (Metrex Research Corp.)

MaxiCide (Henry Schein Inc.)

MaxiCide Plus (Henry Schein Inc.)

ProCide (Cottrell, Ltd.)

Sterall (Colgate-Hoyt)

Wavicide-01 (Wave Energy Systems Inc.)


Phenol
Asepti-Phene RTU Disinfectant Deodorant, EPA Reg. No. 334417-303
(Huntington Laboratories; exclusively distributed by Patterson Dental)

Birex SE, EPA Reg. No. 1043-92-51003 (Biotrol International)

Dual-X, EPA Reg. No. 34810-21-51003 (Biotrol International)

Professional Lysol Brand Disinfecting Spray, EPA Reg. No. 777-53


(Reckitt & Colman, Inc.; exclusively distributed by Sultan Chemists)
ProPhene/ProPhene Plus, EPA Reg. No. 46851-1-50611 (Cottrell Ltd.)

Sporicidin Brand Disinfectant Towelette, EPA Reg. No. 8383-7


(Sporicidin International)

Vital Defense - D Disinfectant Solution, EPA Reg. No. 46851-1-9273


(Block Drug)
Sodium hypochlorite (bleach)
Dispatch Hospital Cleaner Disinfectant Towels with Bleach, EPA Reg.
No. 56392-8 (Caltech Industries)

Dispatch Spray, EPA Reg. No. 56392-7 (Caltech Industries)

Sodium bromide and sodium dichloroisocyanurate dihydrate


MicroStat 2, EPA Reg. No. 70369-1 (ChemLink Laboratories, LLC)
Iodophores
Asept-All Iodophore Surface Disinfectant Cleaner Concentrate, EPA Reg.
No. 5185-22418184 (Sultan Chemists)
Biocide, EPA Reg. No. 4959-16-51003 (Biotrol International)

Huntington Iodophor, EPA Reg. No. 303-63 (Huntington Laboratories;


exclusively distributed by Patterson Dental)

Huntington Hi-Sine Detergent Germicide, EPA Reg. No. 303-63


(Huntington Laboratories; exclusively distributed by Patterson Dental)

Iodo Five, EPA Reg. No. 4959-16 (Cottrell, Ltd.)


Quaternary ammonium chlorides
Cetylcide II, EPA Reg. No. 61178-1-3150 (Cetylite Industries Inc.)
Citrus II Hospital Germicidal Deodorizing Cleaner, EPA Reg. No. 183983-68939 (Beaumont Products Inc.)

DisCide TB, EPA Reg. No. 1839-83-10492 (Palmero Health Care)

Madacide-1 Hospital Disinfectant/Decontaminant Cleaner Detergent/


Disinfectant, EPA Reg. No. 1839-83-11703 (Mada Medical Products Inc.)

Precise QTB Hospital Cleaner Disinfectant, EPA Reg. No. 1839-83-56392


(Caltech Industries)

Sterall Plus Spray, EPA Reg. No. 46781-6-48930 (Colgate Oral


Pharmaceuticals, a subsidiary of Colgate-Palmolive Co.)
New Generation Quat/Alcohol Technology
Low-concentration products
Asepti-Care TB+II, EPA Reg. No. 1130-15-1677 (Ecolab Inc.)
Asepti-Wipe, EPA Reg. No. 9480-5-303 (Huntington Laboratories;
exclusively distributed by Patterson Dental)

Cavicide, EPA Reg. No. 46781-6 (Metrex Research Corp.)

Envirocide, EPA Reg. No. 46781-6 (Metrex Research Corp.)

GC Spray-Cide, EPA Reg. No. 1130-15-10214 (GC America Inc.)

Madacide-FD, EPA Reg. No. 1130-15-11703 (Mada Medical Products


Inc.)

MaxiSpray Plus, EPA Reg. No. 46781-6-10597 (Henry Schein Inc.)

Medica DC10 Germicidal Solution, EPA Reg. No. 1130-15-65575 (First


Medica)

MetriWipe, EPA Reg. No. 9480-5-46781 (Metrex Research Corp.)

Sani-Cloth Germicidal Cloth, EPA Reg. No. 9480-5 (PDI)

Sani-Cloth Plus Germicidal Wipes, EPA Reg. No. 9480-6 (PDI)

Sanitex Plus One Step Cleaner, Disinfectant, and Deodorizer, EPA Reg.
No. 1130-15-64285 (Cross Country Paper Products)
High-concentration products
DisCide ULTRA Disinfecting Spray, EPA Reg. No. 10492-5 (Palmero
Health Care)
DisCide ULTRA Disinfecting Towelettes, EPA Reg. No. 10492-4 (Palmero
Health Care)
Super Sani-Cloth Germicidal Wipes, EPA Reg. No. 9480-4 (PDI)

Ref.
J.C. Hoos, Balancing the Art, Science, and Business of Dentistry Hardsurface disinfectant; Dental Explorations LLC.
Cottone,JA Terezhalmy,GT Molinari,JA; Practical Infection Control in
Dentistry; 2nd edition, Williams and Wilkins, Philadelphia, PA; pp. 161166.
Cohn, WG, et al.; Guidelines for Infection Control in Dental HealthCare Settings 2003; MMWR, vol. 52, RR-17, December 13, 2003;
pp. 20-21
APIC Text of infection Control and Epidemiology, 2nd edition, Jan 2005,
51-16

Which of the following would yield the clearest diagnostic image?


A. Long source/film distance, Short object/film distance
B. Short source/film distance, Short object/film distance
C. Long source/film distance, Long object/film distance
D. Short source/film distance, Long object/film distance

Which of the following would yield the clearest diagnostic image?


A. Long source/film distance, Short object/film distance
B.
C.
D.

Source: X-ray Tube head


Object: tooth being x-rayed
Film: X-ray film or digital sensor
Ideally:
Long source/film distance
Short object/film distance
X-rays are created from a point on the tungsten target of the anode (in
an x-ray tube); from this point they radiate in various directions unless
absorbed. Thus, the need for collimation (or directing the ray through
selective absorption of scatter or unusable x-rays). Long source/film
distances allow long collimation. This allows the collimator to absorb
many of the x-rays that are not traveling straight down the x-ray tube
towards the object. The x-rays leaving a long collimator are more
perpendicular to the object being x-rayed. As x-rays pass through the
object some are absorbed depending on what they pass through, some
travel directly through and some are scattered. As the object/film
distance increases scattered x-rays are picked up further and further
from the site from which they originally were scattered (e.g. the tooth).
The longer the object/film distance the more scatter radiation is picked
up as fog. Short object/film distance decreases the fog phenomenon
and yields a truer image of the actual object (less fog).


Reference:
White. Pharoah. Oral Radiology: Principles and Interpretation. (5th
ed) 2004. Pp. 86-93.


According to the EPA and the ADA what are the maximum colony
forming units/ml allowed in Drinking water and being dispensed from
dental units (respectively)?

A. 250 CFU/ml; 200 CFU/ml


B. 500 CFU/ml; 200 CFU/ml
C. 1000 CFU/ml; 500 CFU/ml
D. 200 CFU/ml; 100 CFU/ml


According to the EPA and the ADA what are the maximum colony
forming units/ml allowed in Drinking water and being dispensed from
dental units (respectively)?

A.
B. 500 CFU/ml; 200 CFU/ml
C.
D.

This statement was in response to scientific evidence that the


microbiologic quality of water used in dental treatment could be
improved; and called for the design of dental equipment so that, by the
year 2000, water delivered to patients during nonsurgical dental
procedures consistently contained no more than 200 colony-forming
units per milliliter (cfu/ml) of aerobic mesophilic heterotrophic bacteria
at any point in time in the unfiltered output of the dental unit. Since
1995, technological advances have made this goal possible. In
addition, the CDC now recommends that coolant water used in nonsurgical dental procedures meet EPA regulatory standards for drinking
water, which is less than or equal to 500 colony forming units of
heterotrophic bacteria per milliliter of water. This CDC recommendation
was published in their Guidelines for Infection Control in Dental HealthCare Settings 2003

(http://www.cdc.gov/mmwr/preview/mmwrhtml/rr5217a1.htm)
http://www.ada.org/prof/resources/positions/statements/lines.asp


Which of the following statements is NOT true concerning radiographic
imaging of a horizontal root fracture.

A. The fracture may be difficult to see due to minimal distraction of the


fragments.
B. Root fractures are typically horizontal and oblique- requiring multiple
angles to be taken for proper visualization.
C. The PDL space and width will always be widened on a tooth with a
horizontal root fracture.
D. The fracture line may appears as a more poorly defined gray
shadow, especially if the xrays are not traveling in the same plane as
the fracture.


Which of the following statements is NOT true concerning radiographic
imaging of a horizontal root fracture.

A.
B.
C. The PDL space and width will always be widened on a tooth with a
horizontal root fracture.
D.

The PDL space and width will be normal on a tooth with a horizontal
root fracture.

Fractures can occur at any location along the tooth root, and on any or
all roots of multi-rooted teeth.
These can be hard to image due to the horizontal and oblique planes of
fracture and especially if the fragments havent distracted from each
other; these teeth require multiple angled x-rays to view.
Many times, the only evidence on a radiograph may be seen in the PDL
adjacent to the fracture: there is a localized increase in the PDL space.

Reference: White. Pharoah. Oral Radiology: Principles and


Interpretation. (5th ed) 2004. p. 619-24.


The Centers for Disease Control and Prevention recommends Biological
Monitoring be performed at least?

a. Bi-weekly
b. Daily
c. Weekly
d. After every cycle
e. Monthly


The Centers for Disease Control and Prevention recommends Biological
Monitoring be performed at least?

a.
b.
c. Weekly
d.
e.

Correct functioning of sterilization cycles should be verified for each


sterilizer by the periodic (at least weekly) use of Biological Indicators
(BIs). Users should follow the manufacturer's directions concerning the
appropriate placement of the BI in the sterilizer. A control BI (not
processed through the sterilizer) from the same lot as the test indicator
should be incubated with the test BI. The control BI should yield
positive results for bacterial growth. In addition to conducting routine
biological monitoring, equipment users should perform biological
monitoring.

Whenever a new type of packaging material or tray is used.


After training new sterilization personnel.
After a sterilizer has been repaired.
After any change in the sterilizer loading procedures.

How is the sterilization process monitored?

Sterilization procedures should be monitored through a combination of


mechanical, chemical, and biological techniques designed to evaluate
the sterilizing conditions and the procedure's effectiveness.

Mechanical techniques for monitoring sterilization include assessing


the cycle time, temperature, and pressure of sterilization equipment by
observing the gauges or displays on the sterilizer. Some tabletop
sterilizers have recording devices that print out these parameters.
Correct readings do not ensure sterilization, but incorrect readings
could be the first indication that a problem has occurred with the
sterilization cycle.

Chemical indicators, internal and external, use sensitive chemicals to


assess physical conditions such as temperature during the sterilization
process. Chemical indicators such as heat sensitive tape change color
rapidly when a given parameter is reached. An internal chemical

indicator should be placed in every sterilization package to ensure the


sterilization agent has penetrated the packaging material and actually
reached the instruments inside. An external indicator should be used
when the internal indicator cannot be seen from outside the package.
Single-parameter internal indicators provide information on only one
sterilization parameter and are available for steam, dry heat, and
unsaturated chemical vapor. Multiparameter internal indicators
measure 23 parameters and can provide a more reliable indication
that sterilization conditions have been met. Multiparameter internal
indicators are only available for steam sterilizers (i.e., autoclaves).
Refer to manufacturer instructions for proper use and placement of
chemical indicators.
Indicator test results are shown immediately after the sterilization
cycle is complete and could provide an early indication of a problem
and where the problem occurred in the process. If the internal or
external indicator suggests inadequate processing, the item that has
been processed should not be used. Because chemical indicators do
not prove sterilization has been achieved, a biological indicator (i.e.,
spore test) is required.

Biological indicators (BIs) are the most accepted means of monitoring


the sterilization process because they directly determine whether the
most resistant microorganisms (e.g., Geobacillus or Bacillus species)
are present rather than merely determine whether the physical and
chemical conditions necessary for sterilization are met. Because spores
used in BIs are more resistant and present in greater numbers than are
the common microbial contaminants found on patient care equipment,
an inactivated BI indicates that other potential pathogens in the load
have also been killed.

If sterilizing an implantable device, should users perform


biological monitoring (spore testing) more frequently?

Any load containing implantable devices should be monitored.


Ideally, implantable items should not be used until the results of tests
are known to be negative. As previously noted, the manufacturer's
directions concerning the appropriate placement of the biologic
indicator (BI) in the sterilizer must be followed. A control BI (not
processed through the sterilizer) from the same lot as the test indicator
should be incubated in the same manner as the test BI. The control
biological indicator should yield positive results for bacterial growth.

What should I do if a spore test result is positive?

If the mechanical (e.g., time, temperature, pressure) and


chemical (internal or external) indicators suggest that the sterilizer is

functioning properly, a single positive spore test result probably does


not indicate sterilizer malfunction. Items other than implantable items
do not necessarily need to be recalled; however, sterilizer operators
should repeat the spore test immediately using the same cycle that
produced the positive BI. The sterilizer should be removed from service
and sterilization operating procedures reviewed to determine whether
operator error could be responsible.

If the result of the repeat spore test is negative and operating


procedures were correct, then the sterilizer can be returned to service.
If the repeat spore test result is positive, do not use the sterilizer until
it has been inspected or repaired and rechallenged with BI tests in
three consecutive empty-chamber sterilization cycles. When possible,
items from suspect loads dating back to the last negative BI should be
recalled, rewrapped, and resterilized.

Results of biological monitoring and sterilization monitoring


reports should be recorded.

Common Factors Influencing the Effectiveness of Sterilization

Causes

Potential Problem

Improper cleaning of
instruments

Protein and salt debris may


insulate organisms from
direct contact with the
sterilizing agent and interfere
with the efficacy of the
sterilization agent.

Improper packaging

Wrong packaging material for


the method of sterilization

Excessive packaging material

Prevents penetration of the


sterilizing agent; packaging
material may melt.

Retards penetration of the


sterilizing agent.

Improper loading of the


sterilizer

Overloading

No separation between
packages or cassettes even
without overloading

Increases heat-up time and


will retard penetration of the
sterilizing agent to the center
of the sterilizer load.

May prevent or retard


thorough contact of the
sterilizing agent with all
items in the chamber.

Improper timing and


Insufficient time at proper
temperature
temperature to kill

organisms.
Incorrect operation of the
sterilizer

Modified from Miller CH and Palenik CJ (1998).

References and Additional Resources


American Dental Association. Biological indicators for verifying
sterilization. J Am Dent Assoc 1988; 117:653654.
American Dental Association. Guidelines for infection control in the
dental office and the commercial dental laboratory. J Am Dent Assoc
1996;127:672680.
Andres MT, Tejerina JM, Fierro JF. Reliability of biologic indicators in a
mail-return sterilization-monitoring service: A review of 3 years,
Quintessence Intern 1995;25:865870.
Association of Operating Room Nurses. AORN standards and
recommended practices for perioperative nursing. 1987;Section
III:14.1-III:14.11.
Association of Operating Room Nurses. Recommended practices for
sterilization in perioperative practice settings. In: Fogg D, Parker N,
Shevlin D, eds. Standards, Recommended Practices, and Guidelines,
Denver: AORN, 2002;333342.
CDC. Guidelines for Infection Control in Dental Health-Care Settings,
2003. MMWR, December 19, 2003:52(RR-17).
CDC Recommended infection control practices for dentistry, 1993.
MMWR 1993;42(No. RR-8):112.


Ideally, HEP B vaccinations should begin at least _____ months before
travel?

a. 2 months
b. 3 months
c. 4 months
d. 6 months
e. 9 months


Ideally, HEP B vaccinations should begin at least _____ months before
travel?

a.
b.
c.
d. 6 months
e.

There is no data with which to assess the risk for HBV infection among
U.S. travelers. The risk for HBV infection for international travelers is
considered generally low, except for travelers to countries where the
prevalence of chronic HBV infection is intermediate or high. Some
travelers, such as adventure travelers, Peace Corps volunteers,
missionaries, and military personnel, may be at increased risk for
infection.
Situations or activities that may carry increased risk for HBV infection
for travelers while overseas include the following:
An injury or illness that requires invasive medical attention (e.g.,
injection, IV drip, transfusion, stitching)

Dental treatment
Unprotected sexual contact
Sharing illegal drug injection equipment
Skin-perforation practices (e.g., tattooing, ear piercing, acupuncture)
Cosmetic practices with risk for skin perforation (e.g.,
manicure/pedicure)
Sharing personal grooming items (e.g., earrings, toothbrush, razor)

Prevalence of chronic infection with hepatitis B virus, 2006

Clinical Presentation
Incubation period of hepatitis B is typically 90 days (range: 60150
days) from exposure to onset of jaundice.
Constitutional symptoms such as malaise and anorexia may precede
jaundice by 12 weeks.
Clinical symptoms and signs include nausea, vomiting, abdominal pain,
and jaundice.
Skin rashes, joint pain, and arthritis may occur.
Infants, children<5 years of age, and immunosuppressed adults with
newly acquired HBV infection typically are asymptomatic.
Infected persons 5 years of age, including immunocompetant adults,
30%50% have initial clinical signs or symptoms.
The casefatality rate of acute hepatitis B is approximately 1%.
Acute HBV infection causes chronic (long-term) infection in 30%90%
of persons infected as infants or young children and in <5% of
adolescents and adults.
Chronic infection can lead to chronic liver disease, liver scarring
(cirrhosis), and liver cancer.

Diagnosis
At least one serologic marker is present during each of the different
phases of HBV infection. The serologic markers are typically used to
differentiate between acute, resolving, and chronic infection (Table 28).

Treatment
No specific treatment is available for acute illness caused by hepatitis
B. Antiviral drugs are approved for the treatment of chronic hepatitis B.
Preventive Measures for Travelers
Vaccine
Hepatitis B vaccination should be administered to all unvaccinated
persons traveling to areas with intermediate to high levels of endemic
HBV transmission (i.e., with hepatitis B surface antigen [HBsAg]
prevalence 2%).
Hepatitis B vaccination is currently recommended for all U.S. residents
who work in health-care fields (e.g., medical, dental, laboratory) that
involve potential exposure to human blood.
All unvaccinated U.S. children and adolescents (<19 years of age)
should receive hepatitis B vaccine.
Unvaccinated persons who have indications for hepatitis B vaccination
independent of travel should be vaccinated (e.g., men who have sex
with men, injection drug users, anyone who has recently had a sexually
transmitted disease or has had more than one sex partner in the
previous 6 months).
Vaccine Dose and Administration
The vaccine is usually administered as a three-dose series on a 0-, 1-,
and 6-month schedule (see Table 2-9). The second dose should be
given 1 month after the first dose; the third dose should be given at
least 2 months after the second dose and at least 4 months after the
first dose.
Alternatively, the vaccine ENGERIX-B, manufactured by
GlaxoSmithKline, is also approved for administration on a four-dose
schedule at 0, 1, 2, and 12 months.
There is also a two-dose schedule for RECOMBIVAX HB, a vaccine
produced by Merck & Co., Inc., which has been licensed for children
and adolescents 1115 years of age. Using the two-dose schedule, the
adult dose of RECOMBIVAX HB is administered, with the second dose
given 46 months after the first dose.
A three-dose series that has been started with one brand of vaccine
may be completed with the other brand.
TWINRIX, manufactured by GlaxoSmithKline, is a combined hepatitis A
and hepatitis B vaccine licensed for persons 18 years of age or older.
Primary immunization consists of three doses, given on a 0-, 1-, and 6month schedule.
Special Situations
Ideally, vaccination should begin at least 6 months before travel so the
full vaccine series can be completed before departure. Because some
protection is provided by one or two doses, the vaccine series should
be initiated, if indicated, even if it cannot be completed before
departure. Optimal protection, however, is not conferred until after the

final vaccine dose. Travelers should be advised to return for completion


of the vaccine series.
An accelerated vaccine schedule could be used for those traveling to
endemic areas at short notice and facing imminent exposure because
of behavioral risks or to emergency responders to disaster areas. The
monovalent hepatitis B vaccines can be used at 0, 7, and 14 days. If an
accelerated schedule is used, the patient should receive a booster
dose at least 6 months after the start of the series to promote longterm immunity.
An accelerated vaccine schedule with TWINRIX (hepatitis A and
hepatitis B vaccine) can also be used (doses at 0, 7, and 2130 days).
In this situation, a booster dose should be given at 12 months to
promote long-term immunity.
For children and adults whose immune status is normal, booster doses
of vaccine are not recommended. Serologic testing to assess antibody
levels is not necessary for most vaccinees.
Vaccine Safety and Adverse Reactions
Hepatitis B vaccines have been shown to be safe for persons of all
ages. Pain at the injection site (3%29%) and elevated temperature
higher than 37.7 C (99.9 F) (1%6%) are the most frequently
reported side effects among vaccine recipients.
These vaccines should not be administered to persons with a history of
hypersensitivity to any vaccine component, including yeast. The
vaccine contains a recombinant protein (HBsAg) that is noninfectious.
Limited data indicate that there is no apparent risk of adverse events
to the developing fetus when hepatitis B vaccine is administered to
pregnant women. HBV infection affecting a pregnant woman can result
in serious disease for the mother and chronic infection for the
newborn. Neither pregnancy nor lactation should be considered a
contraindication for vaccination.
Other Preventive Measures
As part of the pre-travel education process, all travelers should be
given information about the risks for hepatitis B and other bloodborne
pathogens from contaminated medical equipment, injection drug use,
or sexual activity and informed of prevention measures (see below),
including hepatitis B vaccination, that can be used to prevent
transmission of HBV.
Regardless of destination, all persons who may engage in practices
that put them at risk for HBV infection during travel should receive
hepatitis B vaccination if previously unvaccinated.
Any adult seeking protection from HBV infection should be vaccinated.
Acknowledgment of a specific risk factor is not a requirement for
vaccination.
Behavioral preventive measures for HBV infection are similar to those
for HIV infection and AIDS.

When seeking medical or dental care, travelers should be advised to


be alert to the use of medical, surgical, and dental equipment that has
not been adequately sterilized or disinfected, reuse of contaminated
equipment, and unsafe injecting practices (e.g., reuse of disposable
needles and syringes).
HBV and other bloodborne pathogens (e.g., HIV and hepatitis C) can be
transmitted if tools are not sterile or if the tattoo artist or piercer does
not follow other proper infection-control procedures (e.g., washing
hands, using latex gloves, and cleaning and disinfecting surfaces and
instruments).
Travelers should be advised to consider the health risks in deciding to
get a tattoo or body piercing in areas where adequate sterilization or
disinfection procedures might not be available or practiced.

Table 2-08. Typical interpretation of serologic test results for


hepatitis B virus infection
Serologic Marker

H
B
s
A
g
1

T
o
t
a
l
a
n
t
i
H
B
c
2

Interpretation

I
g
M
3

a
n
t
i
H
B
c

Never infected

Early acute infection; transient (up to


18 days) after vaccination

Acute infection

+
6
,
7

Serologic Marker

H
B
s
A
g
1

T
o
t
a
l
a
n
t
i
H
B
c
2

Interpretation

I
g
M
3

a
n
t
i
H
B
c

Acute resolving infection

Recovered from past infection and


immune

Chronic infection

False-positive (i.e., susceptible); past


infection; low-level chronic infection;8
or passive transfer of anti-HBc to infant
born to HBsAg-positive mother

Immune if concentration is 10
mIU/mL after vaccine series
completion;9 passive transfer after
hepatitis B immune globulin
administration

From CDC. MMWR Recomm Rep. 2006; 55(RR-16):125.


1Hepatitis B surface antigen.
2IgG antibody to hepatitis B core antigen.
3Immunoglobulin M.
4Antibody to HBsAg.
5Negative test result.
6Positive test result.
7To ensure that an HBsAg-positive test result is not a false-positive,
samples with reactive HBsAg results should be tested with a licensed

neutralizing confirmatory test if recommended in the manufacturers


package insert.
8Persons positive only for anti-HBc are unlikely to be infectious except
under unusual circumstances in which they are the source for direct
percutaneous exposure of susceptible recipients to large quantities of
virus (e.g., blood transfusion or organ transplant).
9Milli-international units per milliliter.

Table 2-09. Recommended doses of currently licensed


formulations of hepatitis B vaccine
Single-Antigen
Vaccine

Combination Vaccine

REC
OM
BIV
AX
HB

ENG
ERI
X-B

CO
MV
AX1

PED
IARI
X2

TWI
NRI
X3

Infants
(<1
year)

Children
(1 - 10
years)

Adults
(>20
years)

Hemodia

Group

Adolesce
nts
1115
years
1119
years

Group

Single-Antigen
Vaccine

Combination Vaccine

REC
OM
BIV
AX
HB

ENG
ERI
X-B

CO
MV
AX1

PED
IARI
X2

TWI
NRI
X3

lysis
patients
and
other
immunoc
ompromi
sed
persons
<20
years7
20
years

Combined hepatitis BHaemophilus influenzae type b conjugate


vaccine. This vaccine cannot be administered before age 6 weeks or
after age 71 months.
2
Combined hepatitis Bdiphtheria, tetanus, pertussis-inactivated
poliovirus vaccine. This vaccine cannot be administered at birth, before
age 6 weeks, or after age 7 years.
3
Combined hepatitis A and hepatitis B vaccine. This vaccine is
recommended for persons 18 years who are at increased risk for both
hepatitis A virus and hepatitis B virus infections.
4
Recombinant hepatitis B surface antigen dose.
5
Not applicable.
6
Adult formulation administered on a 2-dose schedule.
7
Higher doses might be more immunogenic, but no specific
recommendations have been made.
8
Dialysis formulation administered on a 3-dose schedule at 0, 1, 6
months.
9
Two 1.0-mL doses administered at one site, on a 4-dose schedule at 0,
1, 2, and 6 months.

References and Additional Resources


CDC. A comprehensive immunization strategy to eliminate
transmission of hepatitis B virus infection in the United States:
recommendations of the Advisory Committee on Immunization
Practices (ACIP) Part II: immunization of adults. MMWR Recomm Rep.
2006;55(RR-16):125.
CDC. A comprehensive immunization strategy to eliminate
transmission of hepatitis B virus infection in the United States:
recommendations of the Advisory Committee on Immunization
Practices (ACIP) Part 1: immunization of infants, children, and
adolescents. MMWR Recomm Rep. 2005;54(RR-16):123.
Mast E, Goldstein S, Ward JL. Hepatitis B vaccine. In: Plotkin SA,
Orenstein WA, editors. Vaccines. 5th ed. Philadelphia: W.B. Saunders;
2004. p. 299337.
Simonsen L, Kane A, Lloyd J, et al. Unsafe injections in the developing
world and transmission of bloodborne pathogens: a review. Bull World
Health Organ. 1999;77(10):789800.
Sagliocca L, Stroffolini T, Amoroso P, et al. Risk factors for acute
hepatitis B: a casecontrol study. J Viral Hepat. 1997;4(1):636.
Lok AS, McMahon BJ. Practice Guidelines Committee, American
Association for the Study of Liver Diseases (AASLD). Chronic hepatitis
B: update of recommendations. Hepatology. 2004;39(3):85761.
CDC. Updated U.S. Public Health Service guidelines for the
management of occupational exposures to HBV, HCV and HIV and
recommendations for postexposure prophylaxis. MMWR Recomm Rep.
2001;50(RR-11):142.
Bock HL, Loscher T, Scheiermann N, et al. Accelerated schedule for
hepatitis B immunization. J Travel Med.1995;2(4):2137.
Long GE, Rickman LS. Infectious complications of tattoos. Clin Infect
Dis. 1994;18(4):6109. Review.
Mariano A, Mele A, Tosti ME, et al. Role of beauty treatment in the
spread of parenterally transmitted hepatitis viruses in Italy. J Med Virol.
2004;74(2):21620.
CDC. Provisional Recommendations for Hepatitis B vaccination of
adults October 2005. [cited 2006 Oct 31]. Available from:
www.cdc.gov/nip/recs/provisional_recs/hepB_adult.pdf (PDF).


REMOVABLE PROSTHODONTICS

Which of the following characteristics of a cast half-round RPD clasp would


make it more rigid?
a. Lengthening the arm
b. Using a cast round arm
c. Tapering the arm in 2 dimensions
d. Increasing the diameter

Which of the following characteristics of a cast half-round RPD clasp would


make it more rigid?
a.
b.
c.
d. Increasing the diameter

Retentive Clasp Arms

Factors that determine amount of retention a clasp can generate:


1.) size of the angle
2.) how far into the angle of cervical convergence
3.) flexibility of the clasp arm

Due to:
1) Length
2) Relative diameter (regardless of cross sectional form)
3) Cross-sectional shape (round, half-round, or other shape)
4) Material clasp is made of (all alloys): cast gold, cast chrome, wrought
gold, wrought chrome, titanium, or titanium (pure)

Clasps should bear a passive relationship to the teeth except when a


dislodging force is applied.

Retention on all principal abutments should be nearly equal as possible.

Clasp arms should be tapered (in 2 dimensions) from attachment point down
to point of engagement.

The longer the clasp arm: the more flexible

Greater diameter: less flexible

Round clasp- universally flexible in all directions

Most clasps are half round:

Half round less flexible in edgewise direction, but flexes as placed over
height of contour.

Retentive clasp arm on abutment adjacent to distal extension also must flex
during function on the distal base.

Clasp arms must disengage when vertical forces are placed toward tissues.

Gold alloys require more bulk for strength.


Wrought wire is tougher than cast metal.

Tensile strength of wrought wire 25% greater tan the cast metal.

Reciprocal Clasp Arm

Should be rigid.

Shape typically different than retentive arm.


Larger diameter than opposing retentive arm.
Tapered in 1 dimension (vs. 2 for retentive arm)

Ref.:

McGivney, G., Carr, A. McCrackens Removable Partial Prosthodontics.


th
10 ed. (2000) pp. 100-10

Which of the following impression materials is most hydrophilic?


a.
b.
c.
d.

Polyether
Polysulfide
Addition-reaction silicones
Condensation-reaction silicones

Which of the following impression materials is most hydrophilic?


a. Polyether
b.
c.
d.

Irreversible hydrocolloid (alginate), reversible hydrocolloid, and polyether are


hydrophilic.
Polysulfide, polyvinyl siloxanes (addition-reaction silicones), and condensationreaction silicones are hydrophobic.

Contact angle is the angle at which a liguid (dental stone) meets the solid
(impression).

A low contact angle (close to 0o) would be seen on a very hydrophilic


material: the liquid will more completely spread out on the surface and the contact
angle decreases the more the liquid spreads out.

Materials
Irreversible hydrocolloid (alginate), reversible hydrocolloid, and polyether are
hydrophilic and are the easiest to pour. Polysulfide, polyvinyl siloxanes, and
condensation-reaction silicones are hydrophobic, in ascending order, as indicated
by their high contact angles.

The greater contact angle, the greater the probability of air entrapment
during pouring.

Also, high contact angles more readily repel hemorrhage or other moisture in
the gingival sulcus.

Ref.:

http://www.ramehart.com/goniometers/contactangle.htm
Textbook: Shillingburg. Fundamentals of Fixed Prosthodontics. 3rd Ed. 1997.


Which of the following situations would be a contraindication for use of a
Lingualized occlusal scheme for complete dentures?

a. Poor neuromuscular control


b. Immediate Dentures
c. Overdentures
d. Moderate mandibular ridge resorption


Which of the following situations would be a contraindication for use of a
Lingualized occlusal scheme for Complete Dentures?

a. Poor neuromuscular control


b.
c.
d.

Lingualized Denture schemes can be utilized for most situations except flat
ridges, cases with excessive interarch distance, or poor neuromuscular control.

Non-anatomic, non-balanced occlusion (Monoplane Occlusion) Indications:

Class II malocclusion

Class III malocclusion

Severe residual ridge resorption

Excessive interarch distance

Poor neuromuscular skills

Poor patient adaptability

Reverse occlusal curve in existing dentures

Non-anatomic setup on existing dentures

Balanced Occlusion (Non-Anatomic Teeth) Indications:

Poor neuromuscular coordination

Cross-bite relationships

Class III malocclusion

Class II malocclusion

Those that posture forward or function greater than 4-5 mm anterior


to CR in protrusive

Lingualized Occlusion:

Virtually all cases except flat ridges or cases with excessive

interridge distance

Poor neuromuscular control is NOT indicated for lingualized


occlusion

Ref.:
Rahn. Textbook of Complete Dentures. Williams & Wilkins. P366-9

In what treatment stage would a RPD be completed?


a.
b.
c.
d.
e.

Urgent Phase
Disease Control Phase
Reevaluation Phase
Corrective Phase
Maintenance Phase

In what treatment stage would a RPD be completed?


a.
b.
c.
d. Corrective Phase
e.

Systemic phase:

Review medical history

Determine any modifications/contraindications to treatment

Urgent phase:

Address any painful conditions or pathology that requires prompt


attention

Disease control phase:

Address all active disease processes: e.g caries, endo pathology, perio
disease, etc.

Re-evaluation phase:

Do not progress to Restorative phase until all disease is controlled.

Restorative (Corrective) phase:

Restoration of dentition including all final restorations (e.g. crowns,


FDPs, RPDS, implants, dentures)

Maintenance phase:

Follows all completed treatment to ensure successful outcomes.


In a Kennedy Class II situation, how would you prevent rotation about the
fulcrum line?

a. Indirect retainer on the distal abutment tooth


b. Indirect retainer on a contralateral (from the edentulous area) canine
c. Indirect retainer on a ipsilateral (from the edentulous area) canine
d. Minor connector on an anterior tooth
e. Retentive arm on the distal abutment tooth


In a Kennedy Class II situation, how would you prevent rotation about the
fulcrum line?

a.
b. Indirect retainer on a contralateral (from the edentulous area) canine
c.
d.
e.

Axis of Rotation : (Think X,Y,Z axis or pitch, roll and yaw)


Three axes to be concerned with in RPD design and must be accommodated/
counteracted.

Fulcrum Line is a theoretical line around which an RPD tends to rotate.


b. The length of the fulcrum (lever arm) determines the amount of force
transferred to the abutment tooth..
c. Through the most distal abutment (rest), on either side of the arch, multiple
fulcrum lines can exist.

Rotation about the fulcrum line is resisted by the indirect retainer, which
should be positioned 90 degrees to the fulcrum line and as far away as possible,
usually on a premolar or canine. The axes of rotation possible in a Kennedy I/II RPD
are rotation about the fulcrum line, rotation around a longitudinal axis formed by
the crest of the ridge, and rotation around a vertical axis located near the center of
the arch.

Indirect retainer should be placed here!!!

Ref.:

McGivney, G., Carr, A. McCrackens Removable Partial Prosthodontics.


th
10 ed.

http://www.scielo.br/scielo.php?
script=sci_arttext&pid=S010364402008000300015&lng=en&nrm=iso&tlng=en


A maxillary arch missing #4,6,7,8,9,10,11 would be classified according to
Kennedy Classification and Applegates rules for classifying RPDs?

a. Kennedy Class 3, no modification


b. Kennedy Class 3, modification 1
c. Applegate Class 4, modification 1
d. Kennedy Class 4, modification 1
e. None of the above
f.


A maxillary arch missing #4,6,7,8,9,10,11 would be classified according to
Kennedy Classification and Applegates rules for classifying RPDs?

a.
b. Kennedy Class 3, modification 1
c.
d.
e.

Kennedy classified partially edentulous arches according to:


-Position
-Number of edentulous area

Purpose of Kennedy classification system.


- Immediate visualization of tooth lost.
- Immediate differentiation between tooth-borne and tissue supported RPDs.
- Communication among dentists and lab.
- Universally understood.

Kennedy(1925)-Applegate
Class I: Bilateral distal extension
Class II: A unilateral distal extension
Class III: A unilateral tooth supported
Class IV: Anterior edentulous span crossing the midline
Modification Space: additional edentulous areas (tooth bounded)

Applegates Rules for applying Kennedys Classification:


Rule 1: Classification should follow rather than precede any extraction of teeth that
might alter the original classification

Rule 2: If a third molar is missing and not to be replaced, it is not considered


in the classification.

Rule 3: If a third molar is present and is to be used as an abutment, it is


considered in the classification.

Rule 4: If a second molar is missing and is not to be replaced, it is not


considered in the classification (e.g., if the opposing second molar is likewise
missing and is not to be replaced).

Rule 5: The most posterior edentulous area or areas Always determines the
class.


Rule 6: Edentulous areas other than those determining the classification are
referred to as modifications and are designated by their number.

Rule 7: The extent of the modification is not considered, only the number of
additional edentulous areas.

Rule 8: There can be no modification areas in Class IV arches. (Other


edentulous areas lying posterior to the single bilateral areas crossing the midline
would instead determine the classification ).

Ref.:
Lovely, M. Review of Removable Partial Dentures. (2005) pp 164.65.

McGivney, G. McCrakens Removable Partial Prosthodontics. (2000) pp.


20-21

In a Kennedy Class III removable partial denture:


a.
b.
c.
d.

There
There
There
There

is
is
is
is

no
no
no
no

indirect retention
major connector
clasp assembly distal to the edentulous space
tooth-borne support

In a Kennedy Class III removable partial denture:


a. There is no indirect retention
b.
c.
d.

Kennedys classification of RPDs:

-class I: bilateral edentulous areas located posterior to the remaining natural


teeth.

-class II: a unilateral edentulous area located posterior to the remaining


natural teeth.

-class III: a unilateral edentulous area with natural teeth natural teeth
remaining both anterior and posterior to it.

-class IV: a single, but bilateral (crossing the midline), edentulous area
located anterior to the remaining natural teeth.

Factors to consider in RPD design:


- Preservation of teeth and peridental structures and maintenance of their health
in the long term.
- Minimal coverage of teeth and gingiva to help reduce plaque accumulation.
- The relative amounts of tooth-borne and tooth-mucosa-borne areas in the arch.
- Anatomic limitations like exostoses, undercuts, or anomalies.
- The inclination, position and contour of teeth.
- Contingency situations, like the loss of one or more teeth.
- The forces likely to be placed on the prosthesis by the patient.
- Ease of placement and removal based on patients mental status and dexterity.
- Esthetics.
- Desires of the patient.
- House classification (philosophical, exacting, indifferent, hysterical remember,
hysterical patients are NOT fit to wear dentures!)


class III RPD
design

Ref.:

Krol AJ, Jocobson TE, Finzen FC. Removable partial design: outline syllabus.
th
5 Ed. Indent. 1999.

Which of the following is an advantage of balanced occlusion with nonanatomic teeth:

a. Better penetration of food bolus


b. Superior esthetics
c. Can be used in cross-bite relationship
d. Easy occlusal adjustments

Which of the following is an advantage of balanced occlusion with nonanatomic teeth:

a.
b.
c. Can be used in cross-bite relationship
d.

NON-ANATOMIC, NON-BALANCED OCCLUSION (Monoplane Occlusion)

Indications:
- Class II malocclusion
- Class III malocclusion
- Severe residual ridge resorption
- Excessive interarch distance
- Poor neuromuscular skills
- Poor patient adaptability
- Reverse occlusal curve in existing dentures
- Non-anatomic setup on existing dentures

Advantages:
- More adaptable to unusual jaw relations
- Can be used in cross-bite cases
- The mandible does not get locked into one position
- Greater comfort and efficiency
- Improved denture stability
- Accommodates to changes in horizontal and vertical relations
-

Relining and rebasing are easier


Disadvantages:
Less efficient mastication
Esthetically inferior
Clogging of occlusal surfaces
Poor food penetration
Difficult to establish balanced occlusion

BALANCED OCCLUSION, Non-Anatomic Teeth


Indications:
Poor neuromuscular coordination
Cross-bite relationships
Class III malocclusion
Class II malocclusion

Those that posture forward or function greater than 4-5 mm anterior to CR in


protrusive
Disadvantages:
Compensating curve may cause same damaging effects as cuspal inclines
Occlusal adjustments are more difficult

Although non-anatomic teeth are used, it should not be considered a


monoplane occlusion.

BALANCED OCCLUSION, Anatomic Posterior Teeth

Advantages:
- Esthetics
- Better penetration of food bolus
- Anatomic occlusion arranged in harmony with muscles of mastication and
TMJ during functional and non-functional movements.

Disadvantages:
- Precise technique required for set-up
- Cuspal inclines tend to create greater lateral forces that can harm ridges
- More time is required to establish a balanced occlusion

LINGUALIZED OCCLUSION
Indications:
Virtually all cases except flat ridges or cases with excessive interridge
distance
Poor neuromuscular control is not indicated for lingualized occlusion
Advantages:
Better adapted to different types of ridges
Greater masticatory efficiency
Eliminates lateral interferences
Maintains esthetics and food penetration
Bilateral balance is possible
Disadvantages:
Less masticatory efficiency than balanced occlusion
Less resistance of denture base rotation than balanced occlusion
May result in increased lateral forces
More wear

The only contraindications to lingualized occlusion are flat ridges and


excessive inter-ridge distance

Which is not an advantage of alginate impression materials:


a. Good surface detail
b. Setting time dependent on operator handling
c. Hydrophilic
d. Elastic enough to be drawn over the undercuts

Which is not an advantage of alginate impression materials:


a.

b. Setting time dependent on operator handling


c.
d.

ALGINATES or irreversible hydrocolloids are sodium or potassium salts of


alginic acid and therefore water soluble.

Hydrocolloids are hydrophilic, and thus can be used in the presence of


moisture (saliva) and are less expensive than other impression materials.

They also pour well with stone, have a pleasant taste and odor, and are nontoxic and non-staining.

The setting reaction rate can be controlled by varying the temperature of the
mixing water.

To ensure accuracy, pouring should be completed within 15 min. after the


impression is removed from the mouth. For best results, the cast should be
separated 1 hour after pouring.
-

Properties:
Good surface detail

Reaction is faster at higher temperatures

Elastic enough to be drawn over the undercuts, but tears over the deep
undercuts

Not dimensionally stable on storing due to evaporation

Non toxic and non irritant

Setting time can depend on technique

Alginate powder is unstable on storage in presence of moisture or in warm


temperatures

Advantages:
Non toxic and non irritant

Good surface detail

Ease of use and mix

Cheap and good shelf life

Setting time can be controlled with temperature of water used

Problems/Disadvantages:
Poor dimensional stability

Incompatibility with some dental stones

Low tear strength


Less surface detail than other materials

Less dimensional stability

Setting time very dependent on operator handling

Must be poured immediately

Only one pour possible

Messy to work with

Ref.:

Rosentiel, Land, Fujimoto. Contemporary Fixed Prosthodontics. Elsevier.


2006.

Carr, et. al. McCrackens Removable Prosthodontics, 11th ed.

Which of the following impression techniques will result in the least fine
detail on the master cast?
a. Anatomic
b. Functional
c. Mucostatic
d. Selective pressure

Question may contain specifics: Kennedy class 2- which impression is


appropriate?

Which of the following impression techniques will result in the least fine
detail on the master cast?
a.
b. Functional
c.
d.

An ANATOMIC impression captures the hard tissues in an unloaded state;


unfortunately, it tends to distort and overextend soft tissue areas.

A denture base made to anatomic form exhibits less stability under rotating
forces, and fails to maintain its occlusal relation with opposing dentition.

A FUNCTIONAL impression records the shape of the patients mouth over a


period of time as developed by the patients own functional movements and
anatomy and is a static representation of the dynamic forces placed on the tray
and impression material. This impression lacks to the fine detail seen with other
techniques.

A MUCOSTATIC impression captures the fine details of the tissue using a


fast-set material in an unloaded state (ie: ZOE). It captures intimate tissue detail of
very healthy, unloaded keratinized tissues. Tray contains free-flowing, quick
setting, and rigid setting impression material.

A SELECTIVE PRESSURE technique is preferred in many cases as it allows


the primary denture bearing areas (posterior ridges and tuberosities for the
maxilla, buccal shelves and retromolar pad areas in the mandible) to be selectively
loaded while the secondary supporting areas are not subjected to as much force.
The secondary supporting areas are the anterior ridge and midline raphe in the
maxilla and anterior and posterior ridges in the mandible. This helps to simulate
tissue conditions under function and helps the final prosthesis to fit better.

It captures primary denture bearing areas in a pre-loaded and pressurized


state. Soft tissues are captured anatomically with minimal overextension.

Ref.:

MacCrackens Removable Partial Prosthodontics, 10th ed .

Krol AJ, Jocobson TE, Finzen FC. Removable partial design: outline syllabus.
th
5 Ed. Indent. 1999.

Felton DA, Cooper LE, Scurria MS. Predictable impression procedures for
complete dentures. Dent Clin N Amer Jan 1996; 40(1): 39-51.

The part of a removable partial denture that assists the direct retainers in
preventing displacement of distal extension bases by functioning as part of the
resistance arm of a lever anterior to the fulcrum line is termed a (an)

a.
reciprocal clasp arm

b.
indirect retainer

c.
direct retainer

d.
guiding plane

The part of a removable partial denture that assists the direct retainers in
preventing displacement of distal extension bases by functioning as part of the
resistance arm of a lever anterior to the fulcrum line is termed a (an)

a.

b.
indirect retainer

c.

d.

First-class levers

First class lever


A first-class lever is a lever in which the
fulcrum is located between the input

effort and the output load. In operation, a


force is applied (by pulling or pushing) to
a section of the bar, which causes the
lever to swing about the fulcrum,
overcoming the resistance force on the
opposite side. The fulcrum may be at the
center point of the lever as in a seesaw
or at any point between the input and
output.

Examples: Seesaw (also known as a teeter-totter), Hammer Claw (when pulling a


nail with the hammer's claw), Hand trucks (are L-shaped but work on the same
principle, with the axis as a fulcrum), Scissors (double lever), Wheel and axle
(because the wheel's motions follows the fulcrum, load arm, and effort arm
principle), Chopsticks (with hand the middle finger acts as a pivot. The whole
system is a double lever).
Class I: (the see-saw) resistance fulcrum effort
KENNEDY CLASS II RPD: resistance is on the contralateral side of the arch from the
fulcrum and effort
-resistance is the clasp
-fulcrum is the terminal rest
-effort is downward force on the distal extension
-like KII, where

Second-class levers

Second class lever


In a second class lever the input effort is
located at the end of the bar and the
fulcrum is located at the other end of the
bar, opposite to the input, with the
output load at a point between these two
forces.

1. Examples: Dental elevator , Nutcracker, Paddle, Wheelbarrow, Bottle opener


2.
Class II: (the wheelbarrow) fulcrum resistance effort
3.
KENNEDY CLASS I RPD: effort is upward force on the distal extension,
otherwise the clasp would disengage
4.
5.

Third-class levers

different from second-class levers and


some first-class levers. However, the
distance moved by the resistance (load)
is greater than the distance moved by
the effort. Since this motion occurs in the
same length of time, the resistance
Third class lever. For the lever in this
necessarily moves faster than the effort.
diagram to work correctly, one must
Thus, a third-class lever still has its uses
assume that the fulcrum is attached to in making certain tasks easier to do. In
the bar or acting in opposition to the
third class levers, effort is applied
other two forces.
between the output load on one end and
For this class of levers, the input effort is the fulcrum on the opposite end.
higher than the output load, which is

Examples: Baseball bat, Broom, Fishing rod, Hockey stick, Mandible, Stapler, Tongs,
Tweezers, Hammer, Tennis racket
Class III: (the fishing pole) fulcrum effort resistance
DOES NOT OCCUR IN RPD DESIGN
-TMJ muscles and teeth act as a class III lever
-cantilever: when fulcrum and resistance are connected on the same body
An RPD can act as an:
inclined plane, if not properly supported by well defined and executed rests &
tissue support
a fulcrum, the length of the fulcrum (lever arm) determines the amount of force
transferred to the abutment tooth

All of
a.
b.
c.

the following describe a reciprocal clasp arm, except


it is placed above the height of contour
it is a rigid clasp arm
it must contact the tooth after the retentive clasp arm passes over the height
of contour
d. it must contact the tooth before the retentive arm passes over the height of
contour
e. ideal placement should be at the junction of the gingival and middle third of
the abutment tooth crown

Which is an advantage of the RPA clasp design when compared to an RPI design:
a. Avoids problems associated with large tissue undercuts.
b. The circumferential-type retentive arm is significantly more retentive than
the infrabulge retentive arm.
c. Simple in design, provides better esthetics.
d. When designed with a distal rest, it causes less distal displacement of the
primary abutment tooth.

Which is not an advantage of the RPI clasp design:


a. I bar more esthetic
b. Proximal plate and I bar move away from the tooth during function to reduce
torque
c. Infrabulge contact provides more contact of the retentive arm to the tooth
surface
d. Mesial minor connector and proximal plate provide reciprocation and
eliminate need for a lingual arm
e. Mesial rest eliminates class I lever

Which is not a requirement of a properly designed clasp assembly:

a. Support - against vertical forces


b. Retention - resist forces in a occlusal direction
c. Encirclement - of more than half its abutment tooth circumference
d. Elasticity flex with equal and opposite forces as the occlusion forces
e. Passivity - at rest when seated

All of the following describe a reciprocal clasp arm, except


a.
b.
c. it must contact the tooth after the retentive clasp arm passes over the height
of contour
d.
e.

Which is an advantage of the RPA clasp design when compared to an RPI design:
a. Avoids problems associated with large tissue undercuts.
b.
c.
d.

Which is not an advantage of the RPI clasp design:


a.
b.
c. Infrabulge contact provides more contact of the retentive arm to the tooth
surface
d.
e.

Which is not a requirement of a properly designed clasp assembly:


a.
b.
c.

d. Elasticity flex with equal and opposite forces as the occlusion forces
e.

A DIRECT RETAINER (clasp assembly) is a unit of a removable partial denture that


engages an abutment tooth in such a manner as to resist displacement of the
prosthesis away from basal seat tissues.
It is usually composed of
a retentive arm,
a reciprocal (bracing) element or arm,
a rest and
a minor connector
Retention is derived by placing a clasp arm into an undercut area so that it is
forced to deform
upon vertical dislodgment. Resistance of the clasp to deformation generates
retention.
Resistance is proportionate to the flexibility of the clasp arm. Non-flexible
portions of clasp
arms must be placed occlusal to the height of contour (suprabulge area).
REQUIREMENTS OF DIRECT RETAINERS
All clasp assemblies should meet the following requirements:
Support - resistance to gingival displacement (occlusal rests)
Reciprocity - resistance to orthodontic movement of teeth using reciprocal
arms or elements placed against guiding planes. During placement and
removal of the partial denture the retentive arm flexes over the height of
contour and generates energy. At this point the rigid reciprocal arm should
contact the guiding plane and prevent orthodontic movement from taking
place.
Stability - resistance to lateral movement (reciprocal arms, minor
connectors)
Retention - retentive arms located in undercuts on the abutments

Encirclement of greater than 180 of the tooth - prevents the prosthesis


from moving away from the tooth

Passivity - at rest, a direct retainer should not exert force against a tooth
Wherever possible direct retainers should be selected to fit the existing teeth. This
is preferable to preparing teeth to fit a particular clasp design. It may be possible
to select a different clasp design to meet the retentive requirements for a partial
denture. Nonetheless, judicious tooth preparation should not be avoided at all
costs, since it can immeasurably improve prosthesis biomechanics.
CIRCUMFERENTIAL (CIRCLE OR AKERS) CLASP

the most simple and versatile clasp (clasp of choice in tooth-borne cases)
clasp assembly has one retentive arm opposed by a reciprocal arm
originating from the rest
the retentive arm begins above the height of contour, and curves and tapers
to its terminal tip, in the gingival 1/3 of the tooth, well away from the gingiva
the bracing arm is in the middle 1/3 of the tooth, and is broader occlusogingivally, does not taper and is either entirely above the height of contour
or completely on a prepared guiding plane it should never be designed into
an undercut, as it is a rigid element.

Advantages:

- Excellent bracing qualities

- Easy to design and construct


- Less potential for food accumulation
below the clasp compared to bar clasps
Disadvantages:

- More tooth coverage than bar clasps


- More metal is displayed than with bar or
combination clasps
- Adjustments are difficult or impossible
due to the half round nature of the clasp

RING CLASP

Encircles nearly the entire abutment tooth


Usually used with mesially and lingually tilted mandibular molars (with a m-l
undercut) or mesially and buccally tilted maxillary molars (with a m-b
undercut)
The undercut is on the same side as the rest seat (i.e. adjacent to edentulous
span)
Use a cast circumferential clasp with lingual retention and buccal bracing, in
preference to a ring clasp whenever possible, unless a severe tilt of the tooth
will not permit

Advantages:
- Excellent bracing
- Allows use of an available undercut
adjacent to edentulous area

Disadvantages:
EMBRASURE (DOUBLE AKERS) CLASP

Covers a large area of tooth


surface, therefore requiring
meticulous hygiene
Very difficult to adjust due to the
extreme rigidity of the reciprocal
arms
The lower bracing arm should be at
least 1 mm from the free gingival
margin and
relieved to prevent impingement of
the gingival tissues.

Used in a quadrant where no edentulous area exists, or where a distal


approach clasp cannot be used on the most posterior tooth (i.e. No usable
retentive undercut).
Two rests, two retentive arms, and two bracing arms
Double rests with definite shoulders to prevent weakening of clasp arms,
separation of teeth and food impaction
Buccal and lingual proximal areas must be opened (i.e. Blend with axial
contours, reduce height of contours, round occluso-axial line angles)
Use minimum retention prone to distortion
Use with discretion use another clasp if possible

Advantages:
Disadvantages:
- Allows placement of direct retainer
- Extensive interproximal reduction
where none could otherwise be placed
is usually required
(especially
- Covers large area of tooth surface - contralateral to the edentulous span
hygiene considerations
on a Class II case)

"C" CLASP (HAIR-PIN OR REVERSE ACTION)

The retentive area (undercut) is adjacent the occlusal rest.


The upper arm is a minor a connector giving rise to tapered lower arm.

Advantages:
- Allows use of undercut adjacent to
edentulous space
Disadvantages:
- Almost impossible to adjust
- Non-esthetic

Difficult to fabricate so the upper


portion of the retentive arm clears the
opposing
occlusion
Covers extensive tooth surface and
acts as a food trap
Insufficient flexibility on short crowns
due to insufficient clasp arm length

Cast suprabulge clasps should be used in most tooth borne cases. Exceptions
to this rule include:
Esthetic concerns. Since wrought-wire clasps can be placed into
greater undercuts (0.02") than cast clasps (0.01") they can be placed
lower on teeth, allowing better esthetics in some cases. Infrabulge
clasps are also less visible.
Where a posterior abutment is mobile or of questionable prognosis, the
treatment plan could call for the use of the stress-breaking qualities of
a wrought clasp on the anterior abutment. This would allow the
prosthesis to be converted into a distal extension type if the weak
posterior abutment should be lost.
Where abutments are mobile, the tooth borne segment is extensive,
the use of the stress-breaking clasps should be considered.
Stress releasing clasp assemblies include:
- the bar clasp with mesial rest (e.g. RPI)
- the RPA clasp
- the combination clasp
BAR CLASPS

The bar clasp is a cast clasp that arises from the partial denture
framework and approaches the retentive undercut from gingival direction
(as opposed to a circumferential clasp that approaches the undercut from
the occlusal direction).

Retentive clasps are identified by shape of retentive terminal, i.e. T, Y, L, I,


U, and S.
- The shape is unimportant as long as the direct retainer is mechanically
and functionally stable, covers minimal tooth structure with minimum
display (the I bar most often meets these requirements)
- T-and Y-shaped terminal ends are the most misused clasps. The full area
coverage of the T and Y terminal ends is rarely necessary for adequate
retention.
- L-shaped clasp is same as an I clasp with a longer horizontal component.
The terminal is a double I clasp.
- The S-shaped terminal end is used to avoid a mesial soft tissue undercut.
- Soft tissue relief is provided under the approach arm with 28 or 30 gauge
wax, to prevent tissue impingement
Contraindications:
- deep cervical undercuts - food trap or impingements result
- severe soft tissue or bony undercuts - food trap or impingements result
- insufficient vestibular depth for approach arm (requires 4 mm - 3 mm
from free gingival margin, 1 mm for thickness of the approach arm)
- pronounced frenal attachments in area - impingement

THE R-P-I CLASP

The components of this clasp assembly are:


"R" - rest (always mesial)
"P" - proximal plate
"I" - I-bar (retentive arm)
The rest is located on the mesio-occlusal surface of a premolar or
mesiolingual surface of a canine.
The minor connector is located in the mesio-lingual embrasure but is not in
contact with the adjacent tooth (prevents wedging).
The proximal plate (essentially a wide minor connector) is located on a guide
plane on the distal surface of the tooth. The superior edge of the proximal

plate is located at the bottom of the guide plane (at approximately the
junction of the occlusal and middle third of the guide plane).
The I-bar clasp is located on the buccal surface of the premolar and on the
mesio-buccal surface of the canine. The I-bar originates at the gridwork and
approaches the tooth from the gingival direction. The bend in the I-bar
should be located at least 3 mm. from the gingival margin. This distance will
prevent food entrapment and provide the length for the necessary flexibility
in the clasp arm. The clasp is usually cast and is placed just below the height
of contour line.
The guiding plane is a parallel surface prepared on the occlusal one third of
the distal surface of the tooth. The guiding plane extends lingually enough so
that, along with the mesial rest, it can prevent lingual migration of the tooth.
It is approximately 2 to 3 mm in height.
Contraindications to the R.P.I. Clasp:
- Insufficient depth of the vestibule. (The inferior border of the I-bar must
be located at least 4 mm. from the gingival margin.)
- No labial or buccal undercut on the abutment
- Severe soft tissue undercut
- Disto-buccal undercut (less than 180 encirclement)

RPA CLASP

This clasp assembly is similar to the RPI design except a wrought wire
circumferential clasp (Akers) is used instead of the I-bar. This clasp arises
from the proximal plate and terminates in the mesiobuccal undercut. It is
used when there is insufficient vestibule depth or when a severe tissue
undercut exists.

COMBINATION CLASP

The combination clasp is similar to the cast circumferential clasp with the
exception that the retentive arm is fabricated from a round wrought wire
(platinum-gold-palladium alloy or chromecobalt alloy).
- a cast reciprocal arm
- the wrought wire is flexible (round form)
- more adjustable than cast or 1/2 round forms
- better esthetics (due to its round form and smaller diameter - 18 gauge)
- can be used with a mesial or buccal undercut
- can be placed in 0.02" undercut due to its flexibility (allows lower
placement for better esthetics)
- can be used in tooth borne cases

Ref.:
Krol A.J.: Clasp Design for Extension Base Removable Partial Dentures. J
Prosthet Dent 29:408-415, 1973.
Demer W.J.: An Analysis of Mesial Rest-I-Bar Clasp Designs. J Prosthet Dent
36:243-253, 1976.
Eliason, C.: RPA Clasp Design for Distal Extension Removable Partial
Dentures. J Prosthet Dent 49:25,1983.
Robert W. Loney: Removable Partial Denture Manual2008

Of the following cases, which does not require extra-oral occlusal adjustment
on delivery as opposed to intraoral adjustment.
a. Tooth supported RPD without a soft tissue component
b. Long distal extension cases
c. Extension bases on mobile soft tissue
d. RPD opposing a complete denture

Of the following cases, which does not require extra-oral occlusal adjustment
on delivery as opposed to intraoral adjustment.
a. Tooth supported RPD without a soft tissue component
b.
c.
d.
The occlusion on a Kennedy Class I RPD should not interfere with the vertical
dimension of occlusion or interfere with eccentric movements. Choices b, c,
and d are cases where the denture and stability are compromised and should
be adjusted and optimized extra-orally on a semi-adjustable articulator.
Pick up impressions using alginate of the arch (es) are made and
poured and trimmed.

A facebow transfer or facebow transfer mounting preservation is used


to mount the maxillary cast.

The mandibular cast is mounted using jaw relation records.

The mounting should be verified

Occlusion should be adjusted to maintain VDO and not mutilate the


denture teeth

Eccentric interferences are adjusted away

The anatomy of the denture teeth is refined and polished.

The RPD(s) are inserted and the bite and eccentric movements are
evaluated.

Ref.
Phoenix, RD, Cagna, DR, DeFreest, CD; Stewarts Clinical Removable Partial
Prosthodontics; Quintessence Publishing Co.; Chicago, IL; 2003; pp. 437-441.

Which of the following materials is a tissue conditioner?


a. Coe-Comfort
b. Viscogel
c. Coe-Soft
d. Lynal

Which of the following materials is a tissue conditioner?


a.
b.
c. Coe-Soft
d.

The difference between a soft liner and a tissue conditioner is the viscosity;
you will see more flow, change and adaptation to the tissues in a less viscous
material (a tissue conditioner). Coe-Comfort, viscogel and lynal have the
highest viscosity and are recommended for use as soft-liners. Coe-soft and
Softone are recommended for use as tissue conditioners because they flow
easily and have a low viscosity.
The use of resilient liners is designed to distribute functional and
nonfunctional stresses more evenly and to have a dampening effect because
of elastic behavior. These properties make resilient liners useful for treating
patients with atrophic ridge or resorption, bony undercuts, bruxism,
congenital or acquired oral defects requiring obturation, xerostomia and
dentures opposing natural teeth. Commercially available products include
rubbery acrylic-, silicone-, fluoric- and olefin-type materials.
Ref.
Wright PS The success and failure of denture soft lining materials in clinical
use. J Dent. 1984;12(4):319-7.
Dootz ER, Koran A, Craig RG. Physical property comparison of 11 soft denture
lining materials as a function of accelerated aging. J Prosthet Dent.
1993;69(1):114-9

Category II rotational path RPD design:


a. Requires rest be seated simultaneously
b. Uses rotational centers located at the end of long occlusal rest
c. Is commonly used for anterior tooth replacement
d. Is commonly applied for posterior tooth replacement
e. Incorporates a pure rotational path of placement

Category II rotational path RPD design:


a.
b.
c. Is commonly used for anterior tooth replacement
d.
e.

A conventional RPD that uses a straight path of placement requires that all
rests be seated
simultaneously. The incorporation of a dual, curved, or rotational path of
placement permits
one portion of the framework to be seated first, followed by the remainder of
the framework.
This procedure usually permits a reduction in the number of clasps in the
framework without
compromising the biomechanical properties of the prosthesis. Rotational
path designs that
eliminate clasps reduce tooth coverage by components and often enhance
esthetics. There are 3 types- Anteroposterior (AP) where the ant segment is
seated first, Posteroanterior (PA), and Lateral where the edentulous side is
seated first. There are two categories:
Category I
Posterior tooth replacement - bilaterally, pure rotational path removable
partial denture. Can
be either anterior-posterior (AP) which is used in the maxillary arch for
esthetic reasons, or
posterior-anterior (PA) used to great advantage when mandibular molars are
tipped mesially.
Uses rotational centers that are located at the end of long occlusal rests.
Rotational centers are
seated first, then the remainder of the framework is rotated to seat the
conventional clasp
portion.
Category II
Both the rotational centers and the rigid retentive elements are located at
the gingival
extensions of minor connectors. Incorporates a dual path of placement. The
initial straight
path of placement serves to seat the rotational centers. A curved or
rotational path then allows

the rests and remainder of the framework to be seated. The most common
application of the
Category II design involves the replacement of missing anterior teeth with
elimination of
anterior clasps.
Advantages of Rotational Paths
1. Minimizes number of clasps, reducing tooth coverage
2. May reduce plaque accumulation.
3. Anterior clasps may often be eliminated, improving esthetics.
4. May be used in preference to an anterior fixed prosthesis to attain better
esthetics.
5. Minimal tooth preparation when compared to a precision attachment or
fixed prosthesis.
6. May often be used in absence of lingual or facial undercuts.
7. Distortion of rigid retentive components is unlikely.
8. May prevent further tipping of abutment teeth contacted by rigid retainer.
Disadvantage of the Rotational Paths
1. Adjustment of the rigid retentive component is difficult.
2. Less tolerance for error.
3. Requires well prepared rest seats.
Ref.
Jacobsen, T. E. , Krol, A. J. Rotational Path Removable Partial Design. Journal
of Prosthetic Dentistry. Vol. 48 No.4 October 1982. p 370-376.
Krol, A. J., Jacobsen, T.E., Removable Partial Denture Design, 4th ed. Indent
Publisher. 1990 pg. 69-88.

Anterior teeth protect the posterior teeth in all mandibular excursions and
the posterior teeth protect the anterior teeth at the intercuspal position. This
statement describes:
a. bilateral balanced occlusion
b. unilateral balanced occlusion or group function
c. mutually protected occlusion
d. three-point contact balanced occlusion

Anterior teeth protect the posterior teeth in all mandibular excursions and
the posterior teeth protect the anterior teeth at the intercuspal position. This
statement describes:
a.
b.
c. mutually protected occlusion
d.

Mutually protected occlusion is also known as canine protected occlusion.


According to this concept of occlusion, the anterior teeth bear all of the load
and the posterior teeth are disoccluded in any excursive position of the
mandible. The desired result is an absence of frictional wear.
The position of maximum intercuspation coincides with the optimal
condylar position of the mandible. All posterior teeth are in contact with the
forces being directed along their long axes. The anterior teeth either in
contact lightly or are very slightly out of contact, relieving them of the
obliquely directed forces that would be the result of anterior tooth contact.
As a result of the anterior teeth contact protecting the posterior teeth in all
mandibular excursions and the posterior teeth protecting the anterior teeth
at the intercuspal position, this type of occlusion became known as a
mutually protected occlusion. This arrangement of the occlusion is probably
the most widely accepted because of its ease of fabrication and greater
tolerance by patients.
In order to prosthodontically reconstruct with mutually protected
occlusion, it is necessary to have anterior teeth that are periodontally
healthy. In the presence of anterior bone loss or missing canines, the mouth
should probably be restored to group function (unilateral balance). The
added support of the posterior teeth on the working side will distribute the
load that the anterior teeth may not be able to bear.
The use of mutually protected occlusion is dependent upon the
orthodontic relationship of opposing arches. A mutually protected occlusion
cannot be used in the situation of reverse occlusion, or cross bite, in which
the maxillary and mandibular buccal cusps interfere with each other in a
working side excursion.
In bilateral balanced occlusion a maximum number of teeth contact in all
excursive positions of the mandible.
In unilateral balanced occlusion (group function) all teeth on the working side
are in contact during a lateral excursion, and the teeth on the nonworking
side are free of any contact.
Ref.:

Shillingburg H. T et al. Fundamentals of Fixed Prosthodontics 3rd Ed.,


Quintessence 1997. Contemporary Fixed Prosthodontics, 2nd Ed. Rosenstiel
et al.

Which of the following is not a sign seen in the Ellsworth-Kelly Combination


Syndrome:
a. papillary hyperplasia
b. downgrowth of the maxillary tuberosities
c. supra-eruption of the lower anterior teeth
d. candida infection of the hard palate following the outline of the RPD
e. ridge resorption of posterior mandible

Which of the following is not a sign seen in the Ellsworth-Kelly Combination


Syndrome:
a.
b.
c.
d. candida infection of the hard palate following the outline of the RPD
e.

The Glossary of Prosthodontic Terms1 defines combination syndrome as the


characteristic features that occur when an edentulous maxilla is opposed by
natural mandibular anterior teeth, also called anterior hyperfunction
syndrome.
Combination syndrome is often seen in patients with a maxillary complete
denture and a mandibular bilateral distal-extension partial denture. This
syndrome described by Kelly in 1972 consists of
(1) loss of bone from the anterior part of the maxillary ridge,
(2) Downgrowth of the maxillary tuberosities,
(3) papillary hyperplasia of the tissues of the hard palate,
(4) extrusion of the lower anterior teeth, and
(5) the loss of bone beneath the removable partial denture bases.
In addition, the following have been added as a subset to the classic signs
listed above: loss of vertical dimension of occlusion, occlusal plane
discrepancy, anterior spatial repositioning of the mandible, poor adaptation
of the prostheses, epulis fissuratum, and periodontal changes. However,
these changes are not generally associated with combination syndrome.
Candidiasis is an overgrowth of Candida albicans, an organism in the normal
flora. Although this may be seen in elderly patients wearing complete
dentures, it is due to a decrease in their resistance to infection, not the result
of combination syndrome.
Papillary hyperplasia is an overgrowth of tissue due to a source of
inflammation. The palate is the most common site. In combination
syndrome this is caused by the ever-increasing poor fit of the complete
denture.
Emulous fissured (redundant growth of tissue under a denture flange) is also
often seen with combination syndrome. In addition, the loss of vertical
dimension of occlusion, occlusal plane discrepancy, anterior spatial
repositioning of the mandible, poor adaptation of the prostheses, and
periodontal changes are often noted with this syndrome.
Treatment considerations for patients who are completely edentulous in the
maxillary arch and have mandibular anterior teeth remaining must consider
the prevention of combination syndrome. One must discourage excessive

occlusal pressures in the maxillary anterior region in both centric and


eccentric occlusal contacts. The design of the mandibular removable partial
denture should incorporate positive occlusal support of the mandibular
anterior as well as maximal coverage of the basal seat beneath the distalextension bases. The rigid portions of the RPD should function to minimize
movement in lateral, rotational, and anteroposterior directions. The
maxillary complete denture must have excellent retention, proper vertical
dimension and centric relation position. There should be no incisal contact of
the anterior teeth in centric position and minimal contact in eccentric
positions as long as the posterior teeth can maintain contact. Balanced
occlusion should be developed with the posterior teeth by using the proper
cuspal angulation in conjunction with the condylar and incisal guidances.
Patient education and frequent recall and maintenance care for these
patients are essential to avoid combination syndrome.
Ref.:
Sanders TR, Gillis RE, Discarding RP (1979), The maxillary complete denture
opposing the mandibular bilateral distal-extension partial denture: Treatment
considerations Journal of Prosthetic Dentistry 41(2):124-128 and Chaska SN
Synopsis of Oral Pathology 6th Ed. Mosby)
Palmqvist S, Carlsson GE, Owall B. The combination syndrome: a literature
review. J Prosthet Dent. 2003 Sep;90(3):270-5.

All of the following are positive characteristics of a crown and bridge


impression material except:
a. Hydrophilic
b. High wetting angle
c. High detail reproduction
d. Moderate rigidity
e. Low contact angle

All of the following are positive characteristics of a crown and bridge


impression material except:
a.
b. High wetting angle
c.
d.
e.

Definitions of the terms used to describe characteristics of


impression materials.
A. Hydrophilic: (water loving) The relative ability of an impression material
to tolerate
moisture in an impression site and still provide an accurate impression. A
material that will
tolerate a modest amount of moisture is considered hydrophilic.
Hydrocolloids are the most
hydrophilic of the materials listed below.
B. Hydrophobic: (water fearing). An impression material with a very low
ability to tolerate
moisture in an impression site and still provides an accurate impression.
Despite
manufacturer claims, all polyvinyl siloxane materials are inherently
hydrophobic.
C. Wetting Angle: the relative ability of an impression material to flow into
small crevices.
This is really defined by the materials desire to stick to itself.
1. High Wetting Angle: Materials like Permadyne (polyether) tray material
have a greater desire to
stick to themselves, than flow onto another surface. Therefore, Permadyne
tray material
would have a very high wetting angle, a poor characteristic if it were to be
used by itself as
a crown and bridge impression material.
2. Low Wetting Angle: Water is the best example of a material with a low
wetting angle. It
has very little desire to stick to itself and would rather run into every nook
and cranny. This

is a very good characteristic for a syringable material, but would be a


problem for a tray
material.
D. Detail Reproduction: the ability of an impression material to accurately
reproduce
small detail. Crown and bridge impression materials require high detail
reproduction. These
materials are able to reproduce details as fine as 20-30 microns. Impression
materials with
moderate detail reproduction are able to reproduce details as fine as 100150 microns.
Impression materials with moderate detail reproduction are quite acceptable
for removable
prosthodontics. With the exception of alginates and methacrylates, all
impression materials
listed below have an excellent ability to reproduce details. Alginates and
methacrylates have
a moderate ability to reproduce fine detail.
E. Dimensional Stability: The ability of an impression material to maintain
the accuracy of
the impression over time. Practically, an impression from a material with high
dimensional
stability can be poured several weeks later and still produce an accurate
model. Polyvinyl
siloxanes are the most dimensionally stable of the commonly used materials.
Alginates and
hydrocolloids have low dimensional stability.
F. Rigidity: the relative stiffness of the material after setting. A material with
high rigidity
needs less support from a tray to prevent distortion on pouring; makes a
good jaw relation
material. Rigid impression materials are excellent for implant impressions
where removable
impression posts must accurately transfer the spacial position of the implant.
Rigid
impression materials will also be more difficult to remove in a full arch
impression of
periodontally involved teeth. An example of a rigid impression material is
Permadyne tray
material, a polyether.
G. Tear Strength: the relative ability of a material to resist tearing after
setting, even in

thin sections. This is important in capturing an accurate impression of


subgingival margins
where the material flows into thin sulcular spaces. Polyether has the highest
tear strength of
the commonly used materials. Hydrocolloid has low tear strength.
H. Taste: a category that becomes very important to your patients. Recently
Discus Dental
has introduced Berry and chocolate flavored polyvinyl siloxane. Rubber base
and polyether
still take worst of show in the taste category.
I. Cost: Simply put, you want the least expensive impression material that
will give you
consistently good results. There are two costs you must consider: initial cost
and the cost of
retakes. If you consider how much chair time is worth in your office, you will
understand
that the impression material with the least initial cost would be the most
expensive if you
had to retake a significant number of impressions. The relative cost data
provided is initial
cost.
J. Disinfection: the relative ease that an impression material can be
disinfected after
setting without risk of distortion. Polyvinyls are virtually impervious to all
disinfection
protocols. Hydrocolloids, Alginates, Polyethers, and Methacrylates all have
specific
disinfection protocols, which if not adhered to will cause distortion of the
impression.
K. Contact Angle: the relative ability of the set impression material to have
another
material (dental stone) flow over its surface. A low contact angle would mean
that stone
flows easily and bubble free models were relatively easy to produce. A
material with a high
contact angle would need more attention to produce an excellent model. All
the materials
listed below have relatively low contact angles except polyvinyl siloxane.
Surfactants are
available in spray form to lower the contact angle of any impression material.

Characteristics of common impression materials suitable for taking


impressions for complete dentures, removable partial dentures,
crowns and bridges.
( + positive characteristic. - negative characteristic.)
Hydrocolloid (Rubberloid)
+ Hydrophilic: high. Will still capture accurate impression in the presence of
a little blood
or saliva.
+ Wetting angle: low. Easier to capture a perfect full arch impression with
than with
polyvinyls.
+ Detail reproduction: excellent.
- Dimensional stability: poor. Must be poured immediately unless stored in
K2SO4.
Absolutely only one accurate pour per impression.
+/- Rigidity: low. Impressions more easy to remove than polyvinyls or
polyethers
Hydrocolloids must have rigid trays. Not suitable for triple trays.
- Tear strength: low. Will capture a subgingival impression well but has
tendency to tear
on removal. Not as strong as polyethers or polyvinyls.
+ Taste: neutral. Much better than polyether or polysulfide.
+ Cost: low
- Disinfection: difficult. High risk of distortion from disinfection if not
performed correctly.
Due to hydrophilic nature, tend to swell when immersed in water or
disinfectant. Protocol:
spray with 2% gluteraldehyde, place in plastic bag, and time for 10 minutes.
Rinse
immediately. Soak in K2SO4
+ Comment: Setting reaction not contaminated by latex proteins from
rubber gloves.
- Comment: will not adhere to itself. Cannot be used to border mold and for
correctable
impression technique.
- Comment: requires special equipment and trays.
Alginate (Jeltrate, Kromoscop)
+ Hydrophilic: high. Will still capture accurate impression in the presence of
a little blood
or saliva.

+ Wetting angle: low. Easier to capture a perfect full arch impression with
than with
polyvinyls.
- Detail reproduction: moderate. Not accurate enough for crown and
bridge. Suitable for
primary impressions and framework impressions for partial dentures when
altered cast
technique is employed.
- Dimensional stability: poor. Must be poured immediately. (Maximum 10minute delay or
distortion will occur.) Absolutely only one accurate pour per impression.
- Rigidity: low. Impressions more easy to remove than polyvinyls or
polyethers. Alginates
must have rigid trays or they will distort. Not suitable for triple trays.
- Tear strength: low. Will capture an impression well but has tendency to
tear on removal.
Not as strong as polyethers or polyvinyls.
+ Taste: neutral, which makes it much better than polyether or polysulfide.
+ Cost: low range
- Disinfection: difficult. High risk of distortion from disinfection if not
performed correctly.
Due to hydrophilic nature, tend to swell when immersed in water or
disinfectant. Protocol:
spray with Lysol and time for 3 minutes or spray with 2% gluteraldehyde and
time for 10
minutes and place in plastic bag. Rinse immediately and pour.
+ Comment: Setting reaction not contaminated by latex proteins from
rubber gloves.
- Comment: will not adhere to itself. Cannot be used to border mold and for
correctable
impression technique.
- Comment: although the most frequently used impression material for
complete and
partial dentures, it is virtually impossible to accurately border mold alginate
impressions.
This difficulty is due to the viscosity of the material, the tendency of stock
trays to distort
tissues or fail to adequately support impression material, and relatively short
working time.
Alginate is an excellent primary impression material or a final impression
material for partial
denture framework.
Polysulfide (rubber base) (Permalastic)

+ Hydrophilic: low to moderate. Will still capture accurate impression in the


presence of a
little blood or saliva.
+ Wetting angle: low. Easier to capture a perfect full arch impression with
than with
polyvinyls.
+ Detail reproduction: excellent.
0 Dimensional stability: fair. May allow two pours of accurate casts if no
thin subgingival
margins present.
+/- Rigidity: low to moderate depending on viscosity of material selected..
Impressions
easier to remove than polyethers. Polysulfides are not suitable for triple
trays.
+ Tear strength: medium. Will capture a subgingival impression, usually
without tearing
on removal. Much better than hydrocolloids; better than polyvinyls.
- Taste: bitter. Polysufides fall in to the category of You know it has to be
good because it
tastes so bad.
+ Cost: low range
- Disinfection: difficult. High risk of distortion from disinfection if not
performed correctly.
Due to hydrophilic nature, tend to swell when immersed in water or
disinfectant. Protocol:
spray with 2% gluteraldehyde and time for 10 minutes. Rinse and bone dry
immediately.
+ Comment: Setting reaction not contaminated by latex proteins from
rubber gloves.
+ Comment: Polysulfides are an encellent choice for subperiosteal implant
impressions
because of excellent detail reproduction, low wetting angle, and favorable
rigidity
characteristics when using heavy and light body dual viscosity technique.
- Comment: will not adhere to itself. Can not be used to border mold and for
correctable
impression technique.
Polyether (Impergum, Permadyne)
+ Hydrophilic: moderate. Will still capture accurate impression in the
presence of a little
blood or saliva.
+ Wetting angle: low. Easier to capture a perfect full arch impression with
than with

polyvinyls.
+ Detail reproduction: excellent.
+ Dimensional stability: excellent. Allows multiple pours of accurate casts
for several
weeks after impression.
+/- Rigidity: high. Impressions more difficult to remove than polyvinyls.
Excellent for
implant impressions. Polyethers are very suitable for triple trays.
+ Tear strength: high. Will capture a subgingival impression without tearing
on removal.
Much better than hydrocolloids; better than polyvinyls.
- Taste: bitter. Polyethers fall in to the category of You know it has to be
good because it
tastes so bad.
- Cost: high range $.38 -.84 / ml
- Disinfection: difficult. High risk of distortion from disinfection if not
performed correctly.
Due to hydrophilic nature, tend to swell when immersed in water or
disinfectant. Protocol:
spray with 2% gluteraldehyde and time for 10 minutes. Rinse and bone dry
immediately.
+ Comment: Setting reaction not contaminated by latex proteins from
rubber gloves.
+ Comment: will adhere to itself. Can be used to border mold and for
correctable
impression technique.
Polyvinyl Addition Silicones (Express, Cinch, Aquasil, Splash)
+ Hydrophobic generally. (Aquasil slightly hydrophilic.) Any moisture from
blood or saliva
can prevent an accurate impression
- Wetting angle: moderately high. Makes polyvinyls more difficult to
capture a perfect full
arch impression than hydrocolloid, polysulfide, or polyether.
+ Detail reproduction: excellent.
+ Dimensional stability: excellent. Allows multiple pours of accurate casts
for several
weeks after impression.
+ Rigidity: generally moderate. Impressions remove more easily than
polyethers. Some
polyvinyls have insufficient rigidity to be suitable for triple trays.
+ Tear strength: much better than hydrocolloids; not as good as polyether.
+ Taste is generally neutral which makes it much better than polyether or
polysulfide.

+ Cost: moderate range $.15-.41 / ml


- Disinfection: Excellent. Polyvinyls are virtually impervious to all
disinfection protocols.
Polyvinyls are the only impression materials that may actually be cold
sterilized without
danger of distortion. Disinfection protocol: 30 minute soak in 2%
gluteraldehyde. Cold
sterilization protocol: 8 hour soak in 2% gluteraldehyde.
- Pouring: moderate difficulty. Addition silicones release hydrogen on
setting. Many require
1 to 3 hour de-gassing period before pouring impression or master cast will
have surface
porosity. Greater tendency to trap air bubbles due to moderately high
contact angle. This
means greater care necessary when flowing stone.
- Comment: Contamination by latex proteins from rubber gloves may
interfere with setting
reaction.
- Comment: Most will not adhere to itself after set. Therefore, most cannot
be used to
border mold or for correctable impression technique. Aquasil is an exception.
F. Polyethyl and Polymethyl Methacrylate (Coe Comfort, Coe Soft,
Lynol,
Hydrocast, and Visco-Gel)
Polyethyl and polymethyl methacrylates are commonly used as as tissue
conditioners,
temporary soft-liners, and functional impression materials for removable
prosthodontics.
Tissue Conditioners differ from soft liners because they have a flow period
in which they
continue to adapt to tissues after they have reached their set. Functional
impression
materials have an extended flow period. Hydrocast and Visco-Gel can be
used as excellent
functional impression materials. All are polyethyl or polymethyl methacrylate
materials
combined with an alcohol-based plastisizer. It is the plasitcizer that makes
each unique.
+ Hydrophilic: moderate. Will still capture accurate impression in the
presence of a little
blood or saliva.
+ Wetting angle: low. Very easy to capture a perfect full arch impression.

+ Detail reproduction: moderate. Suitable for tissue impressions for


complete and partial
dentures
+ Dimensional stability: fair. Must be poured within one hour to insure
stability4.
Absolutely only one accurate pour per impression.
+/- Rigidity: low. Methacrylates must have rigid trays that support to within
3 millimeters
of peripheral borders of denture or partial denture or they will distort.
Impressions relatively
easy to remove.
+ Tear strength: low. Will capture an impression well but thin areas have a
tendency to
tear on removal. Not as strong as polyethers or polyvinyls.
+ Taste: neutral, which makes it much better than polyether or polysulfide.
- Cost: moderate range $.38 -.84 / ml
- Disinfection: moderate. Protocol: soak in 8% hydrogen peroxide for 8
minutes. Rinse and
dry prior to pouring. Will not distort due to water absorption. However, due to
its alcohol
base, methacrylates will distort easily when exposed to alcohol based
disinfectants such as
Lysol.
+ Comment: This is an excellent impression material for reline or rebase of
removable
prosthetics. Once relined and trimmed, the patient wears the prosthesis for
one day to two
weeks (depending on the product) producing a perfectly molded functional
impression. Each
version is unique due to its plastisizers. Methacrylates are sticky in their
working phase and
there is definitely a learning curve in order to use effectively.
+ Comment: will adhere to itself. Can be used to border mold and for
correctable
impression technique. Acrylic additives are available to stabilize extended
borders of
impression if necessary.
Coe Comfort: a true tissue conditioner. It has longer adaptive flow (1 to 2
days) than Coe
Soft, but will need to be replaced every 3 to 4 days.
Coe Soft: also a true tissue conditioner. It will flow for 2 to 4 hours and will
need to be
replaced in one week. It is more resistant to stain and odor than Coe
Comfort.
Caulk Lynol: is not a tissue conditioner; it is a temporary soft-liner. It trades
moldability

for longevity. It only flows for about 30 minutes. However it has a flexible
life span of
about 2 3 weeks. Use Lynol for patients who need a cushion for longer than
one week, like
patients with immediate or transitional dentures.
Hydrocast (Kaysee Dental): is a unique product that has extended flow
characteristics for
two to four weeks and will retain its resiliency for 2 - 3 months. It may be
used as a tissue
conditioner, a soft-liner, and a final impression for the tissue surface of a
complete or partial
denture. Its primary disadvantage is that it handles differently than the other
3 temporary
soft-liners and there is a learning curve in order to use it successfully.
Visco-Gel: is a product similar to Hydrocast, but with modified plastisizers
that shorten its
flow characteristics to 3 to 7 days and its resilient period to one month. It has
a shortened
working time as compared to Hydrocast and is slightly less sticky.
Conclusion
The choice of impression material is often a very personal choice base
mainly on personal
preference and experience. It is however important to remember the
characteristics of the
material. A skilled dentist will use a variety of impression materials
depending on the
purpose of the impression and the specific condition of the impression site.
Ref.
www.modental.org/docs/events/ce/ceimpression.pdf

A patient wearing a new bilateral RPD complains of soreness on the tissue


bearing ridge crest
areas 24 hours after insertion. The most likely cause would be?
a. Occlusal discrepancies
b. Over extended denture
c. Under extended denture
d. Excessive vertical dimension of occlusion
e. Nutrition or hormonal problems

A patient wearing a new bilateral RPD complains of soreness on the tissue


bearing ridge crest
areas 24 hours after insertion. The most likely cause would be?
a. Occlusal discrepancies
b.
c.
d.
e.
Generalized irritation or soreness of the basal seat may be attributable to
any of the above factors, but is most likely due to errors in occlusion.
Ref.:
Zarb and Bolender. (2004). Prosthodontic treatment for edentulous patients.
12th Ed. P. 425-6
www.usc.edu/hsc/dental/Resources/Courseware

Clasps should be so designed that, upon insertion or removal of a removable


partial denture, the reciprocal arms contact the abutment teeth when the
retentive arms engage the height of contour in order to?
a.
b.
c.
d.

Assure complete seating of the framework


Permit insertion and removal without applying excessive force
Prevent distortion of the clasps
All of the above

Clasps should be so designed that, upon insertion or removal of a removable


partial denture, the reciprocal arms contact the abutment teeth when the
retentive arms engage the height of contour in order to?
a.
b. Permit insertion and removal without applying excessive force
c.
d.

Reciprocation of clasps is related to the timing of contact between the


bracing and retentive clasp arms in order to prevent undue forces on the
abutment teeth during seating and removal of the RP. In order to provide
true reciprocation, the timing must be correct. The bracing arm must be in
contact with the abutment as the retentive arm passes over the bulge and
into the undercut.

Which statement regarding denture phonetics is CORRECT?


a. Whistling results when the space between the tongue and palate is too
large
b. When anterior teeth are set too far lingually T sounds like D
c. Plosive sounds such as P and B are made by the tongue contacting
the lips
d. If the maxillary incisors are too short, F sounds like V

Which statement regarding denture phonetics is CORRECT?


a.
b. When anterior teeth are set too far lingually T sounds like D
c.
d.

Whistling is generated when the opening between the tongue and palate is
too small, when the posterior arch form is too narrow and when the
mandibular anterior teeth are set too far lingually.
When the anterior teeth are set too far lingually, T sounds like D. When
the anterior teeth are set too far labially, D sounds like T. If the denture
base is too thick in the rugae area, D sounds like T.
Plosive sounds are made exclusively with the lips. Insufficient lip support will
result in defective plosive sounds.
F V and Ph are the best guide for the anteroposterior positioning of the
anterior teeth. If the incisors are too short, V sounds like F. If the
incisors are too long, F sounds like V. If the upper teeth touch the labial
side of the lower lip, maxillary incisors are too labial or mandibular incisors
are too lingual.
Ref.
Rah, Heartwell. Textbook of complete dentures. 5th edition. page 466

Which one of the following impression materials is elastic, sets by a physical


reaction, and is subject to syneresis and imbibition?
a.
b.
c.
d.

Irreversible Hydrocolloid
Polysulfide Rubber
Condensing Silicone
Polyether

Which one of the following impression materials is elastic, sets by a physical


reaction, and is subject to syneresis and imbibition?
a. Irreversible Hydrocolloid
b.
c.
d.
Syneresis. This is a process whereby water is forced out onto the surface of
the impression as the gel molecules are drawn closer together, with the main
driving force being the relief of internal stresses. The water evaporates from
the surface and causes the impression material to shrink
Imbibition. This is the uptake of water that occurs if the material has
become dry, possibly due to inadequate storage technique. Distortion of the
impression will result if this occurs, as the internal stresses that are always
present are relieved during this process
Alginates
The alginates are based on alginic acid, which is derived from a marine plant.
The structure of alginoc acid is quite complex. Some of the hydrogen
molecules in the carboxyl groups are replaced by sodium, thus forming a
water soluble salt with a molecular weight of 20,000 200,000. The setting
process in this material is by the creation of crosslinks between the polymer
chains of sodium alginate.
Composition of a typical alginate impression material:
Component
Sodium alginate
18
Calcium sulphate dehydrate
ions
Sodium phosphate
time
Potassium sulphate
Fillers (diatomaceous earth)
consistency
Sodium silicofluoride

Amount %
14

Purpose
Hydrogen former
Provides calcium

Controls working

10
56

Setting of model
Controls

Controls pH

Ref.
Noort VR, Introduction to dental materials 2nd edition. P 188-191

Which of the following is not an acceptable impression technique for a


RPD final impression?
a.
b.
c.
d.
e.

Anatomic
Functional
Selected Pressure
Hydrostatic
Mucostatic

Which of the following is not an acceptable impression technique for a


RPD final impression?
a.
b.
c.
d. Hydrostatic
e.
Anatomic: useful to capture Hard tissues in an unloaded state
Functional: Developed by pts own anatomy and functional movements.
Selected pressure: Custom tray designed to selectively load primary and
secondary denture bearing areas.
Mucostatic: Captures intimate soft tissue detail of very healthy, unloaded
keratinized tissues by using a tray which contains but not pressurizing the
free flowing- fast setting, rigid set material.
Impression options:
Stock Tray and Alginate
Custom tray and border mold
Polysulfide and PVS most commonly used.
For severe undercuts or lone standing teeth:
Rubber Base: use Light/Medium body.
PVS: Heavy/Med body: use spacer, and then wash with light body.
Selective pressure design:
Wax placed over all denture bearing areas and is then trimmed away from
primary support areas.
Maxillary
Primary support area: Posterior Alveolar Ridges
Secondary support area:
Anterior Alveolar Ridges & Rugae*
* susceptible to resorption so ensure block-out (mushroom shaped wax)
Mandibular
Primary support area: Buccal Shelf
Secondary support area:
Posterior and Anterior Alveolar Ridges
Ref.:
McCraken. Removable Partial Prosthodontics. 10th ed. (2000) pp. 342-56.

In taking a final impression on a patient with obvious undercuts, which of


the following materials would be contraindicated?
a.
b.
c.
d.
e.

Alginate
Reversible Hydrocolloid
Irreversible Hydrocolloid
Polyvinylsiloxane
Polyether

In taking a final impression on a patient with obvious undercuts, which of the


following materials would be contraindicated?
a.
b.
c.
d.
e. Polyether
Rigid materials: to record mobile soft tissue
Plaster of Paris
Metal Oxide-ZOE
Thermoplastic Material:
Modeling Plastic (compound)
Impression Waxes and resins:
Iowa/ Korrecta Wax:
1) flows at mouth temperature,
2) Useful for Functional Impression
Elastic Material:
Reversible Hydrocolloid
Irreversible Hydrocolloid
Low tear strength, less surface detail, not dimensionally stable w/ time,
hydrophilic, cheap
Polysulfide (Rubber Base)
3mm thick required, High tear strength, good bordermold, pour within 1
hour.
For severe undercuts or lone standing teeth: use Light/Medium body
rubber base
Polyether (Impregum)
Accurate, good detail and border molding. Stiff!! (Maybe great for triple
trays), can break casts- Not good for pts with undercuts! Hydrophobic.
Silicones- condensation or addition (Polyvinylsiloxane)
Affected by latex and ferric sulfate. Low distortion. Hydrophilic and
hydrophobic. Pour up to a week later (or more). Low distortion/shrinkage.
Ref.:
McCraken. Removable Partial Prosthodontics. 10th ed. (2000) pp. 227-8,
315-35.

Ideally, the deepest portion of an occlusal rest should be?


a.
b.
c.
d.
e.

1mm
1.5mm
2mm
2.5mm
3mm

Ideally, the deepest portion of an occlusal rest should be?


a.
b.
c.
d. 2.5mm
e.

Types of Rest:
1. Occlusal rest (see below)
2. Occlusal Onlay (aka hooded rest) (onlay has 1-2mm bevel over cuspsconsider mesial guide planes on these teeth.
3. Embrasure rest: 2 adjacent teeth- to avoid wedging, open to lingual to
allow for metal bulk. Want 1.5mm reduction- dont break contact.
4. Cingulum rest- cuspids preferred, curved shape (apex towards incisal) MD: 2.5-3mm. F-L: 2mm w 1.5mm depth. Keep closer to gingiva. If perforation
thru enamel expected- build up using cast metal or composite
5. Incisal rest: unesthetic, maybe good for taking altered cast impressions to
prevent framework rotation (then cut off prior to final delivery)
Occlusal rest seats should be triangular in shape with the base of the triangle
located at the marginal ridge and the apex pointing toward the center of the
tooth.
All features of the rest seat should be rounded.
Reduction at the marginal ridge should be at least 1mm, resulting in a rest
that is at least 1mm thick at its thinnest point. (Ideally 1.5mm)
The angle formed by the floor of the rest seat and the proximal surface
should be less than 90 degrees to avoid an inclined plane situation. Floor of
occ rest should be below marginal ridge.
Caution needs to be taken not to create undercuts with a round bur when
making rest seat preparations (see pictures).
Guide planes are created prior to rest seat preparation. Cutting a guide
plane in the proximal surface would remove part of the rest seat if it is done
after rest seat preparation.

Potrebbero piacerti anche